Sie sind auf Seite 1von 190

Aritmetica II

PROFMAT
Fabio E. Brochero Martinez
Carlos Gustavo T. de A. Moreira Nicolau C. Saldanha
16 de Marco de 2012
Conte udo
0 Fundamentos 2
0.1 Princpio da Indu cao Finita . . . . . . . . . . . . . . . . . 2
0.2 Princpio da Casa dos Pombos . . . . . . . . . . . . . . . 9
0.3 Divisibilidade . . . . . . . . . . . . . . . . . . . . . . . . . 13
0.4 mdc, mmc e Algoritmo de Euclides . . . . . . . . . . . . . 16
0.5 O Teorema Fundamental da Aritmetica . . . . . . . . . . 24
0.6 Congruencias . . . . . . . . . . . . . . . . . . . . . . . . . 31
0.7 Bases . . . . . . . . . . . . . . . . . . . . . . . . . . . . . 34
0.8 O Anel de Inteiros M odulo n . . . . . . . . . . . . . . . . 37
0.9 A Funcao de Euler e o Teorema de Euler-Fermat . . . . . 43
0.10 Equacoes Lineares M odulo m . . . . . . . . . . . . . . . . 53
1 Potencias e Congruencias 61
1.1 Polin omios . . . . . . . . . . . . . . . . . . . . . . . . . . 61
1.2 Ordem e Razes Primitivas . . . . . . . . . . . . . . . . . . 73
1.3 Resduos Quadraticos e Smbolo de Legendre . . . . . . . 81
1.3.1 Lema de Gau . . . . . . . . . . . . . . . . . . . . 84
1.4 Lei de Reciprocidade Quadratica . . . . . . . . . . . . . . 88
1.4.1 Uma demonstra cao combinat oria . . . . . . . . . . 90
1.4.2 Uma demonstra cao trigonometrica . . . . . . . . . 92
1.4.3 Uma demonstra cao usando corpos nitos . . . . . 94
2 Func oes Multiplicativas e as f ormulas de inversao de Mo-
bius 99
2.1 As funcoes d, e . . . . . . . . . . . . . . . . . . . . . . 102
2.2 Funcao de Mobius e Formula de Inversao . . . . . . . . . 110
3 Fra c oes Contnuas 118
3.1 Reduzidas e Boas Aproximacoes . . . . . . . . . . . . . . 127
3.2 Boas Aproximacoes s ao Reduzidas . . . . . . . . . . . . . 129
3.3 Fra coes Contnuas Peri odicas . . . . . . . . . . . . . . . . 134
3.4 O espectro de Lagrange . . . . . . . . . . . . . . . . . . . 135
CONTE

UDO
4 Equa c oes diofantinas nao lineares 139
4.1 Teorema de Pit agoras e triplas Pitagoricas . . . . . . . . . 139
4.2 Tri angulos ret angulos de Pit agoras e Platao . . . . . . . . 141
4.3 Triplas Pitagoricas Primitivas . . . . . . . . . . . . . . . . 143
4.4 Tri angulos pitagoricos e o metodo geometrico . . . . . . . 146
4.5 Tri angulos com lados inteiros e angulos em progress ao
aritmetica . . . . . . . . . . . . . . . . . . . . . . . . . . . 149
4.6 Outra rela cao de angulos . . . . . . . . . . . . . . . . . . . 151
4.7 Contando tri angulos pitagoricos com um cateto dado . . . 153
4.8 N umeros que s ao somas de dois quadrados . . . . . . . . . 155
4.9 Tri angulos pitagoricos com catetos consecutivos . . . . . . 159
4.9.1 Equacao de Pell . . . . . . . . . . . . . . . . . . . 159
4.10 Solucao fundamental da equa cao de Pell . . . . . . . . . . 164
4.11 Outras equa coes do tipo x
2
Ay
2
= c . . . . . . . . . . . 167
4.12 Contando tri angulos pitagoricos com hipotenusa xada . . 169
4.12.1 Inteiros de Gau . . . . . . . . . . . . . . . . . . . 170
4.13 Descenso Innito de Fermat . . . . . . . . . . . . . . . . . 178
Bibliograa 185
CONTE

UDO 1
Introdu cao
Aritmetica II - PROFMAT
Nosso texto para a disciplina Aritmetica II visa tratar de temas
elementares e interessantes da Teoria dos N umeros. Assim, pretendemos
que este livro seja acessvel e nao dependa de pre-requisitos sosticados
de algebra ou an alise, mas que apresente ao leitor resultados relevantes
de topicos como equa coes diofantinas de grau 2, funcoes aritmeticas e
aproximacoes diofantinas.
Come camos com uma revisao de fundamentos de aritmetica (que
constam do programa do curso de Aritmetica I do PROFMAT). A seguir,
no Captulo 1, tratamos de ordens, razes primitivas, resduos quadra-
ticos e da lei de Gauss da reciprocidade quadratica, um dos resultados
mais importantes e surpreendentes da teoria elementar dos n umeros.
No captulo seguinte tratamos de um tipo particularmente importante
de funcoes aritmeticas: as funcoes multiplicativas, e de um importante
topico relacionado: as formulas de inversao de Mobius. No Captulo 3
apresentamos a teoria elementar das Fra coes Contnuas, que fornecem
as melhores aproximacoes racionais de um n umero real dado, fazendo,
dessa forma, uma introducao `a teoria das Aproximacoes Diofantinas.
Finalmente, no Captulo 4, tratamos de Equacoes Diofantinas de grau
2, para as quais ha diversos resultados cl assicos, bonitos e importan-
tes, comecando com o estudo das triplas pitagoricas, discutindo ideias
geometricas e tratando da equa cao de Pell, relacionada com fra coes con-
tnuas e aproximacoes diofantinas.
Esperamos estimular os leitores, especialmente os professores de ma-
tematica do ensino medio, a se interessarem pela Teoria dos N umeros, e
a prosseguirem seus estudos sobre o tema. Observem que inclumos exer-
ccios rotineiros mas tambem varios problemas bastante difceis, muitos
deles de olimpadas de matem atica. Estimulamos o leitor a tentar fazer
todos os problemas mas com a advertencia que ele nao deve se sentir
frustrado se mesmo com esforco nao conseguir resolver alguns dos pro-
blemas: muitos deles s ao difceis tambem para os autores.
Agradecemos a todos os que nos ajudaram ao longo da elabora cao
com sugestoes variadas. Mas devemos um agradecimento muito especial
a nosso amigo e colega Eduardo Tengan. ET e nosso co-autor no livro de
Teoria dos N umeros publicado pelo projeto Euclides. O livro do Projeto
Euclides serviu de ponto de partida para a elabora cao deste livro e nos
tres achamos natural que ET tambem fosse contado como co-autor desta
vez: apenas por insistencia dele aceitamos preparar o livro sem o nome
dele. De qualquer forma devemos registrar aqui que sem a participacao
de ET nao existiriam nem o livro do Projeto Euclides nem este.
Captulo 0
Fundamentos
Neste captulo preliminar recordaremos alguns topicos fundamentais:
o Princpio da Inducao Finita e o Princpio da Casa dos Pombos, MDC,
Congruencias e os Teoremas de Fermat e de Euler.
0.1 Princpio da Inducao Finita
Seja P(n) uma propriedade do n umero natural n, por exemplo:
n pode ser fatorado em um produto de n umeros primos;
1 + 2 + +n =
n(n+1)
2
;
a equa cao 2x+3y = n admite solucao com x e y inteiros positivos.
Uma maneira de provar que P(n) e verdadeira para todo natural
n n
0
e utilizar o chamado Princpio da Inducao Finita (PIF), que e
um dos axiomas que caracterizam o conjunto dos n umeros naturais. O
PIF consiste em vericar duas coisas:
1. (Base da Indu cao) P(n
0
) e verdadeira e
2. (Passo Indutivo) Se P(n) e verdadeira para algum n umero natural
n n
0
, entao P(n + 1) tambem e verdadeira.
Na base da indu cao, vericamos que a propriedade e valida para
um valor inicial n = n
0
. O passo indutivo consiste em mostrar como
utilizar a validade da propriedade para um dado n (a chamada hip otese
de inducao) para provar a validade da mesma propriedade para o inteiro
seguinte n + 1. Uma vez vericados a base e o passo indutivo, temos
2
0.1. PRINC

IPIO DA INDU C

AO FINITA 3
uma cadeia de implicacoes
P(n
0
) e verdadeira (base)
passo
indutivo
= P(n
0
+ 1) e verdadeira
passo
indutivo
= P(n
0
+ 2) e verdadeira
passo
indutivo
= P(n
0
+ 3) e verdadeira
.
.
.
de modo que P(n) e verdadeira para todo natural n n
0
.
Vejamos alguns exemplos.
Exemplo 0.1. Demonstrar que, para todo inteiro positivo n,
1 + 2 + +n =
n(n + 1)
2
.
Solu c

ao: Observemos que 1 =


12
2
donde a igualdade vale para n = 1
(base da indu cao). Agora suponha que a igualdade valha para n = k
(hip otese de indu cao):
1 + 2 + +k =
k(k + 1)
2
Somando k + 1 a ambos lados da igualdade, obtemos
1 + 2 + +k + (k + 1) =
k(k + 1)
2
+ (k + 1) =
(k + 1)(k + 2)
2
de modo que a igualdade tambem vale para n = k + 1. Pelo PIF, a
igualdade vale para todo n umero natural n 1.
Exemplo 0.2. Demonstrar que, para todo n umero natural n,
M
n
= n(n
2
1)(3n + 2)
e m ultiplo de 24.
Solu c

ao: Veja que se n = 0 entao M


0
= 0, que e um m ultiplo de 24
(base da indu cao).
Agora, suponhamos que para certo inteiro k o n umero M
k
e divis-
vel por 24 (hip otese de indu cao) e vamos mostrar que M
k+1
tambem e
divisvel por 24 (passo indutivo). Calculamos primeiramente a diferen ca
M
k+1
M
k
= (k + 1)
_
(k + 1)
2
1
__
3(k + 1) + 2
_
k(k
2
1)(3k + 2)
= k(k + 1)[(k + 2)(3k + 5) (k 1)(3k + 2)]
= 12k(k + 1)
2
.
4 CAP

ITULO 0. FUNDAMENTOS
Um dos n umeros naturais consecutivos k e k + 1 e par donde k(k + 1)
2
e sempre par e 12k(k + 1)
2
e divisvel por 24. Por hip otese de indu cao,
M
k
e divisvel por 24 e temos portanto que M
k+1
= M
k
+ 12k(k + 1)
2
tambem e divisvel por 24, como se queria demonstrar.
Uma variante do PIF e a seguinte versao (`as vezes apelidada de
princpio de inducao forte ou princpio de inducao completa), em que
se deve mostrar
1. (Base da Indu cao) P(n
0
) e verdadeira e
2. (Passo Indutivo) Se P(k) e verdadeira para todo natural k tal que
n
0
k n, entao P(n + 1) tambem e verdadeira.
Exemplo 0.3. A sequencia de Fibonacci F
n
e a sequencia denida re-
cursivamente por
F
0
= 0, F
1
= 1 e F
n
= F
n1
+F
n2
para n 2
Assim, seus primeiros termos sao
F
0
= 0, F
1
= 1, F
2
= 1, F
3
= 2, F
4
= 3, F
5
= 5, F
6
= 8, . . .
Mostre que
F
n
=

n

n

onde =
1+

5
2
e =
1

5
2
sao as razes de x
2
= x + 1.
Solu c

ao: Temos que F


0
=

0

= 0 e F
1
=

1

= 1 (base de
indu cao). Agora seja n 1 e suponha que F
k
=

k

para todo k com


0 k n (hip otese de indu cao). Assim,
F
n+1
= F
n
+F
n1
=

n

n

+

n1

n1

=
(
n
+
n1
) (
n
+
n1
)

=

n+1

n+1

pois
2
= + 1 =
n+1
=
n
+
n1
e analogamente
n+1
=

n
+
n1
.
Observe que, neste exemplo, como o passo indutivo utiliza os valores
de dois termos anteriores da sequencia de Fibonacci, a base requer veri-
car a formula para os dois termos iniciais F
0
e F
1
e nao apenas para o
primeiro termo.
0.1. PRINC

IPIO DA INDU C

AO FINITA 5
Exemplo 0.4. Prove usando o princpio da inducao matematica que
1 3 5 (2n 1)
2 4 2n

1

2n + 1
.
Solu c

ao: Vejamos que se n = 1 temos que


1
2
<
1

3
. Suponhamos que
para um certo n temos
1 3 5 (2n 1)
2 4 2n

1

2n + 1
.
Para o caso seguinte temos que
1 3 5 (2n 1) (2n + 1)
2 4 2n (2n + 2)
H.I.
<
1

2n + 1

2n + 1
2n + 2
,
e assim basta mostrar que
1

2n + 1

2n + 1
2n + 2

1

2n + 3
. Isso sua vez
e equivalente a mostrar que

2n + 1
2n + 2

1

2n + 3
, mas, elevando ao
quadrado, tambem vai ser equivalente a mostrar (2n + 1)(2n + 3)
(2n + 2)
2
, o que (expandindo os dois lados) e claramente verdadeiro, o
que conclui a prova.
Exemplo 0.5. Demonstrar que, para quaisquer naturais n m, o coe-
ciente binomial
_
n
m
_
def
=
n!
m!(n m)!
e inteiro.
Solu c

ao: Procederemos por indu cao sobre a soma m+n. Se m+n = 0


entao m = n = 0 e
_
0
0
_
= 1 e inteiro (base de indu cao). Para o passo
indutivo, observe primeiramente que para 0 < m < n temos a seguinte
identidade de binomiais
_
n
m
_
=
_
n 1
m
_
+
_
n 1
m1
_
que segue diretamente das denicoes:
_
n 1
m
_
+
_
n 1
m1
_
=
(n 1)!
m!(n m1)!
+
(n 1)!
(m1)!(n m)!
=
_
(n m) +m
_
(n 1)!
m!(n m)!
=
_
n
m
_
.
Agora suponhamos que
_
n
m
_
e inteiro para m + n k (hip otese de
indu cao). Note que podemos supor tambem que 0 < m < n, ja que se
m = n ou m = 0 temos
_
n
m
_
= 1 e o resultado vale trivialmente. Assim,
se m+n = k +1, temos que
_
n
m
_
=
_
n1
m
_
+
_
n1
m1
_
e inteiro tambem pois
cada somando da direita e inteiro pela hip otese de indu c ao.
6 CAP

ITULO 0. FUNDAMENTOS
Exemplo 0.6. Sejam x
1
, . . . , x
n
n umeros reais positivos. Neste exerc-
cio vamos provar que
x
1
+ +x
n
n

n

x
1
x
n
.
Tal desigualdade e conhecida como desigualdade das medias aritmetica
e geometrica.
(a) Utilize o PIF para mostrar a desigualdade das medias para n = 2
k
.
(b) Sejam x
1
, . . . , x
n
reais positivos xados e A =
x
1
++x
n
n
a media arit-
metica destes n umeros. Suponha que a desigualdade valha para n+1
n umeros reais positivos quaisquer; aplicando-a para x
1
, . . . , x
n
, A,
conclua que a desigualdade vale tambem para quaisquer n n umeros
reais positivos.
(c) Combinando os itens anteriores, prove a desigualdade para todo n
natural.
Solu c

ao: Primeiro observemos que se a, b > 0 entao


0 (a b)
2
= a
2
2ab +b
2
= (a +b)
2
4ab,
logo

ab
a+b
2
, assim a desigualdade vale para n = 2. Agora mostremos
que se a desigualdade vale para k entao a desigualdade vale para 2k. De
fato
x
1
+ +x
2k
2k
=
x
1
++x
k
k
+
x
k+1
++x
2k
k
2

x
1
x
k
+
k

x
k+1
x
2k
2

_
k

x
1
x
k
k

x
k+1
x
2k
=
2k

x
1
x
2k
.
Assim a desigualdade e verdadeira para 2, 4, 8, . . . , 2
n
, . . .. Suponhamos
que a desigualdade e verdadeira para n + 1, e sejam x
1
, . . . , x
n
reais
positivos, denamos A =
x
1
++x
n
n
e G =
n

x
1
x
n
, temos que mostrar
que A G, mas de fato sabemos que
A =
x
1
+ +x
n
+A
n + 1

n+1
_
x
1
x
n
A = G
n
n+1
A
1
n+1
.
Daqui facilmente conclumos o que queramos demonstrar.
0.1. PRINC

IPIO DA INDU C

AO FINITA 7
Um terceiro disfarce do PIF e o chamado princpio da boa ordena-
cao (PBO) dos n umeros naturais, que arma que todo subconjunto A
nao vazio de N tem um elemento mnimo. (Voce sabe dizer por que o
princpio da boa ordem nao vale para o conjunto Z de todos os inteiros?)
Vejamos a equivalencia entre os dois princpios. Assuma primeira-
mente o PBO e seja P(n) uma propriedade para a qual P(0) e verdadeira
e P(n) verdadeira implica P(n + 1) verdadeira. Seja B o conjunto dos
n tais que P(n) e falsa; devemos mostrar que B = . Suponha que nao;
pelo PBO o conjunto B possui um menor elemento b. Como 0 / B (pois
P(0) e verdadeira por hip otese) temos que b 1 e assim b1 N e pela
minimalidade de b temos que b 1 / B, ou seja, P(b 1) e verdadeira.
Mas por hip otese temos entao que P(b) tambem e verdadeira, o que e
um absurdo, logo B = .
Assuma agora o PIF e seja A N um subconjunto nao vazio. Dena
agora o conjunto B = {b N | a / A para todo a < b}. Trivialmente
0 B. Armamos que existe k B tal que k + 1 / B e nesse caso k
sera o menor elemento de A. De fato, se isto nao acontecer, teremos que
0 B e k B implica que k + 1 B. Logo, pelo PIF, B = N e A = ,
o que e absurdo.
Exemplo 0.7. Demonstrar que toda fun cao f : N N monotona n ao-
crescente (isto e, n m = f(n) f(m)) e constante a partir de um
certo n umero natural.
Solu c

ao: Seja A N a imagem de f. Pelo PBO, tal conjunto possui


elemento mnimo a
0
. Seja n
0
um natural tal que f(n
0
) = a
0
. Como
a funcao e monotona nao-crescente entao para todo n n
0
temos que
f(n) f(n
0
), mas pela denicao de a
0
temos f(n) a
0
. Logo f(n) = a
0
para todo n n
0
, como queramos demonstrar.
Observa cao 0.8. Dado um conjunto S, uma rela cao em S e chamada
de ordem parcial em S se ela satisfaz os seguintes axiomas:
1. (Reexividade) a a para todo a S.
2. (Anti-simetria) se a b e b a ent ao a = b.
3. (Transitividade) se a b e b c ent ao a c.
Dizemos que e uma ordem total se, dados quaisquer a, b S, ou
a b ou b a. Uma ordem total em S e uma boa ordem se todo
subconjunto A de S possui um elemento mnimo, isto e, um elemento
a A tal que a b para todo b A.

E possvel demonstrar que para
todo conjunto S podemos denir uma ordem total em S que e uma boa
ordem. Este fato usa o axioma da escolha (e na verdade e equivalente a
ele) e esta fora do proposito deste livro. Veja por exemplo [15].
8 CAP

ITULO 0. FUNDAMENTOS
Problemas Propostos
0.1. Demonstrar por inducao que para n 1 natural
(a) 1
2
+ 2
2
+ +n
2
=
n(n+1)(2n+1)
6
.
(b) 1
3
+ 2
3
+ +n
3
= (1 + 2 + +n)
2
.
(c) (1
5
+ 2
5
+ +n
5
) + (1
7
+ 2
7
+ +n
7
) = 2(1 + 2 + +n)
4
.
(d) sen x + sen 2x + + sen nx =
sen
(n+1)x
2
sen
nx
2
sen
x
2
.
0.2. Seja F
n
o n-esimo termo da sequencia de Fibonacci. Demonstrar
que para todo natural n 1 temos
(a) F
1
+F
2
+ +F
n
= F
n+2
1.
(b) F
n+1
F
n1
F
2
n
= (1)
n
.
(c)
_
1 1
1 0
_
n
=
_
F
n+1
F
n
F
n
F
n1
_
.
(d)
_
n
0
_
+
_
n1
1
_
+
_
n2
2
_
+
_
n3
3
_
+ = F
n+1
, onde na soma interpretamos
_
m
k
_
= 0 se k > m.
0.3. Demonstrar que
(a) n
3
n e um m ultiplo de 6 para todo natural n.
(b) 5
n
1 e m ultiplo de 24 para todo n umero natural n par.
(c) 2
n
+ 1 e m ultiplo de 3 para todo natural mpar n.
0.4. Mostre que para todo natural n 4
(a) 2
n
< n!.
(b) 2n
3
> 3n
2
+ 3n + 1.
0.5. Dado um inteiro positivo n, denimos T(n, 1) = n e, para todo
k 1, T(n, k + 1) = n
T(n,k)
. Prove que existe c N tal que, para todo
k 1, T(2010, k) < T(2, k + c). Determine o menor inteiro positivo c
com essa propriedade.
0.6. Mostre que para todo n e k inteiros positivos
_
n
n
_
+
_
n + 1
n
_
+
_
n + 2
n
_
+ +
_
n +k
n
_
=
_
n +k + 1
n + 1
_
.
0.2. PRINC

IPIO DA CASA DOS POMBOS 9


0.7. Demonstre a formula do bin omio de Newton para n natural:
(x +y)
n
=
_
n
0
_
x
n
+
_
n
1
_
x
n1
y + +
_
n
n 1
_
xy
n1
+
_
n
n
_
y
n
.
0.8. Encontrar com demonstracao uma expressao para o multin omio
(x
1
+x
2
+ +x
k
)
n
em termos dos coecientes multinomiais
_
n
i
1
, . . . , i
k
_
def
=
n!
i
1
! i
k
!
onde i
1
+ +i
k
= n.
0.9. Considere n retas em posicao geral em um plano, isto e, sem que
haja duas retas paralelas ou tres retas concorrentes em um mesmo ponto.
(a) Determine em fun cao de n o n umero de regioes em que as retas
dividem o plano.
(b) Demonstre que e possvel colorir essas regioes com duas cores sem
que duas regioes vizinhas tenham a mesma cor (duas regioes sao
vizinhas se elas possuem um segmento de reta em comum).
0.10. Demonstrar que para cada n umero natural n existe um n umero
natural M satisfazendo simultaneamente as seguintes duas condicoes:
(i) M possui n dgitos pertencentes ao conjunto {1, 2}.
(ii) M e divisvel por 2
n
.
0.11 (IMO1987). Mostre que n ao existe uma fun cao f : N N tal que
f(f(n)) = n + 1987 para todo n N.
0.2 Princpio da Casa dos Pombos

E intuitivamente claro que se colocamos n + 1 objetos em n gavetas


entao havera ao menos uma gaveta com mais de um objeto. Isto e
exatamente o que arma o chamado Princpio da Casa dos Pombos
(PCP) ou Princpio das Gavetas de Dirichlet: se temos kn + 1 pombos
e n casinhas, entao existir a uma casinha onde havera pelo menos k + 1
pombos. De fato, se em todas as casas houvesse no m aximo k pombos,
entao o n umero de pombos nao poderia ultrapassar kn.
O PCP parece bastante inocente, mas tem muitas aplicacoes inte-
ressantes, especialmente em argumentos de existencia em que nao se
determina o objeto procurado explicitamente. Como exemplos falam
mais do que 10
3
palavras, vejamos alguns.
10 CAP

ITULO 0. FUNDAMENTOS
Exemplo 0.9. Do conjunto A = {1, 2, . . . , 99, 100}, escolhemos ao acaso
51 n umeros. Demonstrar que entre os n umeros escolhidos sempre exis-
tem dois que sao consecutivos.
Solu c

ao: Para provar isto, primeiro escolhamos gavetas adequadas


ao problema. Distribumos os n umeros de A em 50 gavetas assim
construdas:
{1, 2} {3, 4} {5, 6} {99, 100}
Como ha 50 gavetas das quais retiramos 51 n umeros, sempre existir a
uma gaveta da qual escolhemos dois n umeros e estes, gra cas `a nossa
construcao, serao consecutivos. Podemos generalizar este resultado con-
siderando os n umeros {1, 2, . . . , 2n} e escolhendo dentre eles n + 1 n u-
meros ao acaso.
Exemplo 0.10. Do conjunto A = {1, 2, . . . , 99, 100}, escolhemos ao
acaso 55 n umeros. Demonstrar que entre os n umeros escolhidos sempre
existem dois tais que sua diferen ca e 9.
Solu c

ao: Como no exemplo anterior o problema e descobrir como


formar as gavetas. Consideremos as gavetas numeradas 0, 1, 2, . . . , 8,
onde o n umero n e colocado na gaveta i se, e s o se, o resto na divisao
de n por 9 e i. Como escolhemos 55 = 9 6 + 1 n umeros, pelo PCP
existir a uma gaveta j na qual ha 7 ou mais n umeros escolhidos. Mas
em cada gaveta ha no m aximo 12 n umeros (por exemplo, o conjunto
{1, 10, 19, 28, 37, 46, 55, 64, 73, 82, 91, 100} possui exatamente 12 elemen-
tos). Segue, como no problema anterior, que existir ao dois n umeros que
serao consecutivos em tal conjunto, isto e, dois n umeros cuja diferen ca
e 9.
Exemplo 0.11. Demonstrar que qualquer conjunto de n inteiros possui
um subconjunto n ao vazio cuja soma dos elementos e divisvel por n.
Solu c

ao: Sejam a
1
, a
2
, . . . , a
n
os elementos do conjunto, e denamos
as somas parciais s
j
= a
1
+ + a
j
para j = 1, . . . , n. Se algum dos
s
j
e divisvel por n o problema ca resolvido. Se nenhum e divisvel
por n, entao os possveis restos na divisao por n s ao 1, 2, . . . , n 1 e
como ha n somas parciais pelo PCP existem duas s
j
e s
k
com j < k que
deixam o mesmo. Portanto s
k
s
j
= a
j+1
+ +a
k
e divisvel por n e
{a
j+1
, a
j+2
, . . . , a
k
} e o subconjunto procurado.
Por outro lado, observemos que n e a quantidade mnima de ele-
mentos para que se verique tal condi cao, no sentido em que existem
0.2. PRINC

IPIO DA CASA DOS POMBOS 11


conjuntos A com n 1 elementos tais que a soma dos elementos de
todo subconjunto nao vazio de A nao e divisvel por n. Por exemplo,
A = {1, n+1, 2n+1, . . . , (n1)n+1} e um destes conjuntos (verique!).
Exemplo 0.12. Seja um n umero real. Demonstrar que, para todo
inteiro n 2, existe um inteiro 0 < k < n tal que o modulo da diferen ca
entre k e seu inteiro mais pr oximo e menor ou igual a
1
n
.
Solu c

ao: Vamos denotar por {x} a parte fracionaria do n umero real


x, isto e, o unico real que satisfaz 0 {x} < 1 e x = m + {x} para
algum m Z.
Considere {k} para k = 1, 2, . . . , n. Particione o intervalo [0, 1) em
n partes de tamanho
1
n
:
[0, 1) =
_
0,
1
n
_

_
1
n
,
2
n
_

_
2
n
,
3
n
_

_
n 1
n
, 1
_
Se {k} [0,
1
n
) ou {k} [
n1
n
, 1) para algum k = 1, . . . , n 1, o
problema acabou. Caso contrario, pelo PCP havera duas partes fracio-
narias {j} e {k} com 1 j < k n 1 pertencentes a um mesmo
intervalinho dentre os n 2 restantes. Sendo x = (k j), teremos
{x} =
_
{k} {j} se {k} {j}
1 +{k} {j} se {k} < {j}
e portanto {x} [0,
1
n
) ou {x} [
n1
n
, 1), assimkj satisfaz as condi coes
do problema.
Exemplo 0.13. Demonstrar que dados 7 n umeros reais sempre e pos-
svel escolher 2 deles, digamos a e b, tais que

a b
1 +ab

<
1

3
.
Solu c

ao: Vejamos inicialmente que a funcao y = tan x e crescente


entre (

2
,

2
), e alem disso, para cada real r existe um unico angulo
neste intervalo com r = tan .
Portanto, dados os 7 n umeros reais, a cada um deles podemos fazer
corresponder um angulo no intervalo (

2
,

2
), e dividindo tal intervalo
em 6 partes iguais, i.e., em 6 intervalos de comprimento

6
, abertos `a
esquerda, existir ao 2 angulos e que estejam no mesmo intervalo, e
portanto, | | <

6
.
12 CAP

ITULO 0. FUNDAMENTOS
y = arctan x

2
Podemos supor sem perda de generalidade que a = tan > tan = b
e como a funcao tangente e crescente,
a b
1 +ab
=
tan tan
1 + tan tan
= tan( ) < tan

6
=
1

3
,
como queramos demonstrar.
Problemas Propostos
0.12. Escolhem-se 7 pontos no interior de um ret angulo de dimens oes
2 3. Demonstrar que sempre e possvel encontrar dois pontos tal que
sua distancia e menor ou igual a

2.
0.13. Escolhem-se 9 pontos no interior de um quadrado de lado 1. De-
monstrar que e possvel escolher 3 deles de tal forma que a area do
tri angulo que formam e menor ou igual a
1
8
.
0.14. Dadas 6 pessoas numa festa, demonstrar que necessariamente
existem 3 pessoas que se conhecem mutuamente ou 3 pessoas que n ao
se conhecem mutuamente. Suponha que a rela cao de conhecer e sime-
trica. Este e um caso particular do teorema de Ramsey, veja por exemplo
[8].
0.15. Do conjunto A = {1, 2, . . . , 99, 100} escolhemos 51 n umeros. De-
monstrar que, entre os 51 n umeros escolhidos, existem dois tais que um
e m ultiplo do outro.
0.16. Dado um n umero irracional u, demonstrar que sempre e possvel
encontrar innitos n umeros racionais
p
q
, p, q Z, de tal forma que

u
p
q

<
1
q
2
.
0.3. DIVISIBILIDADE 13
Um problema mais difcil e demonstrar existem racionais
p
q
de tal forma
que

u
p
q

<
1

5q
2
.
Veja o teorema 3.13 e a secao correspondente para este e outros resulta-
dos relacionados `a aproximacao de n umeros reais por n umeros racionais.
0.17 (IMO1985). Dado um conjunto M com 1985 inteiros positivos dis-
tintos, nenhum dos quais tem divisores primos maiores do que 23, mostre
que ha 4 elementos em M cujo produto e uma quarta potencia.
0.18 (OIbM1998). Determinar o mnimo valor de n para o qual, de todo
subconjunto de {1, 2, . . . , 999} com n elementos, e possvel selecionar
quatro inteiros diferentes a, b, c, d tais que a + 2b + 3c = d.
0.19. Demonstrar que de qualquer conjunto de 2
n+1
1 n umeros inteiros
positivos e possvel escolher 2
n
elementos de tal forma que sua soma e
divisvel por 2
n
.
0.20 (IMO2001). Sejam n
1
, n
2
, . . . , n
m
inteiros com m mpar. Deno-
temos por x = (x
1
, . . . , x
m
) uma permuta cao dos inteiros 1, 2, . . . , m,
e denamos f(x) = x
1
n
1
+ + x
m
n
m
. Demonstre que existem duas
permuta coes a e b tais que f(a) f(b) e divisvel por m!.
0.21 (IMO1991). Seja S = {1, 2, . . . , 280}. Encontrar o menor inteiro
n para o qual todo subconjunto de S com n elementos contem cinco
n umeros que sao dois a dois primos entre si.
0.22 (Erdos). Mostrar que toda a sequencia com n
2
+ 1 n umeros re-
ais contem ou uma subsequencia crescente com n + 1 termos ou uma
subsequencia decrescente com n + 1 termos.
0.23. Pintamos todos os pontos do plano de azul, verde ou preto. Mos-
trar que existe no plano um ret angulo cujos vertices tem todos a mesma
cor.
0.3 Divisibilidade
Dados dois inteiros d e a, dizemos que d divide a ou que d e um
divisor de a ou ainda que a e um m ultiplo de d e escrevemos
d | a
se existir q Z com a = qd. Caso contrario, escrevemos d a. Por
exemplo, temos que 5 | 10 mas 10 5.
Eis algumas propriedades importantes da divisibilidade:
14 CAP

ITULO 0. FUNDAMENTOS
Lema 0.14. Sejam a, b, c, d Z. Temos
(i) (d divide) Se d | a e d | b, ent ao d | ax + by para qualquer
combinacao linear ax +by de a e b com coecientes x, y Z.
(ii) (Limitacao) Se d | a, ent ao a = 0 ou |d| |a|.
(iii) (Transitividade) Se a | b e b | c, ent ao a | c.
Demonstra c

ao: Se d | a e d | b, entao podemos escrever a = dq


1
e
b = dq
2
com q
1
, q
2
Z, logo ax+by = d(q
1
x+q
2
y). Como q
1
x+q
2
y Z,
temos d | ax + by. Para mostrar (ii), suponha que d | a e a = 0. Neste
caso, a = dq com q = 0, assim |q| 1 e |a| = |d||q| |d|. Finalmente,
se a | b e b | c, entao existem q
1
, q
2
Z tais que b = aq
1
e c = bq
2
, logo
c = aq
1
q
2
e portanto a | c.
Vejamos como utilizar estas propriedades para resolver alguns pro-
blemas de divisibilidade.
Exemplo 0.15. Encontre todos os inteiros positivos n tais que 2n
2
+1 |
n
3
+ 9n 17.
Solu c

ao: Utilizando o 2n
2
+ 1 divide para reduzir o grau de n
3
+
9n 17, temos que
_
2n
2
+ 1 | n
3
+ 9n 17
2n
2
+ 1 | 2n
2
+ 1
= 2n
2
+ 1 | (n
3
+ 9n 17) 2 + (2n
2
+ 1) (n)
2n
2
+ 1 | 17n 34
Como o grau de 17n34 e menor do que o de 2n
2
+1, podemos utilizar
a limita cao para obter uma lista nita de candidatos a n. Temos
17n 34 = 0 n = 2 ou |2n
2
+ 1| |17n 34| n = 1, 4 ou 5.
Destes candidatos, apenas n = 2 e n = 5 s ao solucoes.
Exemplo 0.16 (IMO1994). Determine todos os pares (m, n) de inteiros
positivos para os quais
n
3
+1
mn1
e inteiro.
Solu c

ao: Vamos tentar reduzir o grau em n utilizando o d divide.


Temos
mn 1 | n
3
+ 1 = mn 1 | (n
3
+ 1) m(mn 1) n
2
mn 1 | n
2
+m.
0.3. DIVISIBILIDADE 15
Da mesma forma,
mn 1 | n
2
+m = mn 1 | (n
2
+m) m(mn 1) n
mn 1 | m
2
+n
e, nalmente,
mn 1 | m
2
+n = mn 1 | (m
2
+n) m(mn 1)
mn 1 | m
3
+ 1
que e a mesma expressao com que comecamos, trocando n por m. Assim,
temos que a condi cao e simetrica em m e n e as divisibilidades acima
s ao todas equivalentes entre si. Portanto podemos supor sem perda de
generalidade que m n. Utilizando a limita cao temos
mn 1 | n
2
+m = mn 1 n
2
+m m(n 1) n
2
+ 1.
Se n = 1 temos m
n
2
+1
n1
= n + 1 +
2
n1
. Como estamos assumindo
m n, se n 4 temos apenas duas possibilidades: m = n ou m = n+1.
Agora temos alguns casos a analisar.
Se m n = 1 devemos ter m 1 | 1
2
+ m = m 1 | m + 1
(m 1) m 1 | 2 e portanto m = 2 ou m = 3, ambos os
casos fornecendo solucoes.
Se m n = 2 devemos ter 2m 1 | 2
2
+ m = 2m 1 |
2(m + 4) (2m 1) 2m 1 | 9 m = 2 ou m = 5,
ambos os casos fornecendo solucoes.
Se m n = 3 devemos ter 3m 1 | 3
2
+ m = 3m 1 |
3(m + 9) (3m 1) 3m 1 | 28 m = 5, que fornece
uma solucao.
Se m = n 4 devemos ter
n
2
1 | n
2
+n n 1 | n
= n 1 | n (n 1) n 1 | 1
o que nao e possvel pois n 4.
Se m = n + 1 5 devemos ter
(n + 1)n 1 | n
2
+ (n + 1)
n
2
+n 1 | (n
2
+n + 1) (n
2
+n 1)
n
2
+n 1 | 2
o que novamente nao e possvel pois n 4.
Logo as solucoes (m, n) s ao (1, 2), (2, 1), (1, 3), (3, 1), (2, 2), (2, 5), (5, 2),
(3, 5), (5, 3).
16 CAP

ITULO 0. FUNDAMENTOS
0.4 mdc, mmc e Algoritmo de Euclides
Dados dois n umeros inteiros a e b com a = 0 ou b = 0, a cada
um deles pode-se associar seu conjunto de divisores positivos, D
a
e D
b
respectivamente, e a interseccao de tais conjuntos D
a
D
b
e nita (pela
limita cao) e nao vazia (ja que 1 pertence `a intersec cao). Por ser nito,
D
a
D
b
possui elemento m aximo, que e chamado de maximo divisor
comum (mdc) dos n umeros a e b. Denotamos este n umero por mdc(a, b)
(alguns autores usam a nota cao (a, b)). Para a = b = 0 convencionamos
mdc(0, 0) = 0. Quando mdc(a, b) = 1 dizemos que a e b s ao primos
entre si.
Por outro lado, se denotamos por M
n
o conjunto dos m ultiplos po-
sitivos de n, dados dois n umeros inteiros a e b com a = 0 e b = 0,
entao a interseccao M
a
M
b
e nao vazia (ja que |ab| esta na intersec-
cao). Como os naturais s ao bem ordenados, M
a
M
b
possui elemento
mnimo. Tal n umero e chamado mnimo m ultiplo comum (mmc) de a e
b e o denotaremos por mmc(a, b) (alguns autores escrevem [a, b]).
Para calcularmos o mdc e o mmc de maneira eciente, vamos descre-
ver o chamado algoritmo de Euclides ou algoritmo das divisoes sucessi-
vas. Primeiramente, vamos relembrar o conceito de divisao euclidiana,
ou divisao com resto, que e uma das quatro operacoes que toda crian ca
aprende na escola. Sua formulacao precisa e: dados a, b Z com b = 0,
existem q, r Z com
a = bq +r e 0 r < |b|.
Tais q e r estao unicamente determinados pelas duas condi coes acima
(veja o argumento a seguir) e s ao chamados o quociente e resto da divisao
de a por b. O resto r e tambem denotado por a mod b.
Para x R, denimos o piso ou parte inteira x de x como sendo
o unico k Z tal que k x < k + 1; denimos o teto x de x como
o unico k Z tal que k 1 < x k. Por exemplo, temos

2 = 1,

2 = 2, 10 = 10 = 10, = 4 e = 3. Podemos agora


mostrar a existencia de q e r satisfazendo as duas condi coes acima: basta
tomar
q =
_
a/b se b > 0
a/b se b < 0
e r = a bq em ambos os casos
e e facil vericar que 0 r < |b| a partir das denicoes das funcoes piso
e teto. Por outro lado, se a = bq
1
+ r
1
= bq
2
+ r
2
com 0 r
1
, r
2
< |b|,
entao temos que r
2
r
1
= b(q
1
q
2
) e um m ultiplo de b com|r
2
r
1
| < |b|,
portanto r
2
r
1
= 0 e assim q
1
= q
2
tambem, o que prova a unicidade.
Podemos agora descrever o algoritmo de Euclides para calcular o
mdc, que se baseia na seguinte simples observa cao:
0.4. MDC, MMC E ALGORITMO DE EUCLIDES 17
Lema 0.17 (Euclides). Se a = bq +r, ent ao mdc(a, b) = mdc(b, r).
Demonstra c

ao: Basta mostrar que D


a
D
b
= D
b
D
r
, ja que se
estes conjuntos forem iguais em particular os seus m aximos tambem
serao iguais. Se d D
a
D
b
temos d | a e d | b, logo d | abq d | r
e portanto d D
b
D
r
. Da mesma forma, se d D
b
D
r
temos d | b e
d | r, logo d | bq +r d | a e assim d D
a
D
b
.
O algoritmo de Euclides consiste na aplicacao reiterada do lema
acima onde q e r s ao o quociente e o resto na divisao de a por b (note que
o lema vale mesmo sem a condi cao 0 r < |b|). Como os restos formam
uma sequencia estritamente decrescente, o algoritmo eventualmente para
quando atingimos o resto 0.
Exemplo 0.18. Calcule mdc(1001, 109).
Solu c

ao: Realizando as divisoes sucessivas, temos


1001 = 109 9 + 20
109 = 20 5 + 9
20 = 9 2 + 2
9 = 2 4 + 1
2 = 1 2 + 0
Assim, temos mdc(1001, 109) = mdc(109, 20) = mdc(20, 9) = mdc(9, 2) =
mdc(2, 1) = mdc(1, 0) = 1.
Exemplo 0.19. Sejam m = n dois n umeros naturais. Demonstrar que
mdc(a
2
m
+ 1, a
2
n
+ 1) =
_
1 se a e par,
2 se a e mpar.
Solu c

ao: Suponha sem perda de generalidade que m > n e observe a


fatora cao
a
2
m
1 = (a
2
m1
+ 1)(a
2
m2
+ 1)(a
2
m3
+ 1) . . . (a
2
n
+ 1)(a
2
n
1)
Logo a
2
m
+ 1 = (a
2
n
+ 1) q + 2 com q Z e assim
mdc(a
2
m
+ 1, a
2
n
+ 1) = mdc(a
2
n
+ 1, 2)
que e igual a 2 se a
2
n
+ 1 for par, isto e, se a for mpar, e e igual a 1
caso contrario.
18 CAP

ITULO 0. FUNDAMENTOS
Alem de servir de ferramenta computacional para o calculo do mdc,
a divisao euclidiana tem consequencias te oricas importantes. O proximo
teorema mostra que e sempre possvel escrever o mdc de dois n umeros
como combinacao linear destes (com coecientes inteiros).
Teorema 0.20 (Bachet-Bezout). Sejam a, b Z. Ent ao existem x, y
Z com
ax +by = mdc(a, b).
Portanto se c Z e tal que c | a e c | b ent ao c | mdc(a, b).
Demonstra c

ao: O caso a = b = 0 e trivial (temos x = y = 0). Nos


outros casos, considere o conjunto de todas as combinacoes Z-lineares
de a e b:
I(a, b)
def
= {ax +by : x, y Z}
Seja d = ax
0
+by
0
o menor elemento positivo de I(a, b) (ha pelo menos
um elemento positivo, verique!). Armamos que d divide todos os
elementos de I(a, b). De fato, dado m = ax+by I(a, b), sejam q, r Z
o quociente e o resto na divisao euclidiana de m por d, de modo que
m = dq +r e 0 r < d. Temos
r = mdq = a(x qx
0
) +b(y qy
0
) I(a, b).
Mas como r < d e d e o menor elemento positivo de I(a, b), segue que
r = 0 e portanto d | m.
Em particular, como a, b I(a, b) temos que d | a e d | b, logo d
mdc(a, b). Note ainda que se c | a e c | b, entao c | ax
0
+by
0
c | d.
Tomando c = mdc(a, b) temos que mdc(a, b) | d o que, juntamente com
a desigualdade d mdc(a, b), mostra que d = mdc(a, b).
Corolario 0.21. Sejam a, b, c Z. A equacao
ax +by = c
admite solu cao inteira em x e y se, e somente se, mdc(a, b) | c.
Demonstra c

ao: Se a equa cao admite solucao inteira, entao mdc(a, b)


divide o lado esquerdo, logo deve dividir o direito tambem. Reciproca-
mente, se mdc(a, b) | c, digamos c = k mdc(a, b) com k Z, pelo
teorema acima existem inteiros x
0
e y
0
tais que ax
0
+by
0
= mdc(a, b) e
multiplicando tudo por k obtemos que x = kx
0
e y = ky
0
s ao solucoes
da equa cao dada.
0.4. MDC, MMC E ALGORITMO DE EUCLIDES 19
Temos uma outra importante consequencia do teorema anterior:
Proposi cao 0.22. Se mdc(a, b) = 1 e a | bc, ent ao a | c.
Demonstra c

ao: Como mdc(a, b) = 1, existem x, y Z tais que


ax + by = 1 = a cx + (bc) y = c. Do fato de a dividir cada termo
do lado esquerdo, temos que a | c.
Lembramos que um natural p > 1 e chamado primo se os unicos
divisores positivos de p s ao 1 e p e um natural n > 1 e chamado composto
se admite outros divisores alem de 1 e n. Observemos que 1 nao e nem
primo nem composto.
Claramente, se p e primo e p a temos mdc(p, a) = 1. Usando a
proposicao anterior e indu cao temos o seguinte resultado:
Corolario 0.23. Seja p um n umero primo e sejam a
1
, . . . a
m
Z. Se
p | a
1
a
m
, ent ao p | a
i
para algum i, 1 i m.
O proximo lema resume algumas propriedades uteis do mdc:
Lema 0.24. Temos
1. Se p e primo, ent ao mdc(a, p) e 1 ou p.
2. Se k e um inteiro, ent ao mdc(a, b) = mdc(a kb, b).
3. Se a | c, ent ao mdc(a, b) | mdc(c, b).
4. Se mdc(a, b) = 1, ent ao mdc(ac, b) = mdc(c, b).
Demonstra c

ao: O primeiro item e claro e o segundo e apenas uma


reformulacao do lema 0.17. Para provar o terceiro item, observe que
mdc(a, b) | a e a | c implicam que mdc(a, b) | c. Como tambem te-
mos mdc(a, b) | b, conclumos que mdc(a, b) | mdc(b, c) por Bachet-
Bezout. Finalmente, para mostrar o ultimo item, note primeiro que
mdc(c, b) | mdc(ac, b) pois mdc(c, b) divide simultaneamente ac e b. Re-
ciprocamente, para mostrar que mdc(ac, b) | mdc(c, b), podemos escrever
ax + by = 1 com x, y Z por Bachet-Bezout. Assim, mdc(ac, b) divide
ac x +b cy = c e tambem divide b, logo divide mdc(c, b).
Vejamos como podemos usar as propriedades acima para solucionar
o seguinte
Exemplo 0.25. Sejam a
n
= 100 + n
2
e d
n
= mdc(a
n
, a
n+1
). Calcular
d
n
para todo n.
20 CAP

ITULO 0. FUNDAMENTOS
Solu c

ao: Aplicando a propriedade 2 temos que


d
n
= mdc(100 +n
2
, 100 + (n + 1)
2
) = mdc(100 +n
2
, 2n + 1).
Como 2n + 1 e mpar, mdc(4, 2n + 1) = 1 e pelas propriedades 4 e 2
temos que
d
n
= mdc(400 + 4n
2
, 2n + 1)
= mdc(400 + 4n
2
(2n + 1)(2n 1), 2n + 1)
= mdc(401, 2n + 1).
Como 401 e primo, entao mdc(401, 2n+1) = 401 se 2n+1 = 401k (com
k = 2r +1 inteiro mpar) e mdc(401, 2n+1) = 1 caso contrario, ou seja,
d
n
=
_
401 se n = 401r + 200 com r Z
1 caso contrario.
A proxima proposicao conecta o mdc e o mmc de dois inteiros e pode
ser utilizada, juntamente com o algoritmo de Euclides, para o calculo
eciente do mmc.
Proposi cao 0.26. Sejam a e b dois n umeros naturais, ent ao
mdc(a, b) mmc(a, b) = a b.
Demonstra c

ao: Escreva d = mdc(a, b) e a = a


1
d e b = b
1
d onde
a
1
, b
1
Z s ao tais que mdc(a
1
, b
1
) = 1. Temos mmc(a, b) = al para
algum l Z; alem disso, b | mmc(a, b) b
1
d | a
1
dl b
1
| a
1
l.
Como mdc(a
1
, b
1
) = 1, isto implica que b
1
| l pela proposicao 0.22.
Pela denicao de mnimo m ultiplo comum, temos que l deve ser o m-
nimo n umero divisvel por b
1
, assim conclumos que l = b
1
e portanto
mmc(a, b) = b
1
a. Logo mdc(a, b) mmc(a, b) = d b
1
a = a b.
A demonstra cao que demos do teorema de Bachet-Bezout nao mostra
como efetivamente encontrar uma solucao de ax + by = mdc(a, b). Po-
rem, isto pode ser feito utilizando-se o algoritmo de Euclides, como mos-
tra o exemplo a seguir. De fato, este exemplo pode servir como ponto de
partida para uma segunda demonstra cao do teorema de Bachet-Bezout
(veja os exerccios).
Exemplo 0.27. Encontre todos os x, y Z tais que
1001x + 109y = mdc(1001, 109).
0.4. MDC, MMC E ALGORITMO DE EUCLIDES 21
Solu c

ao: Fazemos as divisoes sucessivas para o calculo de


mdc(1001, 109) = 1
utilizando o algoritmo de Euclides (veja o exemplo 0.18). Em seguida,
isolamos os restos:
20 = 1001 109 9
9 = 109 20 5
2 = 20 9 2
1 = 9 2 4
Note que a ultima divisao permite expressar o mdc 1 como combinacao
linear de 9 e 2:
9 1 2 4 = 1.
Mas da pen ultima divisao, temos que 2 = 20 9 2, logo substituindo
esta expressao na combinacao linear acima, temos
9 ( 20 9 2) 4 = 1 9 9 20 4 = 1
e agora expressamos 1 como combinacao linear de 20 e 9. Repetindo este
procedimento, eventualmente expressaremos 1 como combinacao linear
de 1001 e 109. Tomamos o cuidado de lembrar quais s ao os coeci-
entes a e b nas equa coes ax +by = mdc(a, b) durante as simplicacoes.
Continuando, obtemos
1 = ( 109 20 5) 9 20 4 = 109 9 20 49
1 = 109 9 ( 1001 109 9) 49 = 1001 (49) + 109 450
Logo uma solucao da equa cao 1001x +109y = 1 e (x
0
, y
0
) = (49, 450).
Para encontrar as demais, escrevemos o lado direito desta equa cao uti-
lizando a solucao particular que acabamos de encontrar:
1001x + 109y = 1001x
0
+ 109y
0
1001(x x
0
) = 109(y y
0
).
Como mdc(1001, 109) = 1 temos pela proposicao 0.22 que 1001 divide
y y
0
, ou seja, y y
0
= 1001t para algum t Z e, portanto, x x
0
=
109t. Assim, as solucoes da equa cao dada s ao todos os pontos da reta
1001x + 109y = 1 da forma
(x, y) = (x
0
109t, y
0
+ 1001t) = (49, 450) + (109, 1001) t
com t Z.
22 CAP

ITULO 0. FUNDAMENTOS
Em geral, o raciocnio do exemplo acima mostra que se mdc(a, b) = 1
e (x
0
, y
0
) e uma solucao da equa cao ax+by = c, entao todas as solucoes
inteiras s ao dadas por x = x
0
bk e y = y
0
+ak com k Z.
Exemplo 0.28. Sejam a, b inteiros positivos com mdc(a, b) = 1. Mostre
que para todo c Z com c > ab a b, a equacao ax + by = c admite
solu coes inteiras com x, y 0.
Solu c

ao: Seja (x
0
, y
0
) uma solucao inteira (que existe pelo teorema
de Bachet-Bezout). Devemos mostrar a existencia de um inteiro k tal
que
x = x
0
bk > 1 e y = y
0
+ak > 1,
ou seja,

y
0
+ 1
a
< k <
x
0
+ 1
b
.
Mas isto segue do fato de o intervalo (
y
0
+1
a
,
x
0
+1
b
) ter tamanho maior
do que 1:
x
0
+ 1
b

_

y
0
+ 1
a
_
=
ax
0
+by
0
+a +b
ab
=
c +a +b
ab
> 1.
Exemplo 0.29. Seja n um n umero inteiro com 1312345 n 1312463.
Determine quantas solu coes inteiras positivas no maximo tem a equacao
983x + 1237y = n. Determine todas elas em termos de n e de outro
par ametro inteiro.
Solu c

ao: Primeiro encontremos todas as solucoes inteiras de


983x + 1237y = mdc(983, 1237).
Para isto, usaremos uma tabela gerada usado o seguinte algoritmo que
calcula mdc(a, b):
1. As primeiras duas linhas da tabela s ao preenchidas como
Resto Quociente x y
a 0 1
b 1 0
2. Da terceira linha para frente, a linha i +1 da tabela e calculada a
partir das duas linhas anteriores da seguinte forma
0.4. MDC, MMC E ALGORITMO DE EUCLIDES 23
Resto Quociente x y
r
i1
q
i1
x
i1
y
i1
r
i
q
i
x
i
y
i
r
i+1
q
i+1
x
i1
q
i+1
x
i
y
i1
q
i+1
y
i
onde q
i+1
e r
i+1
s ao respectivamente o quocente e o resto da divisao
de r
i1
por r
i
.
Usando esta construcao podemos provar por indu cao que, em cada linha
da tabela, temos r
i
= ax
i
+ by
i
. Assim, quando r
l+1
= 0, temos que
r
l
= mdc(a, b).
Em nosso caso especico, geramos a seguinte tabela
Resto Quociente x y
1237 0 1
983 1 0
254 1 1 1
221 3 4 3
33 1 5 4
23 6 34 27
10 1 39 31
3 2 112 89
1 3 375 298
Uma solucao de 983x + 1237y = n (onde n e um inteiro xo entre
1312345 e 1312463) e x
0
= 375n e y
0
= 298n. Portanto, todas as
solucoes desta equa cao s ao
_
x = 375n + 1237k
y = 298n 983k
e k e um inteiro
arbitr ario. Como estamos interessados nas solucoes positivas, devemos
ter
375n + 1237k > 0 e 298n 983k > 0,
isto e
k >
375n
1237
e k <
298n
983
.
Assim, esta equa cao tem no m aximo

298n
983

375n
1237
=
n
1215971

1312463
1215971
= 2
solucoes positivas. Como, para n = 1312447,
375n
1237
< 397872 < 397873 <
298n
983
, temos nesse caso 2 solucoes (correspondentes a k = 397872 e k =
397873). Assim, o n umero m aximo de solucoes inteiras positivas de uma
tal equa cao e 2.
24 CAP

ITULO 0. FUNDAMENTOS
0.5 O Teorema Fundamental da Aritmetica
Estamos agora prontos para enunciar o teorema que caracteriza todo
n umero natural em termos de seus constituintes primos.
Teorema 0.30 (Teorema Fundamental da Aritmetica). Seja n 2 um
n umero natural. Podemos escrever n de uma unica forma como um
produto
n = p
1
p
m
onde m 1 e um natural e p
1
. . . p
m
sao primos.
Demonstra c

ao: Mostramos a existencia da fatora cao de n em primos


por indu cao. Se n e primo nao ha o que provar (escrevemos m = 1,
p
1
= n). Se n e composto podemos escrever n = ab, a, b N, 1 < a < n,
1 < b < n. Por hip otese de indu cao, a e b se decompoem como produto
de primos. Juntando as fatora coes de a e b (e reordenando os fatores)
obtemos uma fatora cao de n.
Vamos agora mostrar a unicidade. Suponha por absurdo que n possui
duas fatora coes diferentes
n = p
1
p
m
= q
1
q
m
,
com p
1
. . . p
m
, q
1
. . . q
m
e que n e mnimo com tal propri-
edade. Como p
1
| q
1
q
m
temos p
1
| q
i
para algum valor de i pelo
corolario 0.23. Logo, como q
i
e primo, p
1
= q
i
e p
1
q
1
. Analogamente
temos q
1
p
1
, donde p
1
= q
1
. Mas
n/p
1
= p
2
p
m
= q
2
q
m

admite uma unica fatora cao, pela minimalidade de n, donde m = m

e
p
i
= q
i
para todo i, o que contradiz o fato de n ter duas fatora coes.
Outra forma de escrever a fatora cao acima e
n = p
e
1
1
. . . p
e
m
m
,
com p
1
< < p
m
e e
i
> 0. Ainda outra formulacao e escrever
n = 2
e
2
3
e
3
5
e
5
. . . p
e
p
. . .
onde o produto e tomado sobre todos os primos mas apenas um n umero
nito de expoentes e maior do que zero. Vamos nos referir a qualquer
destas expressoes como a fatoracao can onica de n em primos.
A fatora cao unica em primos se aplica em contextos mais gerais,
como veremos mais tarde. Aqui, como aplicacao imediata do Teorema
Fundamental da Aritmetica, vamos mostrar a prova atribuda a Euclides
para a existencia de innitos primos (uma prova com mais de 2000 anos
e que ainda funciona!).
0.5. O TEOREMA FUNDAMENTAL DA ARITM

ETICA 25
Teorema 0.31 (Euclides). Existem innitos primos.
Demonstra c

ao: Suponha por absurdo que p


1
, p
2
, . . . , p
m
fossem to-
dos os primos. O n umero N = p
1
p
2
. . . p
m
+ 1 > 1 nao seria divisvel
por nenhum primo p
i
, o que contradiz o Teorema Fundamental da Arit-
metica.
Observe que n ao provamos que p
1
p
2
. . . p
m
+ 1 e primo para algum
conjunto nito de primos (por exemplo, os m primeiros primos). Ali as,
2 3 5 7 11 13 + 1 = 30031 = 59 509 nao e primo. N ao se conhece
nenhuma formula simples que gere sempre n umeros primos.
Embora a quantidade de primos seja innita, uma quest ao natural e
saber o quao raros oufrequentes eles s ao. Na segunda parte do livro,
discutiremos mais a fundo esta quest ao sobre a distribui cao dos primos.
Por outro lado, e interessante notar que existem cadeias arbitrariamente
longas de n umeros compostos consecutivos: na sequencia
(k + 1)! + 2, (k + 1)! + 3, (k + 1)! + 4, . . . , (k + 1)! + (k + 1),
nenhum termo e primo, pois eles admitem fatores proprios 2, 3, 4, . . . , k+
1, respectivamente.
Uma interessante prova alternativa, devida a Erdos, de que existem
innitos primos e a seguinte:
Suponha, por contradi cao, que existe um n umero nito de primos,
digamos p
1
, p
2
, . . . , p
k
. Seja n um n umero natural. Entao podemos es-
crever qualquer n umero m n na forma m = m
2
1
m
2
, onde m
2
1
n
e
m
2
= p
a
1
1
p
a
2
2
p
a
k
k
onde a
k
= 0 ou 1 para cada k.
Assim, considerando todas as possveis maneiras de escrever os naturais
m n, temos: 2
k
escolhas para m
2
e no m aximo

n escolhas para
m
1
. Ou seja, para todo n natural, vale que
n 2
k

n
absurdo, pois esta desigualdade nao vale para n sucientemente grande.
Exemplo 0.32 (OIbM1987). A sequencia p
n
e denida da seguinte
forma:
(i) p
1
= 2.
(ii) Para todo n 2, p
n
e o maior divisor primo da expressao
p
1
p
2
p
3
p
n1
+ 1.
26 CAP

ITULO 0. FUNDAMENTOS
Demonstrar que p
n
e diferente de 5.
Solu c

ao: Dado que p


1
= 2, p
2
= 3, p
3
= 7, segue-se que para qualquer
n 3, p
1
p
2
p
n1
e m ultiplo de 2 e de 3, portanto p
1
p
2
p
n1
+ 1
nao e m ultiplo nem de 2 nem de 3. Alem disso, como p
1
= 2, entao p
n
e mpar para todo n 2, assim p
1
p
2
p
n1
nao e m ultiplo de 4.
Suponhamos que exista n tal que p
n
= 5, isto e, o maior divisor
primo de p
1
p
2
p
n1
+1 e 5. Como 2 e 3 nao dividem p
1
p
2
p
n1
+1,
temos que
p
1
p
2
p
n1
+ 1 = 5
k
.
Portanto
p
1
p
2
p
n1
= 5
k
1 = (5 1)(5
k1
+ 5
k2
+ + 5 + 1),
donde 4 | p
1
p
2
p
n1
, uma contradi cao.
Exemplo 0.33. Determine todas as ternas (a, b, c) de inteiros positivos
tais que a
2
= 2
b
+c
4
.
Solu c

ao: Como a
2
= 2
b
+c
4
(ac
2
)(a+c
2
) = 2
b
, pelo Teorema
Fundamental da Aritmetica existem dois naturais m > n tais que m +
n = b, a c
2
= 2
n
e a +c
2
= 2
m
. Subtraindo as duas ultimas equa coes,
obtemos que 2c
2
= 2
m
2
n
, assim c
2
= 2
n1
(2
mn
1). Como 2
n1
e
2
mn
1 s ao primos entre si e o seu produto e um quadrado perfeito (i.e.
os expoentes das potencias de primos distintos s ao pares), novamente
pelo Teorema Fundamental da Aritmetica 2
n1
e 2
mn
1 devem ser
ambos quadrados perfeitos, logo n1 e par e 2
mn
1 = (2k 1)
2
para
algum inteiro positivo k. Como 2
mn
= (2k 1)
2
+ 1 = 4k(k 1) + 2 e
divisvel por 2 mas nao por 4, temos mn = 1. Assim, fazendo n1 =
2t, temos que todas as solucoes s ao da forma (a, b, c) = (3 2
2t
, 4t +3, 2
t
)
com t N e e facil vericar que todos os n umeros desta forma s ao
solucoes.
Segue do Teorema Fundamental da Aritmetica que todo divisor de
n = p
e
1
1
. . . p
e
m
m
e da forma
p
d
1
1
. . . p
d
m
m
com 0 d
i
e
i
. Assim, obtemos o outro algoritmo usual para cal-
cular o mdc de dois n umeros: fatoramos os dois n umeros em primos e
tomamos os fatores comuns com os menores expoentes. Este algoritmo e
bem menos eciente do que o de Euclides para inteiros grandes (que em
geral nao sabemos fatorar de forma eciente computacionalmente) mas
e instrutivo saber que os dois algoritmos dao o mesmo resultado. Alem
disso, este algoritmo tem consequencias te oricas importantes, como por
exemplo o
0.5. O TEOREMA FUNDAMENTAL DA ARITM

ETICA 27
Corolario 0.34. Se mdc(a, n) = mdc(b, n) = 1, ent ao mdc(ab, n) = 1.
Demonstra c

ao: Evidente a partir do algoritmo descrito acima.


Para encerrar esta secao, vejamos ainda algumas outras aplicacoes
do Teorema Fundamental da Aritmetica.
Proposi cao 0.35. Seja n = p
e
1
1
. . . p
e
m
m
a fatoracao de n em potencias
de primos distintos p
i
e seja
k
(n)
def
=

d|n, d>0
d
k
a soma das k-esimas
potencias dos divisores positivos de n. Ent ao

k
(n) =
p
(e
1
+1)k
1
1
p
k
1
1
. . .
p
(e
m
+1)k
m
1
p
k
m
1
.
Para k = 0, a formula acima deve ser interpretada tomando-se o limite
k 0, de modo que a quantidade de divisores positivos de n e
0
(n) =
(e
1
+ 1) (e
m
+ 1).
Demonstra c

ao: Como a soma na denicao de


k
(n) percorre todos os
n umeros da forma d
k
= p
d
1
k
1
. . . p
d
m
k
m
com 0 d
i
e
i
, temos a seguinte
fatora cao:

k
(n) = (1 +p
k
1
+p
2k
1
+ +p
e
1
k
1
) . . . (1 +p
k
m
+p
2k
m
+ +p
e
m
k
m
).
Somando as progress oes geometricas 1+p
k
i
+p
2k
i
+ +p
e
i
k
i
=
p
(e
i
+1)k
i
1
p
k
i
1
,
o resultado segue.
Proposi cao 0.36 (Fatores do Fatorial). Seja p um primo. Ent ao a
maior potencia de p que divide n! e p

onde
=
_
n
p
_
+
_
n
p
2
_
+
_
n
p
3
_
+
Observe que a soma acima e nita pois os termos
_
n
p
i
_
s ao eventualmente
zero.
Demonstra c

ao: No produto n! = 1 2 . . . n, apenas os m ultiplos


de p contribuem com um fator p. H a
_
n
p
_
tais m ultiplos entre 1 e n.
Destes, os que s ao m ultiplos de p
2
contribuem com um fator p extra
e ha
_
n
p
2
_
tais fatores. Dentre estes ultimos, os que s ao m ultiplos de
p
3
contribuem com mais um fator p e assim por diante, resultando na
formula acima.
28 CAP

ITULO 0. FUNDAMENTOS
Exemplo 0.37. Determine com quantos zeros termina 1000!.
Solu c

ao: O problema e equivalente a determinar qual a maior poten-


cia de 10 que divide 1000! e como ha muito mais fatores 2 do que 5 em
1000!, o expoente desta potencia coincide com o da maior potencia de 5
que divide 1000!, ou seja,
_
1000
5
_
+
_
1000
5
2
_
+
_
1000
5
3
_
+
_
1000
5
4
_
= 249.
Assim, 1000! termina com 249 zeros.
Problemas Propostos
0.24 (IMO1959). Mostre que a fracao
21n+4
14n+3
e irredutvel para todo n
natural.
0.25. Encontre todos os inteiros positivos tais que
(a) n + 1 | n
3
1
(b) 2n 1 | n
3
+ 1
(c)
1
n
+
1
m
=
1
143
(d) 2n
3
+ 5 | n
4
+n + 1
0.26. Demonstre:
(a) se m | a b, ent ao m | a
k
b
k
para todo natural k.
(b) se f(x) e um polin omio com coecientes inteiros e a e b sao inteiros
quaisquer, ent ao a b | f(a) f(b).
(c) se k e um natural mpar, ent ao a +b | a
k
+b
k
.
0.27. Mostre que
(a) 2
15
1 e 2
10
+ 1 sao primos entre si.
(b) 2
32
+ 1 e 2
4
+ 1 sao primos entre si.
0.28. Demonstrar que (n 1)
2
| n
k
1 se, e so se, n 1 | k.
0.29 (IMO1992). Encontrar todos os inteiros a, b, c com 1 < a < b < c
tais que (a 1)(b 1)(c 1) e divisor de abc 1.
Dica: Mostrar primeiro que a 4 e considerar os possveis casos.
0.5. O TEOREMA FUNDAMENTAL DA ARITM

ETICA 29
0.30 (IMO1998). Determine todos os pares de inteiros positivos (a, b)
tais que ab
2
+b + 7 divide a
2
b +a +b.
Dica: Mostre que ab
2
+ b + 7 | 7a b
2
e considerar tres casos: 7a b
2
maior, menor ou igual a zero.
0.31. Mostre que, se n > 1, ent ao
n

k=1
1
k
= 1 +
1
2
+ +
1
n
n ao e um n umero inteiro.
0.32 (OBM1997). Sejam c Q, f(x) = x
2
+c. Denimos
f
0
(x) = x, f
n+1
(x) = f(f
n
(x)), n N.
Dizemos que x R e pre-peri odico se {f
n
(x), n N} e nito. Mostre
que {x Q| xe pre-peri odico} e nito.
0.33. Demonstrar que se mdc(a, 2
n+1
) = 2
n
e mdc(b, 2
n+1
) = 2
n
, ent ao
mdc(a +b, 2
n+1
) = 2
n+1
.
0.34. Demonstrar que se a, b, c, d, m e n sao inteiros tais que adbc = 1
e mn = 0, ent ao
mdc(am+bn, cm+dn) = mdc(m, n).
0.35. Seja F
n
o n-esimo termo da sequencia de Fibonacci.
(a) Encontrar dois n umeros inteiros a e b tais que 233a + 144b = 1
(observe que 233 e 144 sao termos consecutivos da sequencia de Fi-
bonacci).
(b) Mostre que mdc(F
n
, F
n+1
) = 1 para todo n 0.
(c) Determine x
n
e y
n
tais que F
n
x
n
+F
n+1
y
n
= 1.
0.36. Sejam a e b dois inteiros positivos e d seu maximo divisor comum.
Demonstrar que existem dois inteiros positivos x e y tais que axby = d.
0.37. Denimos a sequencia de fra coes de Farey de ordem n como o
conjunto de fracoes reduzidas
a
b
tais que 0
a
b
1, 1 b n. Por
exemplo a sequencia de Farey de ordem 3 e
0
1
,
1
3
,
1
2
,
2
3
,
1
1
.
(a) Demonstrar que se
a
b
e
c
d
sao dois termos consecutivos de uma
sequencia de Farey, ent ao cb ad = 1.
(b) Demonstrar que se
a
1
b
1
,
a
2
b
2
,
a
3
b
3
sao tres termos consecutivos de uma
sequencia de Farey, ent ao
a
2
b
2
=
a
1
+a
3
b
1
+b
3
.
30 CAP

ITULO 0. FUNDAMENTOS
0.38. Utilize inducao em min{a, b} e o algoritmo de Euclides para mos-
trar que ax +by = mdc(a, b) admite solu cao com x, y Z, obtendo uma
nova demonstracao do teorema de Bachet-Bezout.
0.39. Sejam a e b n umeros inteiros positivos. Considere o conjunto
C = {ax +by | x, y N}
Lembre-se de que ja mostramos no exemplo 0.28 que todo n umero maior
que ab a b pertence a C.
(a) Demonstre que o n umero ab a b n ao pertence a C.
(b) Achar a quantidade de n umeros inteiros positivos que n ao pertencem
a C.
0.40 (IMO1984). Dados os inteiros positivos a, b e c, dois a dois primos
entre si, demonstrar que 2abc ab bc ca e o maior n umero inteiro
que n ao pode expressar-se na forma xbc+yca+zab com x, y e z inteiros
n ao negativos.
0.41 (IMO1977). Sejam a, b inteiros positivos. Quando dividimos a
2
+b
2
por a +b, o quociente e q e o resto e r. Encontrar todos os a, b tais que
q
2
+r = 1977.
0.42. Demonstrar que mdc(2
a
1, 2
b
1) = 2
mdc(a,b)
1 para todo
a, b N.
Pelo algoritmo de Euclides aplicado aos expoentes, basta mostrar que
mdc(2
bq+r
1, 2
b
1) = mdc(2
b
1, 2
r
1). Mas isto segue novamente
do lema de Euclides, pois 2
bq+r
1 = 2
r
(2
bq
1) + 2
r
1 e 2
bq
1 =
(2
b
1)(2
b(q1)
+ 2
b(q2)
+ + 2
b
+ 1) e um m ultiplo de 2
b
1.
0.43. Encontrar todas as fun coes f : Z Z Z satisfazendo simulta-
neamente as seguintes propriedades
(i) f(a, a) = a.
(ii) f(a, b) = f(b, a).
(iii) Se a > b, ent ao f(a, b) =
a
ab
f(a b, b).
0.44. Mostre que se n e um n umero natural composto, ent ao n e divisvel
por um primo p com p

n.
0.45 (IMO1989). Prove que, para todo inteiro positivo n, existem n
inteiros positivos consecutivos, nenhum dos quais e potencia de primo.
0.46 (Chi1998). Encontrar todos os n para os quais 1 +
_
n
1
_
+
_
n
2
_
+
_
n
3
_
divide 2
2000
.
0.6. CONGRU

ENCIAS 31
0.47 (IMO2002). Sejam d
1
< d
2
< < d
k
os divisores positivos de
um inteiro n > 1. Seja d = d
1
d
2
+ d
2
d
3
+ + d
k1
d
k
. Mostre que
d < n
2
e encontrar todos os n para os quais d | n
2
. Temos d =
n
2
d
k
d
k1
+
n
2
d
k1
d
k2
+ +
n
2
d
2
d
1
< n
2
(
1
12
+
1
23
+
1
34
+ ) = n
2
(
1
1

1
2
+
1
2

1
3
+
1
3

1
4
+ ) = n
2
. Por outro lado, se p e o menor primo
que divide n
2
, temos que d d
k1
d
k
=
n
2
p
. Como
n
2
p
e o maior divisor
pr oprio de n
2
e d > d
k1
d
k
se k > 2, temos que d | n
2
se, e so se, n = p
e primo.
0.48 (IMO1997). Encontrar todos os pares (x, y) de inteiros positivos
tais que x
y
2
= y
x
.
Dica: Sejam x = p

1
1
. . . p

n
n
e y = p

1
1
. . . p

n
n
as fatoracoes can onicas de
x e y. Mostre que
j
= t
j
e x = y
t
para algum t Q e tente limitar
os valores de t.
0.49. Generalizar o resultado anterior para x
y
n
= y
x
, onde x e y sao
inteiros positivos.
0.50 (IMO1984). Sejam a, b, c, d inteiros mpares tais que 0 < a < b <
c < d e ad = bc. Demonstre que se a+d = 2
k
e b +c = 2
m
para inteiros
k e m, ent ao a = 1.
0.6 Congruencias
Sejam a, b, n Z. Dizemos que a e congruente a b modulo n, e
escrevemos
a b (mod n)
se n | a b, ou seja, se a e b deixam o mesmo resto na divisao por n.
Por exemplo, temos que 17 3 (mod 7) e 10 5 (mod 3).
Proposi cao 0.38. Para quaisquer a, b, c, d, n Z temos:
1. (Reexividade) a a (mod n);
2. (Simetria) se a b (mod n), ent ao b a (mod n);
3. (Transitividade) se a b (mod n) e b c (mod n), ent ao a c
(mod n);
4. (Compatibilidade com a soma e diferen ca) Podemos somar e sub-
trair membro a membro:
_
a b (mod n)
c d (mod n)
=
_
a +c b +d (mod n)
a c b d (mod n)
Em particular, se a b (mod n), ent ao ka kb (mod n) para
todo k Z.
32 CAP

ITULO 0. FUNDAMENTOS
5. (Compatibilidade com o produto) Podemos multiplicar membro a
membro:
_
a b (mod n)
c d (mod n)
= ac bd (mod n)
Em particular, se a b (mod n), ent ao a
k
b
k
(mod n) para
todo k N.
6. (Cancelamento) Se mdc(c, n) = 1, ent ao
ac bc (mod n) a b (mod n).
Demonstra c

ao: Para o item (1) basta observar que n | a a = 0.


Em (2), se n | a b, entao n | (a b) n | b a. Em (3), se
n | a b e n | b c, entao n | (a b) + (b c) n | a c. Em
(4) e (5), se n | a b e n | c d, entao n | (a b) + (c d) n |
(a + c) (b + d), n | (a b) (c d) n | (a c) (b d) e
n | (a b)c +(c d)b n | ac bd. Finalmente, como mdc(c, n) = 1
temos que n | acbc n | (ab)c n | ab pela proposicao 0.22.
As propriedades acima mostram que a rela cao (mod n) (ser con-
gruente m odulo n) tem um comportamento muito similar `a rela cao de
igualdade usual. Sao estas propriedades que tornam as congruencias tao
uteis em problemas de divisibilidade. Vejamos alguns exemplos.
Exemplo 0.39. Demonstrar que 31 | 20
15
1.
Solu c

ao: Isto e equivalente a demonstrar que 20


15
1 (mod 31).
Para isso observemos que
20 11 (mod 31) ()
e assim 20
2
(11)
2
(mod 31) 20
2
121 (mod 31). Como
121 3 (mod 31) temos
20
2
3 (mod 31). ()
Multiplicando () e () membro a membro, obtemos 20
3
33 (mod 31)
e, como 33 2 (mod 31),
20
3
2 (mod 31).
Elevando a 5, temos que 20
15
32 (mod 31) e como 32 1 (mod 31),
obtemos 20
15
1 (mod 31), como desejado.
0.6. CONGRU

ENCIAS 33
Exemplo 0.40. Encontre os restos das divisoes de
1. 3
1000
por 101
2. 5
3
20
por 13
Solu c

ao: Como 3
4
20 (mod 101), elevando ao quadrado obtemos
3
8
400 (mod 101) 3
8
4 (mod 101). Multiplicando por 3
2
,
obtemos 3
10
36 (mod 101). Portanto
3
20
1296 (mod 101) 3
20
17 (mod 101)
3
40
289 (mod 101) 3
40
14 (mod 101)
3
80
196 (mod 101) 3
80
6 (mod 101)
3
80
3
20
(6) (17) (mod 101) 3
100
1 (mod 101).
Assim, elevando a ultima congruencia a 10, obtemos 3
1000
1 (mod 101),
ou seja, 3
1000
deixa resto 1 na divisao por 101.
Para encontrar o resto da divisao de 5
3
20
por 13, note que como
5
4
1 (mod 13), os restos de 5
n
por 13 se repetem com perodo 4:
5
0
1 (mod 13) 5
4
1 (mod 13)
5
1
5 (mod 13) 5
5
5 (mod 13)
5
2
1 (mod 13) 5
6
1 (mod 13)
5
3
5 (mod 13) 5
7
5 (mod 13)
Por outro lado, temos 3 1 (mod 4) = 3
20
1 (mod 4), isto e,
3
20
deixa resto 1 na divisao por 4. Assim, 5
3
20
5
1
(mod 13), ou seja,
5
3
20
deixa resto 5 na divisao por 13.
O problema a seguir tem uma hist oria interessante. Em um artigo
publicado em 1969, D. J. Lewis armava que a equa cao x
3
117y
3
= 5
tem no m aximo 18 solucoes inteiras. Na verdade, ela nao possui ne-
nhuma, como foi provado dois anos mais tarde por R. Finkelstein e
H. London, utilizando metodos de Teoria Algebrica dos N umeros. Em
1973, F. Halter-Koch e V. St. Udresco observaram independentemente
que existe uma prova muito mais simples deste fato, como mostra o
exemplo a seguir.
Exemplo 0.41. Mostre que a equacao x
3
117y
3
= 5 n ao possui solu-
coes inteiras.
Solu c

ao: Observe que como 117 e m ultiplo de 9, qualquer solucao


inteira deve satisfazer
x
3
117y
3
5 (mod 9) x
3
5 (mod 9).
Porem, x s o pode deixar resto 0, 1, . . . , 8 na divisao por 9. Analisando
estes 9 casos, temos
34 CAP

ITULO 0. FUNDAMENTOS
x mod 9 0 1 2 3 4 5 6 7 8
x
3
mod 9 0 1 8 0 1 8 0 1 8
Ou seja, x
3
s o pode deixar resto 0, 1 ou 8 na divisao por 9. Logo x
3
5
(mod 9) e impossvel e a equa cao nao possui solucoes inteiras.
Exemplo 0.42 (AusPol2002). Encontrar todas as ternas (a, b, c) de in-
teiros n ao negativos tais que 2
a
+ 2
b
+ 1 e m ultiplo de 2
c
1.
Solu c

ao: O problema pede para determinar quando 2


a
+ 2
b
+ 1 0
(mod 2
c
1). Note que como 2
c
1 (mod 2
c
1), escrevendo a = cq
1
+a

e b = cq
2
+b

com 0 a

, b

< c temos que


2
a
+ 2
b
+ 1 0 (mod 2
c
1)
(2
c
)
q
1
2
a

+ (2
c
)
q
2
2
b

+ 1 0 (mod 2
c
1)
2
a

+ 2
b

+ 1 0 (mod 2
c
1)
que e o mesmo problema com a

e b

no lugar de a e b. Assim, basta


resolver o problema supondo 0 a, b < c. Temos alguns casos a analisar.
N ao ha solucoes com c = 0 e para c = 1 temos que (a, b, 1) e solucao
para todos os a, b 0. Se c = 2, temos que apenas (0, 0, 2) e solucao com
0 a, b < c = 2, o que da origem `as solucoes (2m, 2n, 2) para todos os m
e n naturais. Se c = 3, temos que apenas (1, 2, 3) e (2, 1, 3) s ao solucoes
com 0 a, b < c = 3, o que nos fornece solucoes (1 + 3m, 2 + 3n, 3) e
(2+3m, 1+3n, 3) para todos os m e n naturais. Finalmente, para c 4,
temos que se a < c 1 ou b < c 1, entao
3 2
a
+ 2
b
+ 1 2
c1
+ 2
c2
+ 1 = 3 2
c2
+ 1 < 2
c
1
e assim 2
a
+2
b
+1 nao pode ser m ultiplo de 2
c
1. Neste caso devemos
ter a = b = c 1 e 2
c1
+ 2
c1
+ 1 0 (mod 2
c
1) 2
c
+ 1 0
(mod 2
c
1) 2 0 (mod 2
c
1), o que nao ocorre pois 2
c
1 15
nao pode dividir 2. Logo nao ha solucoes neste ultimo caso.
Resumindo, as ternas pedidas s ao (m, n, 1), (2m, 2n, 2), (1 +3m, 2 +
3n, 3) e (2 + 3m, 1 + 3n, 3) onde m e n s ao naturais arbitr arios.
0.7 Bases
A nota cao usual para naturais e a chamada base 10, com algarismos
0, . . . , 9. Isto signica, por exemplo, que
196883 = 1 10
5
+ 9 10
4
+ 6 10
3
+ 8 10
2
+ 8 10
1
+ 3 10
0
.
O teorema abaixo mostra como escrever qualquer natural em qualquer
base d.
0.7. BASES 35
Teorema 0.43. Seja n 0 e d > 1. Ent ao existe uma unica sequencia
(os dgitosde n na base d) a
0
, . . . , a
k
, . . . com as seguintes propriedades:
1. para todo k, 0 a
k
< d,
2. existe m tal que se k m, ent ao a
k
= 0,
3. n =

k0
a
k
d
k
.
Demonstra c

ao: Escrevemos n = n
0
= n
1
d + a
0
, 0 a
0
< d, n
1
=
n
2
d + a
1
, 0 a
1
< d e em geral n
k
= n
k+1
d + a
k
, 0 a
k
< d. Nossa
primeira armacao e que n
k
= 0 para algum valor de k. De fato, se
n
0
< d
m
, entao n
1
=
n
0
d
< d
m1
e mais geralmente, por indu cao,
n
k
< d
mk
; fazendo k m temos n
k
< 1 donde n
k
= 0. Segue da que
a
k
= 0 para k m. A identidade do item 3 e facilmente demonstrada
por indu cao.
Para a unicidade, suponha

k0
a
k
d
k
=

k0
b
k
d
k
. Se as sequen-
cias a
k
e b
k
s ao distintas existe um menor ndice, digamos j, para o qual
a
j
= b
j
. Podemos escrever a
j
+

k>j
a
k
d
kj
= b
j
+

k>j
b
k
d
kj
donde
a
j
b
j
(mod d), o que e uma contradi cao, pois 0 < |a
j
b
j
| < d e
portanto a
j
b
j
nao pode ser um m ultiplo de d.
Ignorando os dgitos 0s iniciais, denotamos por (a
n
a
n1
a
1
a
0
)
d
o
natural cuja representa cao na base d tem dgitos a
k
como no teorema
acima:
(a
n
a
n1
a
1
a
0
)
d
def
=

0kn
a
k
d
k
.
Muitos dos famosos criterios de divisibilidade que aprendemos na
escola decorrem diretamente da representa cao acima. Por exemplo, se
N = (a
n
a
n1
a
1
a
0
)
10
, como 10 1 (mod 9), temos que 10
k
1
(mod 9), donde
N =

0kn
a
k
10
k

0kn
a
k
(mod 9).
Segue que N e a soma de seus dgitos na base 10 possuem o mesmo resto
na divisao por 9; em particular N e divisvel por 9 se, e s o se, a soma de
seus dgitos a
0
+ +a
n
e divisvel por 9.
De forma similar, para o criterio de divisibilidade por 11, observemos
que 10 1 (mod 11), logo
N =

0kn
a
k
10
k

0kn
(1)
k
a
k
(mod 11)
e assim um n umero e divisvel por 11 se, e s o se, a soma dos dgitos
em posicao par menos a soma dos dgitos em posicao mpar e divisvel
36 CAP

ITULO 0. FUNDAMENTOS
por 11. De igual forma, podemos encontrar criterios de divisibilidade
por 7, 13 e 37, que deixamos como exerccio para o leitor enuncia-los e
demonstra-los (utilize o fato que 10
3
1 (mod 7), 10
3
1 (mod 13)
e 10
3
1 (mod 37)).
Exemplo 0.44. Encontrar os ultimos dois algarismos em representa cao
decimal de 3
200
.
Solu c

ao: Como
(a
n
a
n1
a
1
a
0
)
10
= 10
2
(a
n
10
n2
+ +a
2
) + (10 a
1
+a
0
)
= 100 (a
n
. . . a
2
)
10
+ (a
1
a
0
)
10
temos que o n umero formado pelos dois ultimos algarismos de (a
n
a
1
a
0
)
10
e o resto da divisao deste n umero por 100, logo o problema se resume a
calcular 3
200
m odulo 100. Podemos utilizar o bin omio de Newton para
simplicar as contas:
3
200
= 9
100
= (10 1)
100
=

0k100
_
100
k
_
10
100k
(1)
k
,
logo 3
200

_
100
99
_
10 +
_
100
100
_
(mod 100) 3
200
1 (mod 100) e
assim os dois ultimos dgitos de 3
200
s ao 01.
Exemplo 0.45. Demonstrar que, para todo n natural mpar,
s
n
= 2
2n
(2
2n+1
1)
termina em 28 quando escrito em notacao decimal.
Solu c

ao: Vamos mostrar por indu cao em n que s


n
termina em 28.
Para n = 1 temos que s
1
= 28. Suponhamos que para algum n 1
mpar s
n
termina em 28 e vamos mostrar que s
n+2
termina em 28 ou,
equivalentemente, que 100 | s
n+2
s
n
. Temos
s
n+2
s
n
= 2
2(n+2)
(2
2(n+2)+1
1) 2
2n
(2
2n+1
1)
= 2
2n
(16 2
2n+5
16 2
2n+1
+ 1)
= 5 2
2n
(51 2
2n+1
3).
Como, para n mpar,
2
2
1 (mod 5) = 2
2n
1 (mod 5)
= 2
2n+1
2 (mod 5),
temos que 51 2
2n+1
3 1 (2) 3 (mod 5) 51 2
2n+1
3 0
(mod 5). Assim, s
n+2
s
n
e divisvel por 5 4 5 = 100.
0.8. O ANEL DE INTEIROS M

ODULO N 37
0.8 O Anel de Inteiros M odulo n
As semelhan cas entre as rela coes de congruencia m odulo n e igual-
dade nao s ao mero fruto do acaso, ambas s ao inst ancias de rela coes de
equivalencia em Z. Em geral, uma rela cao sobre um conjunto X e
dita de equivalencia se ela e reexiva (x x para todo x X), simetrica
(x y y x) e transitiva (x y e y z = x z).
Dar uma rela cao de equivalencia em X e o mesmo que dar uma
parti cao X =

de X, i.e., uma colecao de subconjuntos X

= ,
dois a dois disjuntos, cuja uni ao e X. De fato, dada a parti cao acima,
podemos denir uma rela cao de equivalencia declarando que x y se,
e somente se, x e y pertencem a um mesmo X

. Reciprocamente, se e
uma rela cao de equivalencia, dado um elemento x X podemos denir
a classe de equivalencia x de x como o conjunto de todos os elementos
equivalentes a x:
x = {y X | y x}.
Observe que ou x y = (se x y) ou x = y (se x y). Assim,
as distintas classes de equivalencia x formam uma parti cao de X. O
conjunto {x | x X} das classes de equivalencia de e chamado de
quociente de X por e e denotado por X/. Intuitivamente, X/ e o
conjunto obtido igualando-se elementos equivalentes entre si.
Agora aplicamos esta construcao geral ao nosso caso. O quociente de
Z pela rela cao (mod n) e chamado de anel de inteiros modulo n e e
denotado por uma das nota coes Z/(n), Z/nZ, Z/n ou `as vezes Z
n
. Por
exemplo, para n = 2, temos que Z/2Z possui apenas dois elementos,
0 e 1 (popularmente conhecidos como conjunto dos pares e mpares,
respectivamente).
A denicao de a como um subconjunto de Z raramente sera impor-
tante, sendo apenas uma maneira de formalizar o fato de que estamos
identicandotodos os inteiros que deixam o mesmo resto na divisao por
n (como no exemplo dos pares e mpares acima). Assim, o importante
e sabermos que
a = a

a a

(mod n)
a e a

deixam o mesmo resto na divisao por n.


Se n > 0, a divisao euclidiana diz que todo inteiro a e congruo a um
unico inteiro a

com 0 a

< n; podemos reescrever este fato na nossa


nova linguagem como
Z/(n) = {0, 1, . . . , n 1}.
Os itens (4) e (5) da proposicao 0.38 dizem que as operacoes de soma,
diferen ca e produto s ao compatveis com a rela cao de congruencia. Uma
38 CAP

ITULO 0. FUNDAMENTOS
formulacao mais abstrata da mesma ideia e dizer que as operacoes +,
e passam ao quociente, i.e., que podemos denir a soma, subtra cao e o
produto de classes de congruencia por
a +b = a +b
a b = a b
a b = a b
respectivamente. A d uvida `a primeira vista seria se a escolha de a e b
nao afeta a resposta: anal existem innitos inteiros a

e b

com a = a

e
b = b

. Os itens (4) e (5) da proposicao s ao exatamente o que precisamos:


eles nos dizem que nestas condi coes a b = a

e a b = a

, de
modo que as operacoes acima estao bem denidas.
Por exemplo, emZ/6Z temos as seguintes tabelas de soma e produto:
+ 0 1 2 3 4 5
0 0 1 2 3 4 5
1 1 2 3 4 5 0
2 2 3 4 5 0 1
3 3 4 5 0 1 2
4 4 5 0 1 2 3
5 5 0 1 2 3 4
e
0 1 2 3 4 5
0 0 0 0 0 0 0
1 0 1 2 3 4 5
2 0 2 4 0 2 4
3 0 3 0 3 0 3
4 0 4 2 0 4 2
5 0 5 4 3 2 1
A proxima proposicao diz quando podemos dividir por a m odulo
n, isto e, quando o inverso multiplicativo de a m odulo n esta denido:
Proposi cao 0.46. Sejam a, n Z, n > 0. Ent ao existe b Z com
ab 1 (mod n) se, e somente se, mdc(a, n) = 1.
Demonstra c

ao: Temos que ab 1 (mod n) admite solucao na va-


ri avel b se, e somente se, existem b, k Z tais que ab 1 = nk
ab nk = 1. Pelo corolario 0.21 do teorema de Bachet-Bezout, isto
ocorre se, e s o se, mdc(a, n) = 1.
Dizemos portanto que a e invertvel m odulo n quando mdc(a, n) = 1
e chamamos b com ab 1 (mod n) de inverso multiplicativo de a m odulo
n. O inverso e sempre unico m odulo n: se ab ab

1 (mod n) temos
b b 1 b (ab

) (ba) b 1 b

(mod n).
Assim, b esta bem denido e, em termos de classes de congruencia, temos
que a b = 1; denotamos b por (a)
1
. Note que pela demonstra cao
da proposicao acima calcular (a)
1
e equivalente a resolver a equa cao
diofantina linear ax +ny = 1 e para isto podemos utilizar o metodo do
exemplo 0.27.
0.8. O ANEL DE INTEIROS M

ODULO N 39
Denimos o grupo de unidades (Z/nZ)

Z/nZ do anel de inteiros


m odulo n como o subconjunto formado pelos elementos invertveis de
Z/nZ:
(Z/nZ)

= {a Z/nZ | mdc(a, n) = 1}.


Observe que o produto de elementos de (Z/nZ)

e sempre um elemento
de (Z/nZ)

. Por exemplo, temos a seguinte tabela de multiplica cao em


(Z/15Z)

:
1 2 4 7 8 11 13 14
1 1 2 4 7 8 11 13 14
2 2 4 8 14 1 7 11 13
4 4 8 1 13 2 14 7 11
7 7 14 13 4 11 2 1 8
8 8 1 2 11 4 13 14 7
11 11 7 14 2 13 1 8 4
13 13 11 7 1 14 8 4 2
14 14 13 11 8 7 4 2 1
Uma aplicacao do inverso multiplicativo e o famoso teorema de Wil-
son. Primeiramente precisamos de um lema.
Lema 0.47. Se p e primo, ent ao as unicas solu coes de x
2
= 1 em Z/(p)
sao 1 e 1. Em particular, se x (Z/(p))

{1, 1}, ent ao x


1
= x
em Z/(p).
Demonstra c

ao: Temos
x
2
1 (mod p) p | (x
2
1) p | (x 1)(x + 1)
p | x 1 ou p | x + 1
x 1 (mod p) ou x 1 (mod p)
donde o resultado segue.
Teorema 0.48 (Wilson). Seja n > 1. Ent ao n | (n1)! +1 se, e so se,
n e primo. Mais precisamente,
(n 1)!
_
1 (mod n) se n e primo
0 (mod n) se n e composto e n = 4.
Demonstra c

ao: Se n e composto mas nao e o quadrado de um primo


podemos escrever n = ab com 1 < a < b < n. Neste caso tanto a quanto
b s ao fatores de (n 1)! e portanto (n 1)! 0 (mod n). Se n = p
2
,
p > 2, entao p e 2p s ao fatores de (n 1)! e novamente (n 1)! 0
(mod n); isto demonstra que para todo n = 4 composto temos (n1)!
0 (mod n).
40 CAP

ITULO 0. FUNDAMENTOS
Se n e primo podemos escrever (n1)! 2 3 . . . (n2) (mod n);
mas pelo lema anterior podemos juntar os inversos aos pares no produto
do lado direito, donde (n 1)! 1 (mod n).
Vejamos uma aplicacao do teorema de Wilson.
Teorema 0.49 (Teorema de Wolstenholme). Seja p > 3 um n umero
primo. Ent ao o numerador do n umero
1 +
1
2
+
1
3
+ +
1
p 1
e divisvel por p
2
.
Demonstra c

ao: Note que somando os extremos temos

1ip1
1
i
=

1i
p1
2
_
1
i
+
1
p i
_
= p

1i
p1
2
1
i(p i)
.
Como o mmc dos n umeros de 1 a p1 nao e divisvel por p, basta mostrar
que o numerador da ultima soma e m ultiplo de p. Equivalentemente,
como p (p 1)!, devemos mostrar que o inteiro
S
def
=

1i
p1
2
(p 1)!
i(p i)
e um m ultiplo de p. Para 1 i p 1, denote por r
i
o inverso de
i mod p, ou seja, ir
i
1 (mod p). Note que r
pi
r
i
(mod p), assim
S

1i
p1
2
(p 1)!
i(p i)
ir
i
(p i)r
pi

1i
p1
2
(p 1)!r
i
r
pi

1i
p1
2
r
2
i
(mod p)
pelo teorema de Wilson. Note que como cada r
i
e congruente a um
dos n umeros 1, 2, . . . ,
p1
2
, temos que os r
2
i
s ao congruentes a um
dos n umeros 1
2
, 2
2
, . . . , (
p1
2
)
2
m odulo p. Vamos mostrar que todos eles
aparecem. De fato, se r
2
i
r
2
j
(mod p), entao p | (r
i
r
j
)(r
i
+ r
j
), isto
e, r
i
r
j
(mod p). Multiplicando por ij, temos que j i (mod p),
o implica i = j pois 1 i, j
p1
2
.
Assim, S

1i
p1
2
i
2
(mod p) e como

1i
p1
2
i
2
=
p(p
2
1)
24
e
um m ultiplo de p (pois mdc(p, 24) = 1), o resultado segue.
0.8. O ANEL DE INTEIROS M

ODULO N 41
O teorema de Wilson produz ainda resultados interessantes sobre os
coecientes binomiais. Suponhamos que k e h s ao inteiros positivos tais
que k +h = p 1 onde p e primo. Entao
h!k! (1)
h
(p 1)(p 2) (p h)k! = (1)
k
(p 1)!
(1)
k+1
(mod p).
Portanto
h!k!
_
p 1
k
_
(p 1)! (mod p)
(1)
k+1
_
p 1
k
_
1 (mod p)

_
p 1
k
_
(1)
k
(mod p).
Exemplo 0.50. Demonstrar que se p > 3 e primo, ent ao p
3
|
_
2p
p
_
2.
Solu c

ao: Primeiramente, vamos relembrar algumas identidades com


coecientes binomiais bem conhecidas. Para todo 1 i p 1, temos
que
_
p
i
_
=
p
i
_
p1
i1
_
(basta utilizar a denicao) enquanto que
_
2p
p
_
=
_
p
0
_
2
+
_
p
1
_
2
+ +
_
p
p
_
2
pois podemos escolher p objetos dentre 2p escolhendo i objetos dentre
os p primeiros e p i dos p ultimos para todo i entre 0 e p, logo
_
2p
p
_
=

0ip
_
p
i
__
p
p i
_
=

0ip
_
p
i
_
2
.
Utilizando estas identidades, temos que
_
2p
p
_
2 =

1ip1
p
2
i
2
_
p 1
i 1
_
2
= p
2

1ip1
1
i
2
_
p 1
i 1
_
2
.
Note que
_
p
i
_
=
p!
i!(pi)!
e um m ultiplo de p para 1 i p 1 pois
o denominador desta fracao nao e divisvel por p. Assim,
1
i
2
_
p1
i1
_
2
=
1
p
2
_
p
i
_
2
e inteiro e portanto a soma

1ip1
1
i
2
_
p1
i1
_
2
e inteira e devemos
mostrar que ela e um m ultiplo de p. Para isto observemos que cada
1 i p1 e invertvel m odulo p; seja r
i
tal que 1 r
i
p1 e ir
i
1
(mod p). Pela unicidade de r
i
m odulo p, temos que os r
i
s formam uma
42 CAP

ITULO 0. FUNDAMENTOS
permutacao de 1, 2, . . . , p 1. Assim, como
_
p1
i1
_
(1)
i1
(mod p),
temos

1ip1
1
i
2
_
p 1
i 1
_
2

1ip1
(ir
i
)
2
i
2
_
p 1
i 1
_
2
(mod p)

1ip1
1
i
2
_
p 1
i 1
_
2

1ip1
r
2
i
=

1ip1
i
2
(mod p).
Como

1ip1
i
2
=
p(p1)(2p1)
6
e um m ultiplo de p (pois mdc(p, 6) =
1), a prova acaba.
Os termos grupo e anel empregados nesta secao estao em conformi-
dade com o jarg ao usualmente utilizado em

Algebra. Grupo e o nome
emprestado a um conjunto G juntamente com uma operacao bin aria
(produto) que satisfaz os seguintes tres axiomas:
1. (Associatividade) Para quaisquer a, b, c G, (a b) c = a (b c).
2. (Existencia de elemento neutro) Existe um elemento e G tal que,
para todo a G, a e = e a = a.
3. (Existencia de inverso) Para qualquer elemento a G existe um
elemento a
1
G tal que a a
1
= a
1
a = e.
Se, alem dos tres axiomas acima, o grupo G satisfaz
4. (Comutatividade) Para quaisquer a, b G, a b = b a.
entao G e chamado de grupo abeliano.
Um anel e um conjunto A com duas operacoes bin arias + (soma) e
(produto) satisfazendo axiomas que abstraem as propriedades usuais
dos inteiros (por exemplo). Estes axiomas s ao
1. (A, +) e um grupo abeliano com elemento neutro 0.
2. (Associatividade do produto) (ab)c = a(bc) para todo a, b, c A.
3. (Elemento neutro do produto) Existe um elemento 1 A tal que
1 a = a 1 = a para todo a A.
4. (Distributividade) a (b +c) = a b +a c e (b +c) a = b a +c a
para todo a, b, c A.
Se a b = b a para todo a, b A, dizemos que o anel A e comutativo.
Um anel comutativo A = 0 (isto e, 0 = 1 em A) e chamado de domnio
se, para a, b A, a b = 0 = a = 0 ou b = 0. Por outro lado, se um
anel comutativo A = 0 e tal que todo elemento nao nulo possui inverso
multiplicativo (ou seja, (A\{0}, ) e um grupo) entao dizemos que o anel
A e um corpo. Um importante resultado e a seguinte
0.9. A FUN C

AO DE EULER E O TEOREMA DE EULER-FERMAT43


Proposi cao 0.51. O anel Z/nZ e um corpo se, e so se, n e primo.
Demonstra c

ao: Temos que Z/nZ e um corpo se, e somente se, todo


elemento a = 0 e invertvel, ou seja, se e somente se, mdc(a, n) = 1 para
todo a com 0 < a < n. Mas isto e equivalente a n ser primo, pois se n e
composto e a | n com 1 < a < n, entao mdc(a, n) = a = 1.
Um fato curioso e muito util quando trabalhamos no corpo Z/pZ (p
primo) e a seguinte
Proposi cao 0.52 (Sonho de todo estudante). Seja p um primo. Ent ao
em Z/pZ temos
(a +b)
p
= a
p
+b
p
para quaisquer a, b Z/pZ.
Demonstra c

ao: Devemos mostrar que (a + b)


p
a
p
+ b
p
(mod p)
para todo a, b Z. Temos que se 0 < k < p
_
p
k
_
=
p!
k!(p k)!
0 (mod p)
pois ha um fator p no numerador que nao pode ser cancelado com nada
que apareca no denominador. Assim, utilizando o bin omio de Newton,
temos
(a +b)
p
=

0kp
_
p
k
_
a
pk
b
k
a
p
+b
p
(mod p)
como queramos mostrar.
0.9 A Funcao de Euler e o Teorema de Euler-
Fermat
Dizemos que um conjunto de n n umeros inteiros a
1
, . . . , a
n
forma um
sistema completo de restos modulo n (scr) se
{a
1
, a
2
, . . . , a
n
} = Z/(n),
isto e, se os a
i
representam todas as classes de congruencia m odulo n. Por
exemplo, 0, 1, 2, . . . , n 1 formam um scr m odulo n. Equivalentemente,
podemos dizer que a
1
, a
2
, . . . , a
n
formam um scr m odulo n se, e somente
se, a
i
a
j
(mod n) implicar i = j.
De igual forma, dizemos que os n umeros inteiros b
1
, b
2
, . . . , b
(n)
for-
mam um sistema completo de invertveis modulo n (sci) se
{b
1
, b
2
, . . . , b
(n)
} = (Z/(n))

,
44 CAP

ITULO 0. FUNDAMENTOS
onde (n) representa o n umero de elementos de (Z/(n))

. Em outras
palavras, b
1
, b
2
, . . . , b
(n)
formam um sci m odulo n se, e somente se,
representam todas as classes de congruencia invertveis m odulo n ou,
equivalentemente, mdc(b
i
, n) = 1 para todo i e b
i
b
j
(mod n) implica
i = j. O conjunto {k Z | 1 k n e mdc(n, k) = 1} e um exemplo
de sci m odulo n.
Denicao 0.53. A fun cao
(n)
def
= |(Z/nZ)

|
e chamada de funcao phi de Euler.
Temos (1) = (2) = 1 e, para n > 2, 1 < (n) < n. Se p e primo,
(p) = p1; mais geralmente (p
k
) = p
k
p
k1
pois mdc(a, p
k
) = 1 se,
e somente se, a nao e m ultiplo de p e ha p
k1
m ultiplos de p no intervalo
1 a p
k
. Para calcular a funcao no caso geral, vamos mostrar que
se mdc(n, m) = 1, entao (nm) = (n)(m). Consideremos os n umeros
1, 2, . . . , nm, onde mdc(n, m) = 1 e os arrumamos em forma matricial
assim:
1 2 3 . . . n
n + 1 n + 2 n + 3 . . . 2n
.
.
.
.
.
.
.
.
.
.
.
.
.
.
.
n(m1) + 1 n(m1) + 2 n(m1) + 3 . . . n(m1) +n
Note que, como mdc(ni +j, n) = mdc(j, n), se um n umero nesta tabela
e primo relativo com n, entao todos os n umeros nessa coluna s ao primos
relativos com n. Logo existem (n) colunas nas quais todos os n umeros
s ao primos relativos com n. Por outro lado, toda coluna possui um
conjunto completo de restos m odulo m: se duas entradas s ao tais que
ni
1
+ j ni
2
+ j (mod m), entao i
1
i
2
(mod m) pois n e invertvel
m odulo m ja que mdc(m, n) = 1, logo como 0 i
1
, i
2
< m devemos
ter i
1
= i
2
. Desta forma, em cada coluna existem exatamente (m)
n umeros que s ao primos relativos com m e portanto o total de n umeros
nesta tabela que s ao simultaneamente primos relativos com m e n (i.e.
primos com nm) e (nm) = (n)(m).
Assim, se n = p

1
1
p

k
k
e a fatora cao de n em potencias de primos
distintos p
i
, temos que
(n) =

1ik
(p

i
i
) =

1ik
(p

i
i
p

i
1
i
) = n

1ik
_
1
1
p
i
_
.
Agora estamos prontos para enunciar e provar o importante
0.9. A FUN C

AO DE EULER E O TEOREMA DE EULER-FERMAT45


Teorema 0.54 (Euler-Fermat). Sejam a e m > 0 sao dois inteiros com
mdc(a, m) = 1, ent ao
a
(m)
1 (mod m).
Demonstra c

ao: Observemos que se r


1
, r
2
, . . . , r
(m)
e um sistema
completo de invertveis m odulo m e a e um n umero natural tal que
mdc(a, m) = 1, entao ar
1
, ar
2
, . . . , ar
(m)
tambem e um sistema com-
pleto de invertveis m odulo m. De fato, temos que mdc(ar
i
, m) = 1 para
todo i e se ar
i
ar
j
(mod m), entao r
i
r
j
(mod m) pois a e invertvel
m odulo m, logo r
i
= r
j
e portanto i = j. Consequentemente cada ar
i
deve ser congruente com algum r
j
e, portanto,

1i(m)
(ar
i
)

1i(m)
r
i
(mod m)
a
(m)

1i(m)
r
i

1i(m)
r
i
(mod m).
Mas como cada r
i
e invertvel m odulo m, simplicando o fator

1i(m)
r
i
,
obtemos o resultado desejado.
Como caso particular do teorema anterior obtemos o
Teorema 0.55 (Pequeno Teorema de Fermat). Seja a um inteiro posi-
tivo e p um primo, ent ao
a
p
a (mod p)
Demonstra c

ao: De fato, observemos que se p | a o resultado e evi-


dente. Entao, podemos supor que mdc(a, p) = 1. Como (p) = p 1,
pelo teorema de Euler temos a
p1
1 (mod p), logo multiplicando por
a obtemos o resultado desejado.
Observa cao 0.56. O teorema de Euler-Fermat tambem pode ser pro-
vado utilizando-se o seguinte corol ario do teorema de Lagrange em Teoria
dos Grupos: se G e um grupo nito e g G, ent ao g
|G|
= e (identidade).
Aplicando este resultado para G = (Z/mZ)

, temos que a
(m)
= 1 para
todo a (Z/mZ)

, que e uma formula cao equivalente para o teorema


de Euler-Fermat.
Observemos que o teorema de Euler-Fermat pode ser otimizado da
seguinte forma:
46 CAP

ITULO 0. FUNDAMENTOS
Proposi cao 0.57. Sejam a e n n umeros inteiros tais que mdc(a, n) = 1
e n se fatora como n = p

1
1
p

2
2
. . . p

k
k
em potencias de primos distintos.
Ent ao
a
M
1 (mod n) onde M = mmc((p

1
1
), (p

2
2
), . . . , (p

k
k
)).
Demonstra c

ao: Pelo teorema de Euler-Fermat sabemos que a


(p

j
j
)

1 (mod p

j
j
) para todo j = 1, . . . k. Elevando a M/(p

j
j
), obtemos
a
M
1 (mod p

j
j
). Assim, a
M
1 e m ultiplo de p

j
j
para todo j
e como estes n umeros s ao dois a dois primos entre si conclumos que
n | a
M
1 a
M
1 (mod n), como desejado.
Vejamos agora algumas aplicacoes do teorema de Euler-Fermat.
Exemplo 0.58. Mostre que existem innitos n umeros da forma
20000 . . . 009
que sao m ultiplos de 2009.
Demonstra c

ao: O problema e equivalente a encontrar innitos na-


turais k tais que
2 10
k
+ 9 0 (mod 2009) 2 10
k
+ 9 2009 (mod 2009)
10
k3
1 (mod 2009)
pois 2000 e invertvel m odulo 2009. Como mdc(10, 2009) = 1, pelo
teorema de Euler-Fermat temos que 10
(2009)
1 (mod 2009) =
10
(2009)t
1 (mod 2009) para todo t N, logo basta tomar k =
(2009)t + 3.
Exemplo 0.59. Encontre um n umero n N tal que 2
n
> 10
2000
e
2
n
tenha entre suas 2000 ultimas casas decimais pelo menos 1000 zeros
consecutivos.
Solu c

ao: Sabemos que 2


(5
2000
)
1 (mod 5
2000
) pelo teorema de
Euler-Fermat. Portanto existe b N com
2
(5
2000
)
= 5
2000
b + 1 = 2
2000+(5
2000
)
= 10
2000
b + 2
2000
.
Portanto os 2000 ultimos dgitos de 2
2000+(5
2000
)
coincidem com a repre-
senta cao decimal de 2
2000
, que tem no m aximo 667 dgitos pois 2
2000
<
(2
3
)
667
< 10
667
. Desta forma, ha pelo menos 2000 667 = 1333 zeros
consecutivos dentre as 2000 ultimas casas decimais de 2
2000+(5
2000
)
e
assim n = (5
2000
) + 2000 = 4 5
1999
+ 2000 satisfaz as condi coes do
enunciado.
0.9. A FUN C

AO DE EULER E O TEOREMA DE EULER-FERMAT47


Exemplo 0.60. Mostre que n ao existe inteiro x tal que 103 | x
3
2.
Solu c

ao: Note primeiramente que 103 e primo. Agora suponha que


x
3
2 (mod 103), de modo que 103 x. Elevando ambos os lados
desta congruencia a (103 1)/3 = 34, obtemos x
102
2
34
(mod 103) e
sabemos pelo teorema de Euler-Fermat que x
102
1 (mod 103). Porem,
fazendo as contas, obtemos que 2
34
46 (mod 103), uma contradi cao.
Logo nao ha inteiro x tal que 103 | x
3
2.
Utilizando o mesmo raciocnio do exemplo anterior, temos que se p e
um primo tal que p 1 (mod 3) e p a, entao uma condi cao necessaria
para que x
3
a (mod p) tenha solucao emx e que a
(p1)/3
1 (mod p).
Esta condi cao tambem e suciente, pela existencia de razes primitivas
m odulo p, como mostraremos no nal deste captulo.
Exemplo 0.61. Demonstrar que se p > 2 e primo, ent ao
1
p1
+ 2
p1
+ 3
p1
+ + (p 1)
p1
p + (p 1)! (mod p
2
).
Solu c

ao: Pelo pequeno teorema de Fermat, sabemos que i


p1
1
(mod p) para todo 1 i p 1, isto e, que i
p1
= k
i
p +1 onde k
i
e um
inteiro. Assim, 1
p1
+2
p1
+ +(p1)
p1
= (k
1
+k
2
+ +k
p1
)p+p1
e portanto devemos mostrar que (k
1
+ k
2
+ + k
p1
)p (p 1)! + 1
(mod p
2
).
Multiplicando as equa coes i
p1
= k
i
p + 1, temos
(k
1
p+1)(k
2
p+1) (k
p1
p+1) = 1
p1
2
p1
(p1)
p1
= ((p1)!)
p1
.
Por um lado, (k
1
p+1)(k
2
p+1) (k
p1
p+1) (k
1
+k
2
+ +k
p1
)p+1
(mod p
2
). Por outro, pelo teorema de Wilson sabemos que (p1)! 1
(mod p), ou seja, (p 1)! = Kp 1 para algum K inteiro. Segue que
(k
1
+k
2
+ +k
p1
)p + 1 (Kp 1)
p1
(mod p
2
)
= (k
1
+k
2
+ +k
p1
)p + 1 1
_
p 1
1
_
Kp (mod p
2
)
= (k
1
+k
2
+ +k
p1
)p Kp (mod p
2
)
= (k
1
+k
2
+ +k
p1
)p (p 1)! + 1 (mod p
2
)
o que encerra a prova.
Conclumos esta secao apresentando brevemente uma aplicacao do
Teorema de Euler que tem particular interesse pratico: a Criptograa
RSA. Trata-se de um metodo de criptograa com chave p ublica, isto
e, um metodo que permite a qualquer pessoa transmitir mensagens por
48 CAP

ITULO 0. FUNDAMENTOS
uma via insegura (ou seja, que pode ser monitorada por espi oes) de
modo que, na pratica, apenas o legtimo destinatario, que conhece uma
chave, pode recuperar a mensagem original. A sigla vem dos nomes de
Ron Rivest, Adi Shamir, e Leonard Adleman, que desenvolveram esse
metodo.
Para isso, o receptor publica um inteiro N que e o produto de dois
primos razoavelmente grandes p e q (aproximadamente da mesma ordem
de grandeza); N e p ublico mas a sua fatora cao pq s o e conhecida pelo
receptor. O receptor tambem publica um expoente s (em geral nao muito
grande) com mdc(s, (p 1)(q 1)) = 1. O receptor calcula (usando o
algoritmo de Euclides) o inverso de s mod (p 1)(q 1) = (N), isto e,
um natural r < (p 1)(q 1) com rs 1 (mod (p 1)(q 1)) (donde
rs = 1+k(N), para algum natural k); esse r e chamado a chave privada
da criptograa. Note que apesar de N e s serem p ublicos, nao parece ser
facil calcular (N) ou r (neste contexto, calcular (N) = (p 1)(q 1)
dado N = pq e equivalente a fatorar N, i.e., a encontrar os fatores primos
p e q).
Uma mensagem e um n umero natural m < N. O emissor envia (ou
publica) m := m
s
(mod N), com 0 < m < N. O receptor recupera m
via
m m
r
(mod N).
Para vericar essa equivalencia, podemos observar que
m
r
(m
s
)
r
= m
rs
= m
1+k(p1)(q1)
= m (m
p1
)
k(q1)
m (mod p);
note que, se p | m, os dois lados s ao 0 mod p, e, caso contrario, m
p1
1
(mod p); analogamente m
r
m (mod q), donde m
r
m (mod N). Es-
sas tarefas s ao relativamente rapidas computacionalmente. Mais precisa-
mente, veremos a seguir que existem algoritmos polinomiais para testar
primalidade, assim como para as demais operacoes necess arias (veja o
captulo 7, especialmente a secao sobre o teste de Agrawal, Kayal e Sa-
xena que garante que testar primalidade de um n umero da ordem de N
leva tempo no m aximo polinomial em log N).
Se existem algoritmos polinomiais para testar primalidade, nao e
verdade que sejam conhecidos algoritmos polinomiais (e determinsti-
cos) para obter primos novos de uma determinada ordem de grandeza.
Pelo teorema dos n umeros primos (captulo 5 e apendice A), para todo
N grande, a probabilidade de um n umero escolhido ao acaso entre N
e 2N ser primo e (1 + o(1))/ log N, o que implica que, se testarmos
C log N n umeros ao acaso entre N e 2N, a probabilidade de algum deles
ser primo e da ordem de 1exp(C(1+o(1))), que esta muito perto de
1 para C grande. Se ao inves de sortear n umeros procurarmos o menor
primo maior ou igual a N (testando um por um) entao, novamente pelo
0.9. A FUN C

AO DE EULER E O TEOREMA DE EULER-FERMAT49


teorema dos n umeros primos, em media o n umero de tentativas sera da
ordem de log(n). Entretanto, ha gaps bem maiores do que log N e sabe-
se muito pouco sobre o tamanho dos gaps (para um primo p, o gap g(p)
e igual a q p onde q e o menor primo maior do que p). Por exemplo,
Harald Cramer conjectura que g(p) < C(log(p))
2
(para algum C > 0;
[5]): se isto for verdade entao o algoritmo proposto acima e realmente
polinomial. Pode ser que outra estrategia permita encontrar primos sem
demonstrar esta conjectura, mas nada de tempo polinomial e conhecido.
H a um projeto Polymath sobre este assunto: veja o preprint [10] e as
paginas indicadas juntamente nas referencias. Ainda assim, podemos
considerar que o problema de obter primos e razoavelmente facil e ra-
pido para aplicacoes praticas pois a devemos permitir algoritmos que
dependem de sorteios e que obtem o que e pedido em tempo polinomial
com probabilidade quase igual a 1. No interessante artigo de divulgacao
[12] e discutido o problema de gerar primos grandes, e em particular e
apresentado um algoritmo que funciona em muitos casos e gera primos
grandes cuja primalidade pode ser vericada por criterios bem mais sim-
ples que o teste de Agrawal, Kayal e Saxena, como o teste de Pocklington
(veja o captulo 7).
N ao se conhecem algoritmos polinomiais para fatorar inteiros (gran-
des). A maioria dos especialistas duvida que exista tal algoritmo mas
e preciso enfatizar que a nao-existencia de um tal algoritmo nao e um
teorema. Mais do que isso, a nao-existencia de tal algoritmo implica di-
retamente em P = NP (um dos mais importantes problemas em aberto
da matem atica) mas P = NP nao parece implicar a nao existencia do
algoritmo.
Existe ainda a possibilidade de que nao exista um algoritmo rapido,
mas que ainda assim exista uma m aquina (no sentido literal) capaz de
fatorar inteiros rapidamente. De fato, a mecanica quantica parece per-
mitir a construcao de um computador qu antico e Peter Shor encontrou
um algoritmo que permite a um computador quantico fatorar inteiros
em tempo polinomial [16]. Ate 2010 foram construdos computadores
quanticos mnimos, sucientes para fatorar o n umero 15 pelo algoritmo
de Shor mas insucientes para n umeros maiores [11]. N ao e claro se sera
possvel construir computadores quanticos maiores.
Resumindo, a criptograa RSA e eciente e segura pois e muito mais
rapido achar primos grandes do que fatorar n umeros grandes e ele e
bastante utilizado para encriptar mensagens transmitidas pela internet.
Para mais informa coes sobre a criptograa RSA, veja [4].
Exemplo 0.62. Sabendo que a chave p ublica de criptograa RSA sao
os n umeros N = 24797 e s = 143, determine a chave privada.
Solu c

ao: Vamos fatorar N = 24797 usando o metodo de Fermat que


50 CAP

ITULO 0. FUNDAMENTOS
consiste em encontrar dois n umeros a e b tais que N = a
2
b
2
. De fato,
se x
0
=

24797 + 1 = 158 temos


x
i
_
x
2
i
24797
158

167
159 22
,
logo 24797 = (159 + 22) (159 22) = 181 137. Observemos que o
metodo de Fermat e computacionalmente efetivo quanto os dois fatores
do n umero estao proximos. Portanto (24797) = (181) (137) =
180 136 = 24480. Segue que a chave privada e a solucao da congruencia
143x 1 (mod 24480). Esta congruencia pode ser resolvida usando o
algoritmo estendido da divisao
resto quocente x
24480 0
143 1
27 171 171
8 5 856
3 3 2739
2 2 6634
1 1 9073
Portanto a chave privada e 24480 9073 = 15407.
Problemas Propostos
0.51. Demonstrar que
(a) 61 | 20
15
1.
(b) 13 | 2
70
+ 3
70
.
0.52. Encontrar os ultimos 3 dgitos de 3
2009
em notacao decimal.
0.53. Vericar se 987654321 e divisvel por 9, 11, 13, 17 ou 19.
0.54. Demonstrar que todo n umero palndromo com um n umero par de
dgitos e divisvel por 11. O que acontece com os n umeros palndromos
com um n umero mpar de dgitos?
0.55. Encontrar todos os n umeros N de tres dgitos em representa cao
decimal, tais que N e divisvel por 11 e alem disso N/11 e igual `a soma
dos quadrados dos dgitos de N.
0.9. A FUN C

AO DE EULER E O TEOREMA DE EULER-FERMAT51


0.56. Mostre que o dgito das dezenas de qualquer potencia de 3 e um
n umero par (por exemplo, o dgito das dezenas de 3
6
= 729 e 2).
0.57. Mostre que, para todo n 0, vale que 13 | 7
2n+1
+ 6
2n+1
.
0.58. Encontre todas as solu coes da congruencia x
2
1 (mod 30).
Conclua que existem valores de x tais que 30 n ao divide x + 1 nem
x 1 mas divide x
2
1. Generalize esse resultado.
0.59. Encontre um n umero positivo k < 50 tal que a
k
1 (mod 99)
para todo inteiro a primo relativo com 99.
0.60. Mostre que para todo inteiro a temos que a
561
a (mod 561) e
a
1105
a (mod 1105), mas 561 e 1105 n ao sao primos, o que mostra
que o recproco do pequeno teorema de Fermat e falso.
0.61. Mostre que
a
12
b
12
(mod 91) mdc(a, 91) = mdc(b, 91).
0.62. (P. Sabini) Mostre que entre os n umeros da forma
14, 144, 1444, 14444, 144 44, . . .
os unicos quadrados perfeitos sao 144 = 12
2
e 1444 = 38
2
.
0.63. Seja f : N
>0
N uma fun cao denida do conjunto dos inteiros
positivos no conjunto dos n umeros naturais tal que
(a) f(1) = 0;
(b) f(2n) = 2f(n) + 1;
(c) f(2n + 1) = 2f(n).
Utilize a representa cao em base 2 de n para encontrar uma formula n ao
recursiva para f(n).
0.64. Mostre que todo n umero racional positivo pode ser escrito de ma-
neira unica na forma
a
1
1!
+
a
2
2!
+ +
a
k
k!
onde:
0 a
1
, 0 a
2
< 2, 0 a
3
< 3, . . . , 0 < a
k
< k.
0.65 (OBM1991). Demonstrar que existem innitos m ultiplos de 1991
que sao da forma 19999 . . . 99991.
52 CAP

ITULO 0. FUNDAMENTOS
0.66 (IMO1983).

E possvel escolher 1983 inteiros positivos distintos,
todos menores que 10
5
, tal que n ao existam tres que sejam termos con-
secutivos de uma progressao aritmetica?
Dica: Usar base 3.
0.67. Seja S(n) a soma dos dgitos de n. Encontrar S(S(S(2
2
5
+ 1))).
0.68 (Chi2003). Encontrar todas as ternas (d, m, n) de inteiros positivos
tais que d
m
+ 1 divide d
n
+ 203.
0.69. Seja p > 2 um n umero primo. Demonstrar que
__
p 1
2
_
!
_
2
(1)
(p+1)/2
(mod p).
0.70 (AusPol1996). Mostrar que n ao existem inteiros n ao negativos m, n
tais que m! + 48 = 48(m+ 1)n
0.71. Seja p um n umero primo. Demonstrar que (p 1)! + 1 e uma
potencia de p se, e so se, p = 2, 3 ou 5.
0.72. Demonstrar que para todo n umero primo p > 3, o n umero
_
np
p
_
n
e divisvel por p
3+r
onde p
r
e a maior potencia de p que divide n.
0.73. Demonstrar que

1kn
mdc(n,k)=1
k =
n(n)
2
.
0.74. Demonstrar que se mdc(a, b) = 1, ent ao todos os divisores primos
de a
2
+b
2
sao da forma 4k + 1.
Dica: Utilize o teorema de Euler-Fermat.
0.75. Demonstrar que existem innitos primos da forma 4k + 1.
0.76. Sejam m, n inteiros positivos. Demonstrar que 4mnmn nunca
pode ser o quadrado de um n umero inteiro.
0.77 (IMO1985). Seja d um n umero positivo distinto de 2, 5 e 13. De-
monstrar que e possvel encontrar dois n umeros diferentes a e b que
perten cam ao conjunto {2, 5, 13, d} tais que ab 1 n ao e um quadrado
perfeito.
0.78. Demonstrar que se p | (a
p
b
p
), ent ao p
2
| (a
p
b
p
).
0.79 (IMO1984). Encontre todos os pares de inteiros positivos a, b tais
que ab(a +b) n ao e divisvel por 7, mas (a +b)
7
a
7
b
7
e divisvel por
7
7
.
(a +b)
7
a
7
b
7
= 7ab(a +b)(a
2
+ab +b
2
)
2
.
0.10. EQUA C

OES LINEARES M

ODULO M 53
0.80 (OIbM2001). Demonstrar que para cada inteiro positivo n existe
um inteiro m tal que 2
m
tem no mnimo
2
3
n1 zeros entre seus ultimos
n algarismos em notacao base 10.
0.81 (IMO2003). Seja p um n umero primo mpar. Demonstre que existe
um primo q tal que para todo n, o n umero n
p
p n ao e divisvel por q.
0.82 (IMO1979). Sejam m e n inteiros positivos tais que
m
n
= 1
1
2
+
1
3

1
4
+
1
1318
+
1
1319
.
Mostrar que m e divisvel por 1979.
0.83. Seja p um n umero primo mpar e sejam a e b inteiros n ao divis-
veis por p tais que p | ab. Mostrar que p
k
| a
n
b
n
p
k
| n(ab).
0.84. Sem usar computador (mas podendo usar calculadora) e sabendo
que os fatores de n estao perto um do outro, use o metodo de Fermat
para determine os fatores de
(a) n = 62236177.
(b) n = 6218583803.
0.85. Fatorar (sem usar computador) 801621073 sabendo que tem tres
fatores primos, um muito pequeno e os outros dois muito pr oximos.
0.86. Encontre os fatores de 521827 sabendo que e produto de dois pri-
mos e (521827) = 520056.
0.87. Sabendo que a chave p ublica de criptograa RSA sao os n umeros
N = 26549 e s = 101, determine a chave privada.
0.10 Equacoes Lineares M odulo m
Se mdc(a, m) = 1, como a e invertvel m odulo m, a equa cao
ax b (mod m),
tem solucao unica m odulo m, dada por x a
(m)1
b (mod m) (uti-
lizando o teorema de Euler-Fermat para encontrar o inverso de a
Z/(m)). Assim, todas as solucoes da equa cao acima s ao da forma
x = a
(m)1
b + km onde k Z. No caso geral, se mdc(a, m) = d > 1
temos que
ax b (mod m) = ax b (mod d) b 0 (mod d).
54 CAP

ITULO 0. FUNDAMENTOS
Logo uma condi cao necessaria para que a congruencia linear ax b
(mod m) tenha solucao e que d | b. Esta condi cao e tambem suciente,
ja que escrevendo a = da

, b = db

e m = dm

, temos que
ax b (mod m) a

x b

(mod m

).
Como mdc(a

, m

) = 1, ha uma unica solucao (a

)
(m

)1
b

m odulo m

,
isto e, ha d solucoes distintas m odulo m, a saber x (a

)
(m

)1
b

+
km

(mod m) com 0 k < d. Note ainda que como resolver ax b


(mod m) e equivalente a resolver a equa cao diofantina linear ax+my =
b, poderamos tambem ter utilizado o teorema de Bachet-Bezout e o
algoritmo de Euclides para encontrar as solucoes desta congruencia linear
como no exemplo 0.27. Resumimos esta discuss ao na seguinte
Proposi cao 0.63. A congruencia linear
ax b (mod m)
admite solu cao se, e somente se, mdc(a, m) | b. Neste caso, ha exata-
mente mdc(a, m) solu coes distintas modulo m.
Agora queremos encontrar condi coes para que um sistema de con-
gruencias lineares tenha solucao. O seguinte teorema nos garante a exis-
tencia de tais solucoes.
Teorema 0.64 (Teorema Chines dos Restos). Se b
1
, b
2
, . . . , b
k
sao intei-
ros quaisquer e a
1
, a
2
, . . . , a
k
sao primos relativos dois a dois, o sistema
de equacoes
x b
1
(mod a
1
)
x b
2
(mod a
2
)
.
.
.
x b
k
(mod a
k
)
admite solu cao, que e unica modulo A = a
1
a
2
. . . a
k
.
Demonstra c

ao: Daremos duas provas do teorema chines dos res-


tos. Para a primeira, consideremos os n umeros M
i
=
A
a
i
. Temos que
mdc(a
i
, M
i
) = 1, logo existe X
i
tal que M
i
X
i
1 (mod a
i
). Note que
se j = i entao M
j
e m ultiplo de a
i
e portanto M
j
X
j
0 (mod a
i
).
Assim, temos que
x
0
= M
1
X
1
b
1
+M
2
X
2
b
2
+ +M
k
X
k
b
k
0.10. EQUA C

OES LINEARES M

ODULO M 55
e solucao do sistema de equa coes, pois x
0
M
i
X
i
b
i
b
i
(mod a
i
).
Alem disso, se x
1
e outra solucao, entao x
0
x
1
(mod a
i
) a
i
|
x
0
x
1
para todo a
i
, e como os a
i
s s ao dois a dois primos, temos que
A | x
0
x
1
x
0
x
1
(mod A), mostrando a unicidade m odulo A.
Para a segunda prova, considere o mapa natural
f : Z/(A) Z/(a
1
) Z/(a
2
) Z/(a
k
)
b mod A (b mod a
1
, b mod a
2
, . . . , b mod a
k
).
Note que este mapa esta bem denido, isto e, o valor de f(b mod A)
independe da escolha do representante da classe de b mod A, pois quais-
quer dois representantes diferem de um m ultiplo de A, que tem ima-
gem (0 mod a
1
, . . . , 0 mod a
k
) no produto Z/(a
1
) Z/(a
k
). Ob-
servemos agora que o teorema chines dos restos e equivalente a mos-
trar que f e uma bijecao: o fato de f ser sobrejetor corresponde `a
existencia da solucao do sistema, enquanto que o fato de f ser injetor
corresponde `a unicidade m odulo A. Como o domnio e o contradom-
nio de f tem mesmo tamanho (ambos tem A elementos), para mostrar
que f e uma bijecao basta mostrarmos que f e injetora. Suponha que
f(b
1
mod A) = f(b
2
mod A), entao b
1
b
2
(mod a
i
) para todo i, e como
na primeira demonstra cao temos que isto implica b
1
b
2
(mod A), o
que encerra a prova.
Observa cao 0.65. Como mdc(b, a
1
a
2
...a
k
) = 1 mdc(b, a
j
) =
1, j k, a bijecao f denida na segunda prova do teorema anterior
satisfaz f((Z/(A))

) = (Z/(a
1
))

(Z/(a
2
))

(Z/(a
k
))

.
Em particular, isso nos da uma nova prova de que
(a
1
a
2
...a
k
) = (a
1
)(a
2
)...(a
k
) sempre que mdc(a
i
, a
j
) = 1, i = j.
Por exemplo, para k = 2, a
1
= 3 e a
2
= 5, temos a seguinte tabela,
que mostra, para cada i e j com 0 i < 3 e 0 j < 5, a unica solucao
x com 0 x < 3 5 = 15 tal que x i (mod 3) e x j (mod 5):
0 mod 5 1 mod 5 2 mod 5 3 mod 5 4 mod 5
0 mod 3 0 6 12 3 9
1 mod 3 10 1 7 13 4
2 mod 3 5 11 2 8 14
Vejamos algumas aplicacoes.
Exemplo 0.66. Determine as solu coes de x
130
1 (mod 85).
Solu c

ao: Como x
(85)
= x
64
1 (mod 85), temos que
1 x
130
= (x
64
)
2
x
2
x
2
(mod 85).
56 CAP

ITULO 0. FUNDAMENTOS
Assim temos que encontrar as solucoes do sistema
_
x
2
1 4 (mod 5)
x
2
1 16 (mod 17)
.
O que e equivalente a encontrar x tal que
x 2 (mod 5) e x 4 (mod 17).
Assim, obtemos 4 sistemas, dependendo da escolha dos sinais:
(1)
_
x 2 (mod 5)
x 4 (mod 17)
(2)
_
x 2 (mod 5)
x 4 (mod 17)
(3)
_
x 2 (mod 5)
x 4 (mod 17)
(4)
_
x 2 (mod 5)
x 4 (mod 17)
Observe que se a e a solucao de (1) entao a e solucao de (3), e de igual
forma se a e a solucao de (2) entao a e solucao de (4). Assim, basta
solucionar (3) e (4).
Como x = 13 e solucao de (3), entao 85 13 = 72 e solucao de (1).
Como x = 47 e solucao de (4), entao 85 47 = 38 e solucao de (2).
Portanto, as solucoes da equa cao original s ao 13, 38, 47 e 72 (mod 85).
Exemplo 0.67. Um inteiro e livre de quadrados se ele n ao e divisvel
pelo quadrado de nenhum n umero inteiro maior do que 1. Demonstrar
que existem intervalos arbitrariamente grandes de inteiros consecutivos,
nenhum dos quais e livre de quadrados.
Solu c

ao: Seja n um n umero natural qualquer. Sejam p


1
, . . . , p
n
pri-
mos distintos. O teorema chines dos restos nos garante que o sistema
x 1 (mod p
2
1
)
x 2 (mod p
2
2
)
.
.
.
x n (mod p
2
n
)
tem solucao. Se x
0
e uma solucao positiva do sistema, entao cada um
dos n umeros x
0
+ 1, x
0
+ 2, . . . , x
0
+ n e divisvel pelo quadrado de um
inteiro maior do que 1, logo nenhum deles e livre de quadrados.
0.10. EQUA C

OES LINEARES M

ODULO M 57
Exemplo 0.68. Seja P(x) um polin omio n ao constante com coecientes
inteiros. Demonstrar que para todo inteiro n, existe um inteiro i tal que
P(i), P(i + 1), P(i + 2), . . . , P(i +n)
sao n umeros compostos.
Solu c

ao: Demonstraremos primeiro o seguinte


Lema 0.69. Seja P(x) um polin omio n ao constante com coecientes
inteiros. Para todo par de inteiros k, i, tem-se que P(i) | P(k P(i) +i).
Demonstra c

ao: Dado que (kP(i) +i)


n
i
n
(mod P(i)) para todo n
inteiro nao negativo, e facil ver que P(kP(i)+i) P(i) 0 (mod P(i)).
Suponhamos por contradi cao que a sequencia P(i), P(i+1), . . . , P(i+
n) contem um n umero primo para cada i. Entao a sequencia {P(i)}
i1
assume innitos valores primos. Consideremos os n+1 primos distintos
P(i
0
), P(i
1
), . . . , P(i
n
). Pelo teorema chines dos restos segue que existem
innitas solucoes x do sistema de equa coes
x i
0
(mod P(i
0
))
x i
1
1 (mod P(i
1
))
x i
2
2 (mod P(i
2
))
.
.
.
x i
n
n (mod P(i
n
))
onde, se x
0
e uma solucao, entao x = x
0
+ k(P(i
0
) P(i
n
)) tambem e
solucao para todo k 0. Assim, pelo lema anterior, podemos dizer que
P(x), P(x + 1), . . . , P(x +n) s ao n umeros compostos quando k e suci-
entemente grande, m ultiplos respectivamente de P(i
0
), P(i
1
), . . . , P(i
n
).
Exemplo 0.70. Uma potencia nao trivial e um n umero da forma m
k
,
onde m, k sao inteiros maiores do que ou iguais a 2. Dado n N, prove
que existe um conjunto A N com n elementos tal que para todo sub-
conjunto B A n ao vazio,

xB
x e uma potencia n ao trivial. Em outras
palavras, se A = {x
1
, x
2
, . . . , x
n
} ent ao todas as somas x
1
, x
2
, . . . , x
n
,
x
1
+x
2
, x
1
+x
3
, . . . , x
n1
+x
n
, . . . , x
1
+x
2
+ +x
n
sao potencias n ao
triviais.
58 CAP

ITULO 0. FUNDAMENTOS
Solu c

ao: Vamos provar a existencia de um tal conjunto por indu cao


em n. Para n = 1, A = {4} e solucao e, para n = 2, A = {9, 16} e
solucao. Suponha agora que A = {x
1
, . . . , x
n
} e um conjunto com n
elementos e para todo B A, B = ,

xB
x = m
k
B
B
. Vamos mostrar
que existe c N tal que o conjunto

A = {cx
1
, cx
2
, . . . , cx
n
, c} satisfaz
o enunciado. Seja = mmc{k
B
| B A, B = }, o mnimo m ultiplo
comum de todos os expoentes k
B
. Para cada B A, B = , associamos
um n umero primo p
B
> , de forma que B
1
= B
2
implica p
B
1
= p
B
2
.
Pelo teorema chines dos restos existe um natural r
B
com
r
B
0 (mod p
X
) para todo subconjunto X A, X = B
r
B
1 (mod p
B
).
( e invertvel m odulo p
B
). Tomemos
c =

XA
X=
(1 +m
k
X
X
)
r
X
e vamos mostrar que

A = {cx
1
, cx
2
, . . . , cx
n
, c} continua a satisfazer as
condi coes do enunciado.
Dado B

{cx
1
, cx
2
, . . . , cx
n
}, temos que B

= {cx | x B} para
algum B A. Como c e uma potencia -esima, c tambem e uma
potencia k
B
-esima, portanto,

xB

x = cm
k
B
B
sera uma potencia k
B
-
esima para todo B

= . Alem disso, para subconjuntos de



A da forma
B

{c}, temos

xB

{c}
x = c (1 +m
k
B
B
) =
_

XA
X=,B
(1 +m
k
X
X
)
r
X
_
(1 +m
k
B
B
)
r
B
+1
,
que e uma potencia p
B
-esima, pois r
B
+1 e r
X
(X = B) s ao m ultiplos
de p
B
.
Problemas Propostos
0.88. Determine a menor solu cao inteira positiva do sistema
_

_
x 3 (mod 7)
x 5 (mod 9)
x 2 (mod 8).
0.10. EQUA C

OES LINEARES M

ODULO M 59
0.89. Determine todas as solu coes do sistema
_

_
x 1 (mod 6)
x 7 (mod 10)
x 4 (mod 33).
Observe que o sistema n ao satisfaz as condicoes do teorema Chines dos
restos.
0.90. Determine todas as solu coes de x
2
+x + 18 0 (mod 42).
0.91. Quantos elementos tem (Z/(210))

? Quantos deles tem ordem


24?
0.92. Resolver as equacoes lineares
(a) 7x 12 (mod 127)
(b) 12x 5 (mod 122)
(c) 40x 64 (mod 256)
0.93. Resolver o sistema de congruencias lineares
x 0 (mod 7)
x 1 (mod 12)
x 5 (mod 17)
0.94. Um inteiro positivo n e chamado de auto-replicante se os ultimos
dgitos de n
2
formam o n umero n. Por exemplo, 25 e auto-replicante
pois 25
2
= 625. Determine todos os n umeros auto-replicantes com exa-
tamente 4 dgitos.
0.95. Sejam a, n N
>0
e considere a sequencia (x
k
) denida por x
1
= a,
x
k+1
= a
x
k
para todo k N. Demonstrar que existe N N tal que
x
k+1
x
k
(mod n) para todo k N.
0.96. Demonstrar que o sistema de equacoes
x b
1
(mod a
1
)
x b
2
(mod a
2
)
.
.
.
x b
k
(mod a
k
)
tem solu cao se, e so se, para todo i e j, mdc(a
i
, a
j
) | (b
i
b
j
). (No
caso particular em que mdc(a
i
, a
j
) = 1, o problema se reduz ao teorema
chines dos restos).
60 CAP

ITULO 0. FUNDAMENTOS
0.97. Demonstrar que, para k e n n umeros naturais, e possvel encontrar
k n umeros consecutivos, cada um dos quais tem ao menos n divisores
primos diferentes.
0.98. Demonstrar que se a, b e c sao tres inteiros diferentes, ent ao
existem innitos valores de n para os quais a + n, b + n e c + n sao
primos relativos dois a dois.
0.99. Demonstrar que para todo inteiro positivo m e todo n umero par 2k,
este ultimo pode ser escrito como a diferen ca de dois inteiros positivos,
cada um dos quais e primo relativo com m.
0.100. Demonstrar que existem progressoes aritmeticas de comprimento
arbitr ario formadas por inteiros positivos tais que cada termo e a poten-
cia de um inteiro positivo com expoente maior do que 1.
Captulo 1
Potencias e Congruencias
Neste captulo continuamos a discutir congruencias, com especial
interesse por potencias de um inteiro a m odulo p, p primo.
Inicialmente faremos uma revisao de alguns fatos basicos sobre o
anel dos polin omios com coecientes sobre um corpo qualquer K; o leitor
reconhecera que varios resultados sobre polin omios com coecientes reais
ainda valem quando os coecientes estao, digamos, em Z/(p).
Depois disso discutiremos a existencia de razes primitivas m odulo
n.
Finalmente, discutiremos se um inteiro a e ou nao um quadrado
m odulo p. Para isso apresentaremos o smbolo de Legendre e suas prin-
cipais propriedades. O captulo conclui com o Teorema da Reciprocidade
Quadratica, para o qual daremos tres demonstra coes explorando ideias
diferentes.
1.1 Polinomios
Dado um anel comutativo K, denimos o anel comutativo K[x] como
sendo o conjunto das expressoes da forma f(x) = a
0
+a
1
x+a
2
x
2
+ +
a
n
x
n
com a
i
K, chamados de polin omios com coecientes em K.
A soma e o produto em K[x] s ao denidos da maneira usual: dados
f(x) =

i
a
i
x
i
e g(x) =

i
b
i
x
i
elementos de K[x] temos
f(x) +g(x)
def
=

i
(a
i
+b
i
)x
i
;
f(x) g(x)
def
=

k
c
k
x
k
onde c
k
=

i+j=k
a
i
b
j
.
Denimos o grau deg f(x) de um polin omio f(x) = a
0
+ a
1
x + a
2
x
2
+
+ a
n
x
n
como sendo o maior i tal que a
i
= 0; o grau do polin omio
61
62 CAP

ITULO 1. POT

ENCIAS E CONGRU

ENCIAS
nulo 0 e denido como sendo . Tal conven cao visa a tornar validas
as seguintes identidades para todos os polin omios f(x), g(x) K[x]:
deg
_
f(x) g(x)
_
= deg f(x) + deg g(x) e
deg
_
f(x) +g(x)
_
max{deg f(x), deg g(x)}.
O coeciente do termo de maior grau de um polin omio e chamado de co-
eciente lder. Um polin omio cujo coeciente lder e igual a 1 e chamado
de monico.
Observe que nas denicoes acima x e um smbolo formal e nao um
elemento de K. Apesar disso, cada polin omio f(x) = a
0
+a
1
x +a
2
x
2
+
+a
n
x
n
dene uma fun cao polinomial
f : K K
c f(c) = a
0
+a
1
c +a
2
c
2
+ +a
n
c
n
tambem chamada de f. A distincao entre um polin omio e uma funcao
polinomial e bem ilustrada pelo polin omio f(x) = x
p
x (Z/(p))[x]:
este polin omio e nao nulo pois seus coecientes s ao nao nulos, mas para
todo c Z/(p) temos f(c) = 0 pelo pequeno teorema de Fermat. Dado
um polin omio f(x) K[x], qualquer c K tal que f(c) = 0 e chamado
de raiz ou zero de f(x).
Como veremos nesta secao, polin omios guardam muitas semelhan cas
com n umeros inteiros. Por exemplo, podemos denir divisibilidade de
polin omios de maneira completamente an aloga: d(x) | f(x) em K[x] se,
e s o se, existe g(x) K[x] tal que f(x) = d(x) g(x). Temos tambem
uma generaliza cao da divisao euclidiana:
Proposi cao 1.1 (Algoritmo da divisao). Seja K um corpo. Dados po-
linomios f(x), g(x) K[x], com g(x) = 0, existem q(x), r(x) K[x]
(chamados respectivamente de quociente e resto da divisao de f(x) por
g(x)), unicamente determinados, tais que
f(x) = q(x) g(x) +r(x) com deg r(x) < deg g(x).
Demonstra c

ao: Sejam n = deg f(x) e m = deg g(x). Para demons-


trar a existencia de q(x) e r(x), procederemos por indu cao sobre n. Note
que se m > n, entao basta tomar q(x) = 0 e r(x) = f(x), logo podemos
supor que m n. Se n = m = 0, entao f(x) = a e g(x) = b s ao ambos
constantes nao nulas, logo basta tomar q(x) = a/b e r(x) = 0 neste caso.
Agora suponha que n 1. Escreva f(x) = a
n
x
n
+ f
1
(x) e g(x) =
b
m
x
m
+ g
1
(x) com a
n
= 0, b
m
= 0 e deg f
1
(x) < n, deg g
1
(x) < m.
Observemos que o polin omio f(x)
a
n
b
m
x
nm
g(x) = f
1
(x)
a
n
b
m
x
nm
g
1
(x)
1.1. POLIN

OMIOS 63
e de grau menor que n. Por hip otese de indu cao existem dois polin omios
q(x) e r(x) tais que
f(x)
a
n
b
m
x
nm
g(x) = q(x)g(x) +r(x) com deg r(x) < deg g(x).
Logo podemos escrever f(x) = (
a
n
b
m
x
nm
+q(x)) g(x) +r(x), que era o
que se queria demonstrar.
Para demonstrar que os polin omios q(x) e r(x) s ao unicos, suponha
que
f(x) = q
1
(x)g(x) +r
1
(x) = q
2
(x)g(x) +r
2
(x)
com q
1
(x) = q
2
(x) e deg r
1
(x), deg r
2
(x) < deg g(x). Entao r
2
(x)
r
1
(x) = (q
1
(x) q
2
(x))g(x) = 0 e um m ultiplo de g(x) de grau estrita-
mente menor do que deg g(x), o que e um absurdo.
Corolario 1.2. Seja K um corpo, f(x) K[x] e a K. Ent ao
x a | f(x) f(a) = 0.
Demonstra c

ao: Como deg(xa) = 1, dividindo f(x) por xa temos


que f(x) = (x a)q(x) + r com r K. Assim, substituindo x por a,
temos que f(a) = r donde o resultado segue.
Exemplo 1.3. Seja f(x) Q[x] um polin omio com coecientes racio-
nais tal que f(1) = 3 e f(3) = 5. Determine o resto da divisao do
polin omio f(x) pelo polin omio g(x) = x
2
4x + 3.
Solu c

ao: Seja q(x) e r(x) respectivamente o quociente o resto da


divisao de f(x) por g(x), isto e, f(x) = q(x)g(x) +r(x), onde r(x) e um
polin omio de grau menor ou igual a 1, assim r(x) = ax+b. Observe que
g(x) = (x 1)(x 3), assim g(1) = g(3) = 0, e portanto
3 = f(1) = q(1)g(1) +r(1) = a +b
5 = f(3) = q(3)g(3) +r(3) = 3a +b.
Segue que a = 4 e b = 7 e portanto r(x) = 4x + 7.
Proposi cao 1.4. Seja K um corpo. Um polin omio f(x) K[x] n ao
nulo de grau n tem no maximo n razes em K.
64 CAP

ITULO 1. POT

ENCIAS E CONGRU

ENCIAS
Demonstra c

ao: A demonstra cao e feita por indu cao emn = deg f(x);
os casos n = 0 e n = 1 s ao triviais. Se f(x) tivesse n + 1 razes distin-
tas a
1
, . . . , a
n+1
, entao f(x) = (x a
n+1
)g(x) para algum g(x) K[x]
pelo corolario anterior. Assim, para i = n + 1, teramos 0 = f(a
i
) =
(a
i
a
n+1
)g(a
i
) = g(a
i
) = 0 pois (a
i
a
n+1
) = 0 e invertvel em K.
Logo g(x), de grau n 1, teria n razes distintas a
1
, . . . , a
n
, contradi-
zendo a hip otese de indu cao.
Note que a proposicao anterior e falsa se K nao e um corpo. Por
exemplo, o polin omio f(x) = x
2
1 Z/8Z[x] tem 4 razes em Z/8Z, a
saber 1, 3, 5, 7.
A seguinte proposicao ajuda a achar as razes de um polin omio em
Q[x].
Proposi cao 1.5. Seja f(x) = a
n
x
n
+ + a
0
Z[x] um polin omio de
grau n. Mostre que se p/q e uma raiz racional de f(x), com p, q Z e
mdc(p, q) = 1, ent ao p | a
0
e q | a
n
.
Demonstra c

ao: Multiplicando f(p/q) = 0 por q


n
, segue que
a
n
p
n
+a
n1
p
n1
q + +a
0
q
n
= 0,
donde p | a
0
q
n
e q | a
n
p
n
. Como mdc(p, q) = 1, conclumos que p | a
0
e
q | a
n
.
Exemplo 1.6. Seja f(x) Q[x] um polin omio de grau 5 tal que f(k) =
2
k
para k = 0, 1, . . . , 5. Determine o valor de f(6).
Solu c

ao: Suponhamos que f(x) = a


5
x
5
+ a
4
x
4
+ + a
0
, onde os
a
i
s ao os coecientes do polin omio. Assim se substitumos por 0, 1, 5
obteramos um sistema de 6 equa coes lineares com 6 incognitas, que pode
ser solucionado com algum processo de resolu cao de sistemas lineares.
Dado que nosso objetivo nao e calcular o polin omio e sim o valor de
f(6), empregaremos um metodo distinto. De fato vamos resolver um
problema um pouco mas geral. Suponhamos que f e um polin omio de
grau n tal que f(j) = 2
j
para j = 0, 1, . . . n e queremos calcular f(n+1).
Denamos os polin omios auxiliares P
j
(x) de grau n tal que para cada
i = 0, 1, . . . n se tem que P
j
(i) =
_
1 se i = j
0 se i = j
.

E facil vericar por
diretamente que f(x) =
n

j=0
2
j
P
j
(x). Agora cada um dos polin omios e
1.1. POLIN

OMIOS 65
facilmente calcul avel, ja que s ao polin omios de grau n e conhecemos n
razes; de fato, se S
j
= {0, 1, . . . , n} \ {j}, entao
P
j
(x) =

iS
j
x i
j i
=

iS
j
(x i)
(1)
nj
j!(n j)!
.
Em particular,
P
j
(n + 1) =

iS
j
(n + 1 i)
(1)
nj
j!(n j)!
=
(1)
nj
(n + 1)!
j!(n + 1 j)!
= (1)
nj
_
n + 1
j
_
.
Conclumos que
f(n + 1) =
n

j=0
(1)
nj
_
n + 1
j
_
2
j
= 2
n+1

n+1

j=0
(1)
n+1j
_
n + 1
j
_
2
j
= 2
n+1
(2 1)
n+1
= 2
n+1
1.
Vejamos uma aplicacao dos resultados anteriores quando K = Z/(p),
p primo. A primeira e uma nova demonstra cao do teorema de Wilson:
Teorema 1.7. Seja p um primo. Considere a fun cao simetrica elemen-
tar
i
em 1, 2, . . . , p 1 dada pela soma de todos os
_
p1
i
_
produtos de i
termos distintos dentre 1, 2, . . . , p 1:

1
= 1 + 2 + + (p 1)

2
= 1 2 + 1 3 + + (p 2)(p 1)
.
.
.

p1
= 1 2 . . . (p 1).
Ent ao
1
, . . . ,
p2
sao todos m ultiplos de p e
p1
= (p 1)! 1
(mod p) (teorema de Wilson).
Demonstra c

ao: Pelo teorema de Fermat e pela proposicao anterior,


temos que 1, 2, . . . , p 1 s ao todas as razes de x
p1
1 em Z/(p). Logo
aplicando o corolario e comparando coecientes lderes obtemos a fato-
ra cao
x
p1
1 = (x 1)(x 2) . . . (x p 1).
Mas o polin omio do lado direito e igual a x
p1

1
x
p2
+
2
x
p3
+
(1)
p1

p1
. Comparando coecientes, obtemos o resultado.
66 CAP

ITULO 1. POT

ENCIAS E CONGRU

ENCIAS
Seja K um corpo. Podemos considerar tambem congruencias de
polin omios em K[x]: se a(x), b(x), m(x) K[x], escrevemos
a(x) b(x) (mod m(x)) m(x) | a(x) b(x).
As mesmas demonstra coes do caso inteiro mostram que as congruencias
m odulo m(x) denem uma rela cao de equivalencia em K[x] compatvel
com as operacoes de soma, subtracao e produto. Assim, podemos formar
o anel quociente
K[x]
_
m(x)
_
cujos elementos s ao os conjuntos da forma
a(x)
def
= {b(x) K[x] | b(x) a(x) (mod m(x))}
e as operacoes no anel quociente s ao dadas por
f(x) +g(x)
def
= f(x) +g(x) e f(x) g(x)
def
= f(x) g(x)
sendo independentes das escolhas dos representantes de classe f(x) e
g(x). Se deg m(x) = n, um sistema completo de resduos m odulo m(x)
e dado pelos polin omios de grau menor do que n (os possveis restos na
divisao euclidiana por m(x)):
{a
0
+a
1
x + +a
n
x
n1
| a
i
K}
Em particular,
K[x]
(m(x))
e innito se K tambem o e.
Exemplo 1.8. Determine o resto da divisao de (x+1)
2010
por x
2
+x+1
em Q[x].
Solu c

ao: Multiplicando por x 1 a congruencia x


2
+ x + 1 0
(mod x
2
+x + 1), obtemos x
3
1 (mod x
2
+x + 1). Assim, temos
(x + 1)
2
x (mod x
2
+x + 1)
= (x + 1)
2010
x
1005
= (x
3
)
335
(mod x
2
+x + 1)
= (x + 1)
2010
1 (mod x
2
+x + 1)
Assim, o resto da divisao e 1.
Podemos tentar denir o mdc d(x) de dois polin omios f(x) e g(x)
(com f(x) = 0 ou g(x) = 0) de maneira an aloga ao mdc de inteiros,
tomando o polin omio d(x) de maior grau que divide f(x) e g(x) simul-
taneamente. Entretanto, d(x) nao esta bem determinado, pois qualquer
m ultiplo c d(x) com c = 0 constante ainda satisfaz as condi coes acima.
1.1. POLIN

OMIOS 67
Para evitar esta ambiguidade, denimos o mdc de f(x) e g(x) como
sendo o polin omio monico de maior grau que divide f(x) e g(x) simulta-
neamente. Analogamente, dene-se o mmc de f(x) e g(x) (com f(x) = 0
e g(x) = 0) como o polin omio m onico de menor grau que e divisvel tanto
por f(x) como por g(x).
A divisao euclidiana permite estender resultados de Z para K[x] de
maneira quase trivial. Por exemplo, temos
Teorema 1.9 (Bachet-Bezout). Seja d(x) o maximo divisor comum de
dois polin omios f(x) e g(x). Ent ao existem dois polin omios m(x) e n(x)
tais que f(x)m(x) +g(x)n(x) = d(x).
Demonstra c

ao: An aloga ao teorema 0.20; como naquele teorema


d(x) sera o polin omio m onico de menor grau no conjunto
I(f, g)
def
= {f(x)m(x) +g(x)n(x) | m(x), n(x) K[x]}.
Exemplo 1.10. Sejam f(x) = x
6
+ x + 1 e g(x) = x
4
+ 1 polin omios
em (Z/(2))[x]. Determine dois polin omios m(x), n(x) (Z/(2))[x] tal
que f(x)m(x) +g(x)n(x) = mdc(f(x), g(x)).
Solu c

ao: Observe que em Z/(2) temos que g(x) = (x 1)


4
, isto e,
g(x) tem uma raiz quadrupla em x = 1. Por outro lado f(1) = 1,
logo mdc(f, g) = 1. Para calcular m(x) e n(x) basta usar o algoritmo
estendido da divisao:
f(x) = x
2
g(x) +x
2
+x + 1
g(x) = (x
2
+x)(x
2
+x + 1) +x + 1
x
2
+x + 1 = x(x + 1) + 1.
Segue que
1 = x
2
+x + 1 +x x + 1
= x
2
+x + 1 +x
_
g(x) + (x
2
+x) x
2
+x + 1
_
= (x
3
+x
2
+ 1) x
2
+x + 1 +xg(x)
= (x
3
+x
2
+ 1)(f(x) +x
2
g(x)) +xg(x)
= (x
3
+x
2
+ 1)f(x) + (x
5
+x
4
+x
2
+x)g(x).
68 CAP

ITULO 1. POT

ENCIAS E CONGRU

ENCIAS
Denicao 1.11. Seja K um corpo. Dizemos que um polin omio n ao
constante f(x) K[x] e irredutvel em K[x] se f(x) n ao e o produto de
dois polin omios em K[x] de graus estritamente menores do que deg f(x).
Polin omios irredutveis fazem o papel de n umeros primos para po-
linomios. Por exemplo, x
2
+ 1 R[x] e irredutvel em R[x], pois caso
contrario ele poderia ser escrito como produto de polin omios de grau 1
em R[x], contradizendo o fato de x
2
+ 1 = 0 nao possuir razes reais.
Por outro lado, x
2
+1 e redutvel em C[x] ja que x
2
+1 = (xi)(x+i).
Isto mostra que irredutibilidade e um conceito que depende do anel de
polin omios sobre o qual estamos trabalhando.
Os exemplos mais evidentes de polin omios irredutveis em K[x] s ao
os lineares m onicos, i.e., os da forma x a, a K. Quando estes
s ao os unicos polin omios irredutveis em K[x] dizemos que o corpo K e
algebricamente fechado. Observe que em geral polin omios de graus 2 ou
3 s ao irredutveis em K[x] se, e somente se, nao tem razes em K.
A partir do teorema de Bachet-Bezout, como no caso dos inteiros,
obtemos (c.f. proposicao 0.23 e teorema 0.30):
Proposi cao 1.12. Seja K um corpo e sejam p(x), a
1
(x), . . . a
m
(x)
K[x] com p(x) irredutvel em K[x]. Se p(x) | a
1
(x) . . . a
m
(x), ent ao
p(x) | a
i
(x) para algum i.
Teorema 1.13 (Fatoracao

Unica). Seja K um corpo. Todo polin omio
n ao nulo em K[x] pode ser fatorado como um produto de polin omios ir-
redutveis em K[x]; esta fatoracao e unica a menos da ordem dos fatores
e multiplicacao por constantes n ao nulas.
Outra importante consequencia do teorema de Bachet-Bezout e o
seguinte (c.f. teorema 0.51)
Teorema 1.14. Seja K um corpo e f(x) um polin omio irredutvel em
K[x]. Ent ao K[x]/(f(x)) e um corpo.
Demonstra c

ao: Assim como na demonstra cao de que Z/pZ e um


corpo para p primo, a diculdade aqui e mostrar que todo elemento
a(x) = 0 e invertvel em K[x]/(f(x)). Temos que mdc(a(x), f(x)) = 1
pois f(x) e irredutvel e f(x) nao divide a(x), caso contrario teramos
a(x) = 0. Logo, pelo teorema de Bachet-Bezout, existem r(x), s(x)
K[x] tais que
a(x)r(x) +f(x)s(x) = 1 = a(x)r(x) 1 (mod f(x))
Portanto r(x) e o inverso multiplicativo de a(x).
1.1. POLIN

OMIOS 69
Por exemplo, seja K = Z/(2) e f(x) = x
2
+x+1 K[x]. Temos que
f(x) e irredutvel pois ele tem grau 2 e nao possui razes em K. Assim,
K[x]/(f(x)) e um corpo, que possui 4 elementos. As tabelas de adicao
e multiplica cao deste corpo s ao as seguintes:
+ 0 1 x x + 1
0 0 1 x x + 1
1 1 0 x + 1 x
x x x + 1 0 1
x + 1 x + 1 x 1 0
0 1 x x + 1
0 0 0 0 0
1 0 1 x x + 1
x 0 x x + 1 1
x + 1 0 x + 1 1 x
Exemplo 1.15. Calcule ord
G
(x + 1) no grupo G = (Z/(7))[x]/(p(x))
onde p(x) = x
2
3x 1, isto e, o menor inteiro positivo k tal que
(x + 1)
k
1 em G.
Solu c

ao: Observemos que o grupo G tem 7


2
1 = 48 elementos (todo
elemento pode ser representado como ax+b, com a, b Z/(7), e o unico
elemento nao invertvel e o 0, pois p(x) e irredutvel em (Z/(7))[x]).
Assim, ord
G
(x+1) e um divisor de 48. Os divisores de 48 s ao 1, 2, 3, 4,
6, 8, 12, 16, 24 e 48. Assim temos que testar todas estas potencias
(x + 1)
2
x
2
+ 2x + 1 5x + 2 2x + 2
(x + 1)
3
(2x + 2)(x + 1) 2x
2
+ 2 6x x
(x + 1)
4
x(x + 1) x
2
+x 4x + 1
(x + 1)
6
(x + 1)
3
(x + 1)
3
x
2
3x + 1
(x + 1)
8
(x + 1)
4
(x + 1)
4
2x
2
+x + 1 3
(x + 1)
12
(x + 1)
4
(x + 1)
8
(4x + 1)(3) 2x + 3
(x + 1)
16
(x + 1)
82
2
(x + 1)
24
(x + 1)
83
1
Portanto ord
G
(x + 1) = 48.
Encerramos esta secao com um importante criterio de irredutibili-
dade para polin omios com coecientes inteiros. Primeiro, precisamos de
uma
70 CAP

ITULO 1. POT

ENCIAS E CONGRU

ENCIAS
Denicao 1.16. Um polin omio n ao nulo f(x) Z[x] e dito primitivo
se o mdc de seus coecientes e 1.
Lema 1.17. O produto de dois polin omios primitivos e primitivo.
Demonstra c

ao: Sejam g(x) e h(x) dois polin omios primitivos. Seja


p um primo e suponha por absurdo que p divida todos os coecientes de
g(x)h(x). Assim, em Z/pZ[x] teramos que g(x)h(x) = g(x)h(x) = 0,
onde a barra denota o polin omio obtido reduzindo-se seus coecientes
m odulo p. Por outro lado, g(x) = 0 e h(x) = 0, ja que por hip otese p
nao divide todos os coecientes de g(x) e o mesmo para h(x). Assim,
temos uma contradi cao pois Z/pZ[x] e um domnio, isto e, o produto de
dois polin omios nao nulos em Z/pZ[x] e diferente de zero (de fato, olhe
por exemplo para os coecientes lderes e use o fato de que Z/pZ e um
corpo).
O lema anterior e o passo essencial na prova do famoso lema de Gau,
que permite reduzir a quest ao da irredutibilidade de um polin omio em
Q[x] para a mesma quest ao em Z[x].
Teorema 1.18 (Lema de Gau). Seja f(x) Z[x] um polin omio primi-
tivo n ao constante. Ent ao f(x) e irredutvel em Q[x] se, e somente se,
f(x) e irredutvel em Z[x] (isto e, n ao podemos escrever f(x) = g(x)h(x)
com g(x), h(x) Z[x] n ao constantes).
Demonstra c

ao:

E claro que se f(x) e irredutvel sobre Q[x], entao
ele e irredutvel sobre Z[x]. Reciprocamente, suponha por contradi-
cao que f(x) seja irredutvel sobre Z[x] mas que f(x) = g(x)h(x) com
g(x), h(x) Q[x], ambos nao constantes. Multiplicando esta ultima
igualdade por um inteiro conveniente d > 0, podemos escrever
d f(x) = e g
0
(x)h
0
(x)
com g
0
(x), h
0
(x) Z[x] primitivos e e N. Como f(x) e g
0
(x)h
0
(x)
(pelo lema anterior) s ao primitivos, temos que d e o mdc dos coecientes
de df(x), enquanto que e e o mdc dos coecientes de eg
0
(x)h
0
(x). Logo
d = e e assim f(x) = g
0
(x)h
0
(x) e redutvel sobre Z[x], uma contradi cao.
Finalmente, para polin omios em Z[x], podemos aplicar o
Proposi cao 1.19 (Criterio de Eisenstein). Seja f(x) = a
n
x
n
+ +
a
1
x + a
0
Z[x] um polin omio primitivo n ao constante. Suponha que
exista um n umero primo p tal que p a
n
, p | a
j
para todo 0 j < n e
p
2
a
0
. Ent ao f(x) e irredutvel em Z[x].
1.1. POLIN

OMIOS 71
Demonstra c

ao: Suponha por absurdo que f(x) e redutvel, i.e., exis-


temg(x), h(x) Z[x] tais que f(x) = g(x)h(x) e 0 < deg g(x), deg h(x) <
n. Em Z/pZ[x], temos entao f(x) = g(x)h(x), onde a barra denota o
polin omio obtido reduzindo-se os seus coecientes m odulo p. Porem,
como p | a
j
para todo 0 j < n, temos que f(x) = a
n
x
n
e portanto,
pela fatora cao unica em Z/pZ[x] (teorema 1.13), devemos ter g(x) = bx
i
e h(x) = cx
j
com 0 < i, j < n, i + j = n e b c = a
n
. Mas isto signica
que os coecientes de x
0
em g(x) e h(x) s ao m ultiplos de p, e como
f(x) = g(x)h(x), que a
0
e m ultiplo de p
2
, absurdo.
Exemplo 1.20. Seja p um primo. Demonstrar que o polin omio f(x) =
x
p1
+x
p2
+ +x + 1 e irredutvel em Q[x].
Solu c

ao: Pelo lema de Gau, basta provar a irredutibilidade sobre


Z[x] e para isto utilizaremos o criterio de Eisenstein. Observemos que
f(x) =
x
p
1
x1
, logo
f(x + 1) =
(x + 1)
p
1
x
= x
p1
+
_
p
1
_
x
p2
+ +
_
p
p 1
_
e, com exce cao do coeciente lder, todos os coecientes deste polin omio
s ao m ultiplos de p, sendo que o termo independente
_
p
p1
_
= p nao e
m ultiplo de p
2
. Pelo criterio de Eisenstein, f(x + 1) e irredutvel em
Z[x] e, portanto, f(x) tambem o e.
Observa cao 1.21. Existem polin omios primitivos irredutveis f(x)
Z[x] mas que sao redutveis modulo p para todo primo p, por exemplo
f(x) = x
4
10x
2
+ 1 (veja o exemplo 1.45). Por outro lado, se f(x)
Z[x] admite raiz modulo p para todo primo p sucientemente grande,
ent ao f(x) possui raiz em Z! Veja o excelente artigo de Serre [14] para
uma demonstracao deste fato.
Problemas Propostos
1.1. Sejam f(x) = x
7
4x
5
5x
3
+x
2
+ 1 e g(x) = x
5
+ 2x
3
+ 6x
2
+ 5
polin omios em Q[x]. Determine mdc(f(x), g(x)) e ache dois polin omios
monicos com coecientes inteiros m(x) e n(x) tais que
f(x)m(x) +g(x)n(x) = mdc(f(x), g(x)).
1.2. Sejam f(x) = x
4
+x
2
+1 e g(x) = x
3
+1 polin omios em (Z/(2))[x].
Determine mdc(f(x), g(x)) e ache dois polin omios monicos (com coe-
cientes em Z/(2)) m(x) e n(x) tais que
f(x)m(x) +g(x)n(x) = mdc(f(x), g(x)).
72 CAP

ITULO 1. POT

ENCIAS E CONGRU

ENCIAS
1.3. Seja f(x) C[x] um polin omio que deixa restos 10 e 1 quando
dividido por x 1 e x 10 respectivamente. Encontrar o resto de f(x)
na divisao por (x 1)(x 10).
1.4. Determine o resto da divisao de f(x) = x
100
por g(x) = x
3
+2x
2

x 2.
Dica: Fatore g(x).
1.5. Determine o resto da divisao de f(x) = x
100
por g(x) = x
3
+2x
2
+
x + 2.
1.6. Mostre que o polin omio f(x) = x
4
4x
3
+6x
2
x +1 e irredutvel
em Q[x].
Dica: substituir x por x + a com a adequado e usar o criterio de
Einsentein.
1.7. Mostre que o polin omio f(x) = x
4
20x
2
+16 e irredutvel em Q[x].
1.8. Fatore o polin omio x
8
x (Z/(2))[x] em fatores irredutveis.
1.9. Fatore o polin omio x
25
x (Z/(5))[x] em fatores irredutveis.
1.10. Encontre um valor de a N tal que o polin omio f(x) = x
100
+
ax
98
+ 11 n ao tenha razes racionais mas n ao seja irredutvel em Q[x]
Dica: Calcule um a tal que f(x) seja divisvel por um polin omio
irredutvel de grau 2.
1.11. Seja uma raiz de X
3
3X +1 = 0. Mostre que
2
2 tambem
e uma raiz deste polin omio.
1.12. Seja K = (Z/(3))[x]/(f(x)) onde f(x) = x
2
+ x + 2. Mostre que
f(x) e irredutvel em (Z/(3))[x] e portanto K e um corpo. Construa a
tabela de multiplicacao do grupo K

e usando esta tabela determine o


menor inteiro positivo n tal que x
n
= 1 em K.
1.13. Seja R e n um inteiro positivo. Calcule o resto da divisao do
polin omio (cos +xsin )
n
R[x] por x
2
+ 1.
1.14 (IMO1993). Seja f(x) = x
n
+ 5x
n1
+ 3 onde n > 1. Demonstrar
que f(x) n ao pode se expressar como produto de dois polin omios n ao
constantes com coecientes inteiros.
1.15. Encontrar todos os pares (c, P(x)) onde c e um real e P(x) e um
polin omio n ao nulo tal que
P(x
4
+x
2
+x) = (x
6
+x
5
+x
4
+x
3
+x
2
+x + 1)P(cx).
1.16 (AusPol1998). Encontrar todos os inteiros positivos n e m tais que
todas as solu coes de x
3
17x
2
+mx n
2
= 0 sao inteiras.
1.2. ORDEM E RA

IZES PRIMITIVAS 73
1.17. Dados x, y N, dena a := x(y+1)(y!+1). Mostre que imagem
da fun cao f : N N N dada por
f(x, y) =
y 1
2
_
|a
2
1| (a
2
1)
_
+ 2
e exatamente o conjunto dos n umeros primos.
1.18. Prove a seguinte modicacao do Criterio de Eisenstein: seja f(x) =
a
n
x
n
+ + a
1
x + a
0
Z[x] um polin omio primitivo n ao constante e
sem razes racionais. Suponha que exista um n umero primo p tal que
p a
n
, p | a
j
para todo 0 j < n e p
2
a
1
. Ent ao f(x) e irredutvel em
Z[x].
1.19. (Zagier) Dado um n umero primo, associe a ele um polin omio cujos
coecientes sao os dgitos decimais desse primo (por exemplo, 9x
3
+4x
2
+
3 para o primo 9403). Mostre que este polin omio e sempre irredutvel
em Z[x].
1.20. Encontrar todos os valores de k para os quais o polin omio x
2k+1
+
x + 1 e divisvel por x
k
+x + 1.
1.21 (IMO2002). Encontrar todos os pares de inteiros m, n > 2 tais que
existam innitos valores de k para os quais
k
m
+k 1
k
n
+k
2
1
e inteiro.
1.2 Ordem e Razes Primitivas
Dado a (Z/nZ)

, denimos a ordem de a, denotado por ord a,


como o menor inteiro t > 0 tal que a
t
= 1 em Z/nZ. Se a, n Z
com mdc(a, n) = 1, denimos a ordem de a modulo n, denotado por
ord
n
a, como a ordem de a (Z/nZ)

. Note que pelo teorema de Euler-


Fermat, temos que ord
n
a (n). Se ord
n
a = (n), dizemos que a
e raiz primitiva modulo n. Por exemplo, 2 e raiz primitiva m odulo 5,
pois 2
1
= 2, 2
2
= 4, 2
3
= 8, 2
4
= 16, que e a primeira potencia de 2
congruente a 1 m odulo 5 e 4 = (5).
O resultado basico mais importante sobre ordem e a seguinte
Proposi cao 1.22. Temos que a
t
1 (mod n) se, e so se, ord
n
a | t.
Demonstra c

ao: Como a
ord
n
a
1 (mod n), para todo k N tem-se
a
k ord
n
a
1 (mod n). Por outro lado, se a
t
1 (mod n), pelo algoritmo
74 CAP

ITULO 1. POT

ENCIAS E CONGRU

ENCIAS
da divisao existem inteiros q e r tais que 0 r < ord
n
a e t = q ord
n
a+r.
Portanto
1 a
t
= a
q ord
n
a+r
= (a
ord
n
a
)
q
a
r
a
r
(mod n)
Ou seja, a
r
1 (mod n). Pela minimalidade de ord
n
a, temos que r = 0,
i.e., ord
n
a | t.
Corolario 1.23. ord
n
a | (n).
Exemplo 1.24. Determine a ordem de 2 modulo 7337 (observacao:
7337 = 11 23 29)
Solu c

ao: Como
2
5
1 (mod 11) temos que ord
11
2 = 10,
2
11
= 2048 1 (mod 23) temos ord
23
2 = 11 e
2
7
= 128 12 (mod 29), 2
14
12
2
1 (mod 29) temos ord
29
2 =
28.
assim ord
7337
2 = mmc(10, 11, 28) = 1540.
Exemplo 1.25. Demonstrar que n | (a
n
1) para todo inteiro positivo
a > 1.
Solu c

ao: Ja que mdc(a, a


n
1) = 1, pelo teorema de Euler-Fermat
temos que a
(a
n
1)
1 (mod a
n
1); por outro lado, n e a ordem de
a m odulo a
n
1 ja que a
n
1 (mod a
n
1) e se 0 < t < n temos
0 < a
t
1 < a
n
1 e assim a
n
1 a
t
1. Pela proposicao, temos
portanto n | (a
n
1).
Exemplo 1.26. Demonstrar que n ao existe um inteiro n > 1 tal que
n | 2
n
1.
Solu c

ao: Suponhamos o contrario; seja p o menor divisor primo de n


e r = ord
p
2. Sabemos que 2
n
1 (mod p) e alem disso, pelo teorema
de Fermat, 2
p1
1 (mod p).
Portanto r | n e r | p 1, o que implica que r | mdc(n, p 1). Mas
mdc(n, p1) = 1 pois p e o menor divisor primo de n e assim os divisores
primos de p 1 s ao menores que os divisores primos de n. Isto mostra
que r = 1, isto e 2
1
1 (mod p), donde p | 1, uma contradi cao.
1.2. ORDEM E RA

IZES PRIMITIVAS 75
Exemplo 1.27. Sejam a, m e n inteiros positivos; dena m

e n

por
m = mdc(m, n) m

e n = mdc(m, n) n

, de modo que mdc(m

, n

) = 1.
Mostre que
mdc(a
m
+ 1, a
n
+ 1) =
_

_
a
mdc(m,n)
+ 1 se m

e n

sao mpares.
2 se m

+n

e a sao mpares.
1 se m

+n

e mpar e a e par.
Solu c

ao: Como
mdc(a
m
+ 1, a
n
+ 1) = mdc
_
(a
mdc(m,n)
)
m

+ 1, (a
mdc(m,n)
)
n

+ 1
_
,
o resultado no caso geral seguir a do caso em que mdc(m, n) = 1. Assim,
vamos supor m e n s ao primos entre si e seja d = mdc(a
n
+ 1, a
m
+ 1).
Temos
_
a
n
1 (mod d)
a
m
1 (mod d)
=
_
a
2n
1 (mod d)
a
2m
1 (mod d)
= ord
d
a | mdc(2n, 2m) = 2.
Assim, a
2
1 (mod d). Digamos que m seja mpar (como estamos
supondo mdc(m, n) = 1, nao podemos ter m e n ambos pares), de modo
que
a (a
2
)
(m1)/2
= a
m
1 (mod d) = a 1 (mod d)
d | a + 1.
Se n e mpar tambem, entao d = a+1 ja que a+1 | a
m
+1 e a+1 | a
n
+1
neste caso (utilize a fatora cao a
m
+ 1 = (a + 1)(a
m1
a
m2
+a
m3

+1) ou a implicacao a 1 (mod a+1) = a


m
1 (mod a+1)).
Por outro lado, se n e par, temos
(a
2
)
n/2
= a
n
1 (mod d) = 1 1 (mod d)
= d = 1 ou d = 2
O caso d = 2 ocorre se, e s o se, a
m
+ 1 e a
n
+ 1 s ao ambos pares, ou
seja, quando a e mpar. Isto encerra a an alise de casos e com isso o
problema.
Uma outra caracteriza cao de raiz primitiva e dada pela
Proposi cao 1.28. O n umero a e raiz primitiva modulo n se, e somente
se, {a
t
, t N} = (Z/nZ)

.
76 CAP

ITULO 1. POT

ENCIAS E CONGRU

ENCIAS
Demonstra c

ao: Para todo a Z com mdc(a, n) = 1 temos {a


t
, t
N} (Z/nZ)

. Note que {a
t
, t N} = {1, a, a
2
, . . . , a
ord
n
a1
} e um
conjunto com ord
n
a elementos. De fato, para qualquer t N temos
a
t
= a
r
onde r e o resto na divisao de t por ord
n
a; por outro lado,
os elementos 1, a, a
2
, . . . , a
ord
n
a1
s ao distintos pois caso a
i
= a
j
com
0 i < j < ord
n
a, entao a
ji
= 1 com 0 < j i < ord
n
a, o que e
absurdo.
Assim, {a
t
, t N} = (Z/nZ)

se, e s o se, mdc(a, n) = 1 e ord


n
a =
(n), isto e, se, e s o se, a e uma raiz primitiva m odulo n.
Corolario 1.29. Se m divide n e a e raiz primitiva modulo n, ent ao a
e raiz primitiva modulo m.
Demonstra c

ao: Como o mapa natural (Z/nZ)

(Z/mZ)

que
leva x mod n em x mod m e sobrejetor, temos que se as potencias de
a mod n cobrem todo o (Z/nZ)

, entao as potencias de a mod m tam-


bem cobrem todo o (Z/mZ)

. Pela proposicao, isto implica o corolario.


Razes primitivas s ao muito uteis em diversas quest oes de Teoria dos
N umeros. Entretanto elas nem sempre existem para qualquer m odulo
n. O resto desta secao e dedicado a provar o seguinte importante
Teorema 1.30. Existe alguma raiz primitiva modulo n se, e so se,
n = 2, n = 4, n = p
k
ou n = 2p
k
onde p e primo mpar.
A demonstra cao deste teorema e longa e e composta de varios passos.
Come camos com a seguinte
Proposi cao 1.31. Se k 3, ent ao n ao existe nenhuma raiz primitiva
modulo 2
k
.
Demonstra c

ao: Pelo corolario anterior, basta provar que nao existe


raiz primitiva m odulo 8, e isso segue do fato de que se mdc(a, 8) = 1,
isto e, a = 2r + 1, r N, entao a
2
= 4r(r + 1) + 1 1 (mod 8) (sendo
r(r+1) par, visto que e o produto de dois n umeros consecutivos). Assim,
nao ha elemento de ordem (8) = 4 m odulo 8.
Proposi cao 1.32. Se n = ab, com a 3 e b 3 inteiros tais que
mdc(a, b) = 1, ent ao n ao existe raiz primitiva modulo n.
Demonstra c

ao: Como (n) = (a)(b) e a 3 e b 3, segue que


(a) e (b) s ao pares (verique!). Se mdc(k, n) = 1, entao temos
k
(n)/2
= (k
(b)/2
)
(a)
1 (mod a) e
k
(n)/2
= (k
(a)/2
)
(b)
1 (mod b).
1.2. ORDEM E RA

IZES PRIMITIVAS 77
Assim, k
(n)/2
1 (mod n) e portanto ord
n
k (n)/2 < (n) para
todo k primo com n.
Proposi cao 1.33. Se p e um n umero primo e a Z e uma raiz primi-
tiva modulo p, ent ao a ou a +p e raiz primitiva modulo p
2
.
Demonstra c

ao: Por hip otese, ord


p
a = ord
p
(a + p) = (p) = p 1.
Portanto p 1 | ord
p
2 a, pois a
t
1 (mod p
2
) implica a
t
1 (mod p).
Alem disso, como ord
p
2 a | (p
2
) = p(p 1), devemos ter ord
p
2 a = p 1
ou ord
p
2 a = p(p1) = (p
2
). Do mesmo modo, ord
p
2(a+p) = p1 ou
ord
p
2(a + p) = p(p 1) = (p
2
). Basta provar, portanto, que ord
p
2 a =
p 1 ou ord
p
2(a + p) = p 1. Suponha que ord
p
2 a = p 1. Portanto
a
p1
1 (mod p
2
) e assim
(a +p)
p1
= a
p1
+
_
p 1
1
_
a
p2
p +
_
p 1
2
_
a
p3
p
2
+
1 pa
p2
(mod p
2
).
Portanto (a +p)
p1
nao e congruente a 1 m odulo p
2
, pois p
2
nao divide
pa
p2
(lembre-se de que mdc(a, p) = 1), donde ord
p
2(a+p) = p1.
Proposi cao 1.34. Se p e um n umero primo mpar e a e raiz primitiva
modulo p
2
, ent ao a e raiz primitiva modulo p
k
para todo k N.
Demonstra c

ao: Como a
p1
1 (mod p), mas a
p1
nao e congruente
a 1 m odulo p
2
(ja que a e raiz primitiva m odulo p
2
), temos a
p1
= 1+b
1
p,
onde p nao divide b
1
. Vamos mostrar por indu cao que a
p
k1
(p1)
=
1 + b
k
p
k
, onde p nao divide b
k
, para todo k 1. De fato, para k 1 e
p > 2 primo,
a
p
k
(p1)
= (1 +b
k
p
k
)
p
= 1 +
_
p
1
_
b
k
p
k
+
_
p
2
_
b
2
k
p
2k
+
= 1 +p
k+1
(b
k
+pt)
para algum t Z e assim b
k+1
= b
k
+ pt tambem nao e divisvel por p
pois p b
k
.
Vamos agora mostrar por indu cao que a e raiz primitiva m odulo p
k
para todo k 2. Suponha que a seja raiz primitiva m odulo p
k
. Como
a
ord
p
k+1
a
1 (mod p
k+1
) = a
ord
p
k+1
a
1 (mod p
k
) temos
p
k1
(p 1) = (p
k
) = ord
p
k a | ord
p
k+1 a | (p
k+1
) = p
k
(p 1).
Portanto ord
p
k+1 a = p
k1
(p 1) ou ord
p
k+1 a = p
k
(p 1) = (p
k+1
),
mas o primeiro caso e impossvel pois a
p
k1
(p1)
= 1 + b
k
p
k
com p b
k
.
Logo ord
p
k+1 a = (p
k+1
) e a e raiz primitiva m odulo p
k+1
.
78 CAP

ITULO 1. POT

ENCIAS E CONGRU

ENCIAS
Por exemplo 2 e raiz primitiva m odulo 5
k
para todo k 1. De fato,
2 e raiz primitiva m odulo 5 e, como 2
4
= 16 1 (mod 25), 2 e raiz
primitiva m odulo 25 = 5
2
tambem. Portanto, pela proposicao anterior,
2 e raiz primitiva m odulo 5
k
para todo k 1.
Proposi cao 1.35. Se p e primo mpar e a e um inteiro mpar tal que
a e raiz primitiva modulo p
k
, ent ao a e raiz primitiva modulo 2p
k
. Em
particular, se a e raiz primitiva qualquer modulo p
k
, ent ao a ou a + p
k
e raiz primitiva modulo 2p
k
(pois um deles e mpar).
Demonstra c

ao: Temos, como nas provas acima, (p


k
) = ord
p
k a |
ord
2p
k a e ord
2p
k a | (2p
k
) = (p
k
), logo ord
2p
k a = (2p
k
).
Para completar a prova do teorema 1.30, falta provar que se p e primo
mpar, entao existe raiz primitiva m odulo p. Para isto, precisamos de
dois lemas.
Lema 1.36.

d|n
(d) = n para todo n N.
Demonstra c

ao: Seja d um divisor de n. A quantidade de as tais


que 1 a n e d = mdc(n, a) e igual a (
n
d
) pois d = mdc(n, a)
d | a e 1 = mdc(
n
d
,
a
d
). Como (
n
d
) conta justamente a quantidade
de inteiros entre 1 e
n
d
(inclusive) que s ao primos com
n
d
, temos que

d|n
(
n
d
) =

d|n
(d) conta a quantidade de n umeros a entre 1 e n
(inclusive), particionados segundo os valores de mdc(a, n).
Lema 1.37. Seja p um primo e d um divisor de p 1. Dena N(d)
como a quantidade de elementos a (Z/pZ)

com ord a = d. Ent ao


N(d) (d).
Demonstra c

ao: Podemos supor que N(d) > 0, logo existe a tal que
ord
p
a = d. Logo a
d
= 1 e, para 0 k < d, as classes de a
k
s ao todas
distintas m odulo p. Como (a
k
)
d
= 1 e a equa cao x
d
1 = 0 tem no
m aximo d razes distintas em Z/pZ (pois Z/pZ e um corpo), suas razes
s ao exatamente a
k
, 0 k < d. Por outro lado, se ord
p
a
k
= d, entao
mdc(k, d) = 1, pois caso r = mdc(k, d) > 1, entao (a
k
)
d/r
= (a
d
)
k/r
1
(mod p), logo ord
p
(a
k
) d/r < d. Desta forma,
{b (Z/pZ)

| ord
p
b = d} {a
k
| 0 k < d e mdc(k, d) = 1},
portanto N(d) (d) (na verdade, os dois conjuntos acima s ao iguais,
como cara claro a partir da demonstra cao da proposicao abaixo).
1.2. ORDEM E RA

IZES PRIMITIVAS 79
Proposi cao 1.38. Se p e um primo, ent ao existe uma raiz primitiva
modulo p.
Demonstra c

ao: Para cada a (Z/pZ)

, tem-se ord
p
a | p 1 e
portanto p 1 =

d|p1
N(d). Por outro lado, temos pelos dois lemas
acima que
p 1 =

d|p1
N(d)

d|p1
(d) = p 1.
Logo devemos ter N(d) = (d) para todo d. Em particular, N(p 1) =
(p 1) > 0, logo existem razes primitivas m odulo p.
Corolario 1.39. Seja p um primo. Para cada d | p 1, existem exa-
tamente (d) elementos em (Z/pZ)

com ordem d. Em particular, p


possui exatamente (p 1) razes primitivas.
Com isto, encerramos a demonstra cao do teorema 1.30. Vejamos
algumas aplicacoes.
Exemplo 1.40. Determine uma raiz primitiva modulo 419.
Solu c

ao: Sabemos que (419) = 418 = 2 11 19, e os divisores


proprios de 418 s ao 1, 2, 11, 19, 22, 38 e 209. Assim, precisamos encon-
trar um a que nao tenha nenhum destes n umeros como ordem. Vejamos
se a = 2 funciona. Para isto observemos que, m odulo 419,
2
2
= 4, 2
11
= 2048 372, 2
19
372 256 119
2
22
372
2
114, 2
38
119
2
334, 2
209
119
11
418
logo ord
419
2 = 418, e 2 e raiz primitiva m odulo 419.
Observe que todas as razes primitivas m odulo 419 podem ser geradas
calculando 2
k
(mod 419), onde mdc(k, 418) = 1, assim por exemplo
2
5
= 32 e 2
9
93 (mod 419) tambem s ao razes primitivas.
Exemplo 1.41. Mostre que existe n natural tal que os mil ultimos d-
gitos de 2
n
pertencem a {1, 2}.
Solu c

ao: Observamos inicialmente que para todo k N existe um


n umero m
k
de k algarismos, todos 1 ou 2, divisvel por 2
k
. De fato,
m
1
= 2 e m
2
= 12 satisfazem o enunciado. Seja m
k
= 2
k
r
k
, r
k
N. Se
r
k
e par, tome m
k+1
= 210
k
+m
k
= 2
k+1
(5
k
+r
k
/2), e se r
k
e mpar,
tome m
k+1
= 10
k
+m
k
= 2
k+1
(5
k
+r
k
)/2.
Como m
1000
2 (mod 10), 5 nao divide r
1000
=
m
1000
2
1000
. Portanto,
como 2 e raiz primitiva m odulo 5
1000
pela proposicao 1.34, existe k N
80 CAP

ITULO 1. POT

ENCIAS E CONGRU

ENCIAS
com 2
k
r
1000
(mod 5
1000
). Logo 2
k
= b5
1000
+r
1000
para algum b N
e assim
2
k+1000
= b10
1000
+ 2
1000
r
1000
= b10
1000
+m
1000
,
e as 1000 ultimas casas de 2
k+1000
s ao as 1000 casas de m
1000
, que
pertencem todas a {1, 2}.
Observa cao 1.42. Um grupo G e chamado de cclico se existe um ele-
mento g tal que G = {g
n
| n Z}. O fato de p
n
e 2p
n
, p primo mpar,
admitirem razes primitivas equivale a dizer que os grupos (Z/p
n
Z)

e
(Z/2p
n
Z)

sao cclicos, ou ainda que ha isomorsmos de grupos (Z/p


n
Z)


=
Z/(p
n
) e (Z/2p
n
Z)


= Z/(2p
n
) onde a operacao nos grupos da direita
e a adi cao.
O leitor n ao deve ter diculdades para adaptar a prova acima a m de
mostrar que todo corpo K com um n umero nito de elementos (tal como
o construdo no exemplo apos o teorema 1.14) admite raiz primitiva, isto
e, o seu grupo de unidades K

= K \ {0} e um grupo cclico.


Problemas Propostos
1.22. Determine a ordem de 3 modulo 200.
1.23. Encontre uma raiz primitiva modulo 71.
1.24. Sabendo que ord
13
2 a = 4, ord
11
2 a = 55 e ord
7
2 a = 21, determine
ord
1001
2 a.
1.25. Determine todos os valores inteiros de n tal que n|2
2n
+ 1.
1.26. Determine uma raiz primitiva modulo 7
3
.
1.27. Encontrar as ordens de 2 e 5 modulo 101. Encontrar tambem
todos os elementos de ordem 20 em (Z/101Z)

.
1.28. Demonstrar que 2n | (a
n
+ 1) para todo inteiro positivo a.
1.29 (IMO1978). Sejam m e n inteiros positivos com m < n. Se os
tres ultimos algarismos de 1978
m
sao os mesmos que os tres ultimos
algarismos de 1978
n
, encontrar m e n tais que m + n assume o menor
valor possvel.
1.30. Sejam d e n n umeros naturais tais que d | 2
2
n
+1. Demonstre que
existe um inteiro k tal que d = k2
n+1
+ 1.
1.3. RES

IDUOS QUADR

ATICOS E S

IMBOLO DE LEGENDRE 81
1.31. Seja k 2 e n
1
, n
2
, . . . , n
k
1 n umeros naturais que tem a
propriedade
n
2
| (2
n
1
1), n
3
| (2
n
2
1), . . . , n
k
| (2
n
k1
1) e n
1
| (2
n
k
1)
Demonstrar que n
1
= n
2
= = n
k
= 1.
1.32. Mostrar que x
3
x+1 e irredutvel em Z/3Z[x]. Encontrar todas
as razes primitivas do corpo nito
Z/3Z[x]
(x
3
x+1)
.
1.33 (APMO1997). Encontrar um n no conjunto {100, 101, . . . 1997} tal
que n divide 2
n
+ 2.
1.34. Denimos a funcao de Carmichael : N N como o menor in-
teiro positivo tal que a
(n)
1 (mod n) para todo a primo com n. Ob-
serve que, pelo teorema 1.30, (p
l
) = p
l1
(p 1) para todo p primo
mpar. Mostrar que
(a) (2) = 1, (4) = 2 e (2
l
) = 2
l2
para todo l 3.
(b) Se n = p

1
1
. . . p

k
k
e a fatoracao em primos de n, ent ao
(n) = mmc{(p

1
1
), . . . , (p

k
k
)}.
1.35 (IMO2000). Existe um inteiro N divisvel por exatamente 2000
primos diferentes e tal que N divide 2
N
+ 1?
Dica: Tente construir indutivamente inteiros N
k
divisveis por exa-
tamente k primos distintos e tais que N
k
| 2
N
k
+ 1.
1.36 (IMO1990). Encontrar todos os n umeros naturais n tais que
n
2
| 2
n
+ 1.
1.37 (IMO1999). Encontrar todos os pares (n, p) de inteiros positivos
tais que p e primo, n 2p e (p 1)
n
+ 1 e divisvel por n
p1
.
1.38 (Banco-IMO2000). Determine todas as triplas (a, m, n) de inteiros
positivos tais que a
m
+ 1 | (a + 1)
n
.
1.3 Resduos Quadraticos e Smbolo de Legen-
dre
Seja p > 2 um n umero primo e a, b, c Z com a nao divisvel por p.
Resolver a equa cao quadratica
ax
2
+bx +c 0 (mod p)
82 CAP

ITULO 1. POT

ENCIAS E CONGRU

ENCIAS
e o mesmo que resolver (completando quadrados)
(2ax +b)
2
b
2
4ac (mod p)
(note que 2 e a s ao invertveis m odulo p). Assim, estamos interessados
em encontrar criterios de existencia de solucoes da equa cao
X
2
d (mod p).
Se a equa cao acima admite solucao (i.e. se d e um quadrado perfeito
em Z/pZ) entao dizemos que d e um resduo ou resto quadr atico m odulo
p. H a exatamente (p + 1)/2 resduos quadraticos m odulo p, a saber
0
2
, 1
2
, 2
2
, 3
2
, . . . ,
_
p 1
2
_
2
mod p
ja que todo inteiro x e congruente a i mod p para algum i tal que
0 i (p 1)/2, de modo que x
2
e congruente a um dos n umeros da
lista acima. Note que m odulo p estes n umeros s ao todos distintos: de
fato, temos que
i
2
j
2
(mod p) = p | (i j)(i +j)
p | i j ou p | i +j
i j (mod p)
Mas como 0 i, j (p 1)/2 = 0 < i + j p 1 ou i = j = 0,
temos que a unica possibilidade e i j (mod p).
Embora saibamos a lista completa dos resduos quadraticos, na pra-
tica pode ser difcil reconhecer se um n umero e ou nao resduo quadra-
tico. Por exemplo, voce sabe dizer se 2 e resduo quadratico m odulo
1019? Veremos a seguir o teorema da reciprocidade quadratica, que
permite responder estas quest oes de maneira bastante eciente.
Seja p > 2 um n umero primo e a um inteiro qualquer. Para simpli-
car calculos e nota coes deniremos o chamado smbolo de Legendre:
_
a
p
_
=
_

_
1 se p a e a e um resduo quadratico m odulo p
0 se p | a
1 caso contrario
Proposi cao 1.43 (Criterio de Euler). Seja p > 2 um primo e a um
inteiro qualquer. Ent ao
_
a
p
_
a
(p1)/2
(mod p).
1.3. RES

IDUOS QUADR

ATICOS E S

IMBOLO DE LEGENDRE 83
Demonstra c

ao: Para a 0 (mod p) o resultado e claro, de modo


que podemos supor p a. Pelo teorema de Fermat temos que a
p1
1
(mod p), donde
(a
p1
2
1)(a
p1
2
+ 1) 0 (mod p) p | a
p1
2
1 ou p | a
p1
2
+ 1
a
p1
2
1 (mod p).
Assim, devemos mostrar que a
p1
2
1 (mod p) se, e s o se, a e um
resduo quadratico m odulo p.
Se a e um resduo quadratico, digamos a i
2
(mod p), novamente
pelo teorema de Fermat temos que
a
p1
2
i
p1
1 (mod p).
Assim, os resduos quadraticos 1
2
, 2
2
, . . . , (
p1
2
)
2
m odulo p s ao razes do
polin omio f(x) = x
p1
2
1 em Z/(p)[x]. Mas Z/(p) e corpo, logo f(x)
pode ter no m aximo deg f = (p 1)/2 razes em Z/(p). Isto mostra que
as razes de f(x) s ao exatamente os resduos quadraticos nao congruentes
a zero m odulo p e que, portanto, a
p1
2
1 (mod p) se, e s o se, a e um
resduo quadratico m odulo p.
Corolario 1.44. O smbolo de Legendre possui as seguintes proprieda-
des:
1. se a b (mod p) ent ao
_
a
p
_
=
_
b
p
_
.
2.
_
a
2
p
_
= 1 se p a.
3.
_
1
p
_
= (1)
p1
2
, ou seja, 1 e resduo quadr atico modulo p se, e
so se, p 1 (mod 4).
4.
_
ab
p
_
=
_
a
p
__
b
p
_
.
Demonstra c

ao: Os itens 1 e 2 s ao imediatos a partir da denicao


e 3 segue do criterio de Euler:
_
1
p
_
(1)
p1
2
(mod p) =
_
1
p
_
=
(1)
p1
2
ja que p > 2 e ambos os lados da congruencia s ao iguais a 1.
Da mesma forma, aplicando o criterio de Euler temos que
_
ab
p
_
(ab)
p1
2
a
p1
2
b
p1
2

_
a
p
__
b
p
_
(mod p),
o que mostra que
_
ab
p
_
=
_
a
p
__
b
p
_
, pois novamente ambos os lados da
congruencia s ao iguais a 1.
84 CAP

ITULO 1. POT

ENCIAS E CONGRU

ENCIAS
Exemplo 1.45. Mostre que o polin omio f(x) = x
4
10x
2
+1 e irredu-
tvel em Z[x], mas e redutvel modulo p para todo primo p.
Solu c

ao: Vejamos que f(x) e irredutvel em Z[x]. Observe inicial-


mente que as razes de f(x) s ao todas irracionais: se p, q Z s ao tais
que mdc(p, q) = 1 e f(p/q) = 0 p
4
10p
2
q
2
+ q
4
= 0, temos da
ultima igualdade que q | p
4
= q = 1 e p | q
4
= p = 1 ja que p e
q s ao primos entre si, logo p/q = 1, nenhuma das quais e raiz de f(x)
(cujos zeros s ao

3).
Logo se f(x) for redutvel ele e o produto de dois polin omios de grau
2, que podemos supor m onicos. Como o produto dos coecientes inde-
pendentes destes dois fatores deve ser igual ao coeciente independente
de f(x), que e 1, temos apenas duas possibilidades:
f(x) = (x
2
+ax + 1)(x
2
+bx + 1) ou
f(x) = (x
2
+ax 1)(x
2
+bx 1)
com a, b Z. Em ambos os casos, temos a + b = 0 (coeciente de
x
3
). Logo, no primeiro caso, comparando o coeciente de x
2
temos
ab + 2 = 10 a
2
= 12, o que e impossvel. O segundo caso e
an alogo.
Agora, para p = 2 e p = 3 temos
f(x) (x + 1)
4
(mod 2) e f(x) (x
2
+ 1)
2
(mod 3).
Agora se p > 3 e um primo, temos que ou
_
2
p
_
= 1, ou
_
3
p
_
= 1 ou
_
6
p
_
= 1
ja que
_
2
p
__
3
p
_
=
_
6
p
_
. No primeiro caso, se a
2
2 (mod p) temos
f(x) (x
2
+ 2ax 1)(x
2
2ax 1) (mod p).
Ja no segundo caso, se b
2
3 (mod p) temos
f(x) (x
2
+ 2bx + 1)(x
2
2bx + 1) (mod p).
Finalmente, no ultimo caso, se c
2
6 (mod p) temos
f(x) (x
2
+ 2c 5)(x
2
2c 5) (mod p).
Isto mostra que f(x) e redutvel m odulo p para todo primo p.
1.3.1 Lema de Gau
O criterio de Euler ja nos fornece uma maneira de identicar re-
sduos quadraticos. Veremos agora outro resultado que fornece varias
interpretacoes para o smbolo de Legendre
_
a
p
_
.
1.3. RES

IDUOS QUADR

ATICOS E S

IMBOLO DE LEGENDRE 85
Lema 1.46 (Gau). Sejam p > 2 um n umero primo e a um inteiro
primo relativo com p. Seja s o n umero de elementos do conjunto
_
a, 2a, 3a, . . . ,
p1
2
a
_
tais que seu resto modulo p e maior do que
p1
2
. Ent ao
_
a
p
_
= (1)
s
.
Interpretamos aqui o resto de n m odulo p como sendo o unico inteiro
n

com 0 n

< p e n n

(mod p). Em particular, mesmo se a for


negativo os restos no enunciado serao maiores ou iguais a 0.
Demonstra c

ao: A ideia e imitar a prova do teorema de Euler-Fermat.


Como o conjunto {1, 2, . . . ,
p1
2
} e um sistema completo de invert-
veis m odulo p, para cada j = 1, 2, . . . ,
p1
2
podemos escrever a j
j
m
j
(mod p) com
j
{1, 1} e m
j
{1, 2, . . . ,
p1
2
}. Temos que se i = j
entao m
i
= m
j
donde {m
1
, m
2
, . . . , mp1
2
} = {1, 2, . . . ,
p1
2
}. De fato, se
m
i
= m
j
temos a i a j (mod p) ou a i a j (mod p); como a e
invertvel m odulo p e 0 < i, j (p 1)/2, temos que a primeira possibi-
lidade implica i = j e a segunda e impossvel. Assim, multiplicando as
congruencias a j
j
m
j
(mod p), obtemos
(a 1)(a 2) (a
p1
2
)
1

2
p1
2
m
1
m
2
mp1
2
(mod p)
a
p1
2
_
p 1
2
_
!
1

2
p1
2
_
p 1
2
_
! (mod p)

_
a
p
_

1

2
. . . p1
2
(mod p),
donde (
a
p
) =
1

2
. . . p1
2
, pois ambos os lados pertencem a {1, 1}.
Assim, (
a
p
) = (1)
s
ja s e o n umero de elementos j de {1, 2, . . . ,
p1
2
}
tais que
j
= 1.
Vejamos uma aplicacao facil deste resultado.
Lema 1.47. Seja p um primo mpar. Ent ao
_
1
p
_
= (1)
p1
2
=
_
1 se p 1 (mod 4),
1 se p 3 (mod 4),
e
_
2
p
_
= (1)
p
2
1
8
=
_
1 se p 1 (mod 8),
1 se p 3 (mod 8).
86 CAP

ITULO 1. POT

ENCIAS E CONGRU

ENCIAS
Note que ja demonstramos a primeira formula na secao anterior;
daremos uma segunda demonstra cao.
Demonstra c

ao: No Lema de Gau, tome a = 1. Temos


_
a, 2a, 3a, . . . ,
p1
2
a
_
=
_
(p + 1)/2, . . . , p 3, p 2, p 1
_
donde s = (p 1)/2 e temos que
_
1
p
_
= (1)
(p1)/2
.
Para a segunda formula, tome a = 2. Se p 1 (mod 4), digamos
p = 4k + 1, temos
p1
2
= 2k. Como 1 2j
p1
2
para j k e
p1
2
< 2j p 1 para k + 1 j 2k, temos
_
2
p
_
= (1)
k
=
_
1, se p 1 (mod 8),
1, se p 5 (mod 8).
Se p 3 (mod 4), digamos p = 4k + 3, temos
p1
2
= 2k + 1. Para
1 j k temos 1 2j
p1
2
e para k + 1 j 2k + 1 temos
p1
2
< 2j p 1, donde
_
2
p
_
= (1)
k+1
=
_
1, se p 3 (mod 8),
1, se p 7 (mod 8).
O leitor pode modicar a demonstra cao acima para descobrir for-
mulas an alogas para, digamos,
_
3
p
_
e
_
5
p
_
. A Lei (ou o Teorema) de
Reciprocidade Quadratica, que veremos na proxima secao, faz isso de
forma muito geral.
Daremos agora duas reformulacoes do Lema de Gau, a primeira
geometrica e combinat oria e a segunda mais algebrica, usando funcoes
trigonometricas. Estas reformulacoes ajudam a apreciar melhor o Lema
e serao utilizadas nas proximas secoes.
Lema 1.48. Seja p > 2 primo e a > 0 inteiro, a primo com p. Considere
em R
2
o tri angulo T de vertices (0, 0), (p/2, 0) e (p/2, a). Seja N o
n umero de pontos de coordenadas inteiras no interior de T. Ent ao
_
a
p
_
= (1)
N
.
1.3. RES

IDUOS QUADR

ATICOS E S

IMBOLO DE LEGENDRE 87
A gura ilustra o resultado acima para p = 7 e a = 5: temos N = 7
e
_
a
p
_
= (1)
N
= 1,
consistentemente com o fato de 5 nao ser quadrado m odulo 7. Observe
que os pontos (k, 0), 1 k (p 1)/2 estao sobre um lado de T e
portanto nao devem ser contados.
Demonstra c

ao: Note que a hipotenusa de T esta contida na reta


y = 2ax/p. Alem disso, como a e p s ao primos entre si nao ha nenhum
ponto de coordenadas inteiras sobre a hipotenusa (fora a origem).
Para k inteiro, 1 k p/2, seja N
k
o n umero de pontos de coorde-
nadas inteiras dentro de T. Armamos que N
k
e mpar se e somente se
o resto de ak m odulo p for maior do que p/2. De fato, seja k

este resto:
temos ak = lp + k

, 0 < k

< p. Por outro lado, por denicao, N


k
e o
n umero de inteiros positivos y para os quais y < 2ak/p. Assim,
N
k
=
_
2ak
p
_
=
_
2(lp +k

)
p
_
= 2l +
_
2k

p
_

_
2k

p
_
(mod 2)
(lembre que x e a parte inteira de x). Mas isto demonstra a armacao
pois a ultima expressao e igual a 1 se k

> p/2 e e igual a 0 caso contrario.


Temos N = N
1
+ + N
(p1)/2
. Pelo par agrafo anterior temos que
N s (mod 2) (onde s e como no enunciado do Lema de Gau), o que,
pelo Lema de Gau, completa a demonstra cao.
Lema 1.49. Seja p > 2 primo e a inteiro, a primo com p. Ent ao

1k(p1)/2
sen
_
2
ak
p
_
=
_
a
p
_

1k(p1)/2
sen
_
2
k
p
_
.
Demonstra c

ao: Observe que para cada k entre 1 e (p 1)/2 existe


um unico k

no mesmo intervalo com ak k

(mod p): assim os dois


88 CAP

ITULO 1. POT

ENCIAS E CONGRU

ENCIAS
grandes produtos acima tem o mesmo m odulo. O segundo produto e
claramente positivo. Por outro lado, temos
sen
_
2
ak
p
_
< 0
se e somente se o resto de ak m odulo p for maior do que p/2. Assim, para
s como no Lema de Gau, o n umero de termos negativos no primeiro
produto e igual a s. Pelo Lema de Gau, isto completa a demonstra cao.
Problemas Propostos
1.39. Calcule
_
2
7
_
,
_
3
11
_
e
_
5
13
_
.
1.40. Quantos pontos de coordenadas inteiras existem no interior do
tri angulo de vertices (0, 0), (3/2, 0) e (3/2, 2012) ?
1.41.
Seja p > 3 primo. Quantos pontos de coordenadas inteiras existem no
interior do tri angulo de vertices (0, 0), (p/2, 0) e (p/2, 3) ?
Para quais primos p o polin omio x
2
3 e redutvel modulo p ?
1.42. a) (Euler) Seja F
n
= 2
2
n
+1 o n-esimo n umero de Fermat. Prove
que todo fator primo de F
n
e da forma k2
n+1
+ 1.
b)(Lucas) Prove que, se n 2, ent ao todo fator primo de F
n
e da
forma k2
n+2
+ 1.
c) Mostre que 2
2
5
+ 1 e composto.
1.4 Lei de Reciprocidade Quadratica
Vejamos agora um resultado mais geral do que os das secoes anteri-
ores e que torna facil calcular se n e um quadrado m odulo p.
Teorema 1.50 (Reciprocidade Quadratica). Sejam p e q primos mpa-
res distintos. Ent ao
_
p
q
__
q
p
_
= (1)
p1
2

q1
2
As identidades no Lema 1.47 s ao muitas vezes consideradas itens
(mais faceis) da Lei de Reciprocidade Quadratica.
Na matem atica, alguns teoremas inspiram muitas demonstra coes, `as
vezes usando ideias completamente diferentes: o teorema de Pit agoras
1.4. LEI DE RECIPROCIDADE QUADR

ATICA 89
e o teorema fundamental da algebra s ao exemplos disso. A Lei da Re-
ciprocidade Quadratica tambem ja tem centenas de demonstra coes di-
ferentes, algumas bastante elementares e outras que usam tecnicas mais
avan cadas. Nas subsecoes veremos tres demonstra coes diferentes para
este teorema. Vejamos agora algumas aplicacoes.
Exemplo 1.51. Determinar se 90 e resduo quadr atico modulo 1019
ou n ao.
Solu c

ao:
_
90
1019
_
=
_
1
1019
__
2
1019
__
3
2
1019
__
5
1019
_
= (1) (1) 1
_
1019
5
_
=
_
4
5
_
=
_
2
2
5
_
= 1
Ou seja, 90 e resduo quadratico m odulo 1019.
Exemplo 1.52. Seja p um n umero primo. Mostre que
1. se p e da forma 4n + 1 ent ao p | n
n
1.
2. se p e da forma 4n 1 ent ao p | n
n
+ (1)
n+1
2n.
Solu c

ao: No primeiro item, 4n 1 (mod p), donde elevando a n


obtemos
(4n)
n
= 2
2n
n
n
(1)
n
(mod p).
Por outro lado, pelo criterio de Euler e pela reciprocidade quadratica
temos
2
2n
= 2
p1
2
(1)
p
2
1
8
(1)
n(2n+1)
(1)
n
(mod p)
Portanto n
n
1 (mod p), como queramos demonstrar.
No segundo item, temos 4n 1 (mod p) e assim
(4n)
n
= 2
2n
n
n
1 (mod p)
mas 2
2n1
= 2
p1
2
(1)
p
2
1
8
= (1)
n(2n1)
(mod p), donde 2
2n
2
(1)
n
(mod p). Conclumos que 2n
n
(1)
n
(mod p) e multiplicando
por 2n e utilizando 4n 1 (mod p) obtemos n
n
2n (1)
n
(mod p),
como desejado.
90 CAP

ITULO 1. POT

ENCIAS E CONGRU

ENCIAS
1.4.1 Uma demonstracao combinat oria
Nossa primeira demonstra cao baseia-se em contar pontos de coorde-
nadas inteiras, como no Lema 1.48; vamos inicialmente enunciar uma
versao um pouco diferente daquele resultado.
Lema 1.53. Seja p > 2 primo e q > 0 inteiro mpar, q primo com p.
Considere em R
2
o tri angulo

T de vertices (0, 0), (p/2, 0) e (p/2, q/2).
Seja

N o n umero de pontos de coordenadas inteiras no interior de

T.
Ent ao
_
q
p
_
= (1)

N
.
Vale a pena chamar a atencao para as diferen cas entre o Lema 1.48
e o Lema 1.53. Neste segundo lema, o inteiro a foi (suspeitamente)
renomeado de q, que agora deve ser mpar. Alem disso, o terceiro vertice
de

T nao e um dos vertices de T: na gura (com p = 7 e q = 5)
mostramos os dois tri angulos. Temos N = 7 e

N = 3. Note que neste
exemplo N

N (mod 2); dado o Lema 1.48, e exatamente isso que
precisamos demonstrar que vale em geral.
Demonstra c

ao: Seja

T o tri angulo de vertices (0, 0), (p, 0) e (p, q).
O n umero N (como no Lema 1.48) pode ser interpretado como o n umero
de pontos em

T com coordenadas inteiras e coordenada x par: de fato, a
cada ponto (k, l) no interior de T corresponde o ponto (2k, l) no interior
de

T.
1.4. LEI DE RECIPROCIDADE QUADR

ATICA 91
Podemos girar os pontos como acima de coordenadas x > p/2 de
meia volta ao redor do ponto (p/2, q/2) para obtermos ainda outra in-
terpretacao para N. Consideremos agora o ret angulo R de vertices (0, 0),
(p/2, 0), (0, q), (p/2, q) cortado pelo segmento (0, 0), (p/2, q/2). Para ob-
termos N, contamos os pontos de coordenadas inteiras no interior de R
tais que a coordenada x e par se e somente se o ponto esta abaixo do
segmento.
Vamos representar nesta gura a soma N +

N. Os pontos de coor-
denada x mpar agora s ao contados uma vez (por

N se estiverem abaixo
do segmento e por N se estiverem acima); os pontos de coordenada x
par s ao contados 0 vezes se estiverem acima do segmento e 2 vezes se
estiverem abaixo (representados por dois crculos).
Ora, estamos interessados na paridade de N+

N. Para ns de estudar
a paridade, contar um pontos duas vezes equivale a nao cont a-lo. Assim
N+

N e congruo m odulo 2 ao n umero de pontos de coordenadas inteiras
no interior de R com coordenada x par. Mas este n umero e m ultiplo
de q 1 (temos q 1 pontos por coluna), logo par, completando a
demonstra cao.
Vejamos nalmente a prova da Lei de Reciprocidade Quadratica.
92 CAP

ITULO 1. POT

ENCIAS E CONGRU

ENCIAS
Demonstra c

ao: Consideremos os tri angulos T


1
e T
2
de vertices (0, 0),
(p/2, 0), (p/2, q/2) e (0, 0), (0, q/2) e (p/2, q/2). Sejam N
1
e N
2
o n umero
de pontos de coordenadas inteiras no interior de T
1
e T
2
, respectivamente.
Pela Lema 1.53,
_
q
p
_
= (1)
N
1
,
_
p
q
_
= (1)
N
2
.
Mas N
1
+N
2
= (p 1)(q 1)/4. Assim,
_
q
p
__
p
q
_
= (1)
(p1)(q1)
4
,
como queramos.
1.4.2 Uma demonstracao trigonometrica
Nesta subsecao usaremos as abrevia coes
s(x) = sen(2x), c(x) = cos(2x).
Assim as funcoes s e c tem perodo 1. Por exemplo, s(x) = 0 se e somente
se x e inteiro ou da forma inteiro mais meio. Primeiro demonstraremos
uma formula para um produto de senos.
Lema 1.54. Seja q > 0 um inteiro mpar. Ent ao
s(qx) = (4)
q1
2

0l<q
s
_
x +
l
q
_
.
Assim, por exemplo,
s(3x) = 4s(x)s
_
x +
1
3
_
s
_
x +
2
3
_
.
1.4. LEI DE RECIPROCIDADE QUADR

ATICA 93
H a muitas maneiras de demonstrar estas identidades: a demonstra cao
abaixo usa n umeros complexos e as formulas de Euler:
e(x) = c(x) +is(x) = exp(2ix),
s(x) =
e(x) e(x)
2i
, c(x) =
e(x) +e(x)
2
.
Demonstra c

ao: Seja = e(1/q) e uma raiz do polin omio w


q
1 e
temos
w
q
1 =

0j<q
w
j
.
Seja z = e(x); temos
s(qx) =
z
q
z
q
2i
, s
_
x +
l
q
_
=

l
z
l
z
1
2i
.
Substituindo na identidade do enunciado, devemos portanto demonstrar
que
z
q
z
q
=

0l<q
_

l
z
l
z
1
_
.
Multiplicando o lado esquerdo por z
q
e cada fator do lado direito por z,
devemos mostrar que
z
2q
1 =

0l<q
_

l
z
2

l
_
.
Ora, colocando
l
em evidencia, temos

0l<q
_

l
z
2

l
_
=
q(q1)/2

0l<q
_
z
2

2l
_
.
Como q e mpar, (q 1)/2 e inteiro e portanto
q(q1)/2
= 1; quando l
varia de 0 a q 1 temos que 2l corre um sistema completo de resduos
equivalente portanto a j correr de 0 a q 1. Devemos portanto provar
que
z
2q
1 =

0j<q
_
z
2

j
_
.
Fazendo z
2
= w, esta e a primeira identidade da demonstra cao.
Lema 1.55. Seja q > 0 um inteiro mpar. Ent ao
s(qx)
s(x)
= (4)
q1
2

1l(q1)/2
_
s
_
x +
l
q
_
s
_
x
l
q
__
.
Demonstra c

ao: Segue imediatamente do lema anterior, passando


s(x) para o denominador e juntando os termos correspondentes a l e
q l.
94 CAP

ITULO 1. POT

ENCIAS E CONGRU

ENCIAS
Lema 1.56. Seja p > 2 primo e q inteiro mpar, q primo com p. Ent ao
_
q
p
_
= (4)
(p1)(q1)
4

1k(p1)/2,1l(q1)/2
_
s
_
k
p
+
l
q
_
s
_
k
p

l
q
__
.
Demonstra c

ao: Pelo Lema 1.49 temos


_
q
p
_
=

1k(p1)/2
s(qk/p)
s(k/p)
.
Aplicando o Lema 1.55 a cada termo temos
_
q
p
_
=

1k(p1)/2
_
_
(4)
q1
2

1l(q1)/2
_
s
_
k
p
+
l
q
_
s
_
k
p

l
q
__
_
_
.
Passando as potencias de 4 para fora do produtorio e juntando os dois
produtorios temos a formula do enunciado.
Estamos agora prontos para concluir a segunda demonstra c ao da Lei
de Reciprocidade Quadratica.
Demonstra c

ao: Se p e q s ao primos distintos e maiores do que 2


entao usando duas vezes o Lema 1.56 temos
_
q
p
_
_
p
q
_
=

1k(p1)/2,1l(q1)/2
s
_
k
p
+
l
q
_
s
_
k
p

l
q
_
s
_
k
p
+
l
q
_
s
_

k
p
+
l
q
_
= (1)
(p1)(q1)
4
,
como queramos.
1.4.3 Uma demonstracao usando corpos nitos
Nas duas subsecoes anteriores apresentamos duas demonstra coes bas-
tante elementares da Lei de Reciprocidade Quadratica. Nossa terceira
demonstra cao usa ideias de algebra em um sentido mais moderno, espe-
cialmente corpos nitos.
Ao longo de toda a subsecao, sejam p, q primos mpares distintos.
Desejamos construir um corpo K com Z/(q) K dentro do qual exista
uma raiz p-esima da unidade . Para isso, considere no anel de polin o-
mios (Z/(q))[Z] o polin omio
R
0
=
Z
p
1
Z 1
= Z
p1
+Z
p2
+ +Z + 1;
1.4. LEI DE RECIPROCIDADE QUADR

ATICA 95
seja R
1
um fator irredutvel de R
0
. Finalmente, sejam
K =
(Z/(q))[Z]
(R
1
)
, = Z.
(Mais precisamente, e a classe de equivalencia de Z no quociente.)
Assim, por exemplo, se p = 3 e q = 7 temos
R
0
= Z
2
+Z + 1 = (Z 2)(Z 4)
e podemos tomar R
1
= Z 2, K = Z/(7) e = 2. Por outro lado, se
p = 3 e q = 5 temos que R
0
= Z
2
+ Z + 1 e irredutvel (verique!) e
tomamos portanto R
1
= R
0
: neste caso K e um corpo de 25 elementos:
K = {a +b, a, b Z/(5)},
2
= 1 .
Se p = 5 e q = 7 entao novamente R
0
e irredutvel e K tem 7
4
elementos.
Ja se p = 7 e q = 13 entao R
0
e o produto de tres polin omios irredutveis
de grau 2 (encontre-os!) e K tem 13
2
elementos.
Denimos agora em K as somas de Gau:
g =

0<j<p
_
j
p
_

j
.
Assim, por exemplo, para p = 3 e q = 7 temos g =
_
1
3
_
+
_
2
3
_

2
=
2
=
24 = 5 Z/(7) K. Para p = 3 e q = 5 temos g =
2
= 1+2 K.
A partir de agora usaremos a nota cao
p

=
_
1
p
_
p =
_
p, p 1 (mod 4),
p, p 1 (mod 4).
Lema 1.57. Sejam p, q, K, e g K como acima. Ent ao g
2
= p

.
Demonstra c

ao: Temos
g
2
=

0<j
1
,j
2
<p
_
j
1
j
2
p
_

j
1
+j
2
=

0j<p
_
_

0k<p
_
k(j k)
p
_
_
_

j
.
Ora, demonstraremos no Lema 1.58 abaixo que

0k<p
_
k(j k)
p
_
=
_
(p 1)
_
1
p
_
, j 0 (mod p),
(1)
_
1
p
_
, j 0 (mod p).
Assim
_
1
p
_
g
2
= (p 1) +

0<j<p
(1)
j
= p (1 + +
2
+ +
p1
) = p.
96 CAP

ITULO 1. POT

ENCIAS E CONGRU

ENCIAS
Lema 1.58.

0k<p
_
k(j k)
p
_
=
_
(p 1)
_
1
p
_
, j 0 (mod p),
(1)
_
1
p
_
, j 0 (mod p).
Demonstra c

ao: No caso j = 0 temos


_
k(j k)
p
_
=
_
k
2
p
_
=
_
1
p
_
para todo k = 0, demonstrando a identidade desejada. Para j = 0
considere o conjunto P = Z/(p) {} e a funcao m : P P denida
por
m(k) =
_

_
jk
k
, k = 0, ,
, k = 0,
1, k = .
Observe que m e uma bijecao. Temos

0k<p
_
k(j k)
p
_
=

kP{0,}
_
m(k)
p
_
=

P{,1}
_
k

p
_
=
=
_
_

0k

<p
_
k

p
_
_
_

_
1
p
_
=
_
1
p
_
.
Lema 1.59. Sejam p, q, K e g K como acima. Ent ao g Z/(q) K
se e somente se
_
p

q
_
= 1.
Demonstra c

ao: Sabemos que g


2
= p

. Suponha inicialmente que


_
p

q
_
= 1. N ao existe raiz quadrada de p

em Z/(q) donde g / Z/(q).


Suponha agora que
_
p

q
_
= 1. Existem elementos a Z/(q) com
a
2
= p

. Mas isto signica que em K[X] temos X


2
p

= (Xa)(X+a),
ou seja, os unicos elementos x de K com x
2
= p

s ao x = a. Assim
g = a e g Z/(q).
Generalizamos as somas de Gau: para k um inteiro primo com p,
g
k
=

0<j<p
_
j
p
_

kj
;
assim g = g
1
. Observe que
g
k
=

0<j

<p
_
kj

p
_

=
_
k
p
_
g.
1.4. LEI DE RECIPROCIDADE QUADR

ATICA 97
Lema 1.60. Sejam p, q, K, g K como acima. Ent ao g Z/(q) se e
somente se
_
q
p
_
= 1.
Demonstra c

ao: Seja f : K K denida por f(x) = x


q
. Observe
que f(x) = x para todo x Z/(q) (Fermat). Reciprocamente, f(x) = x
implica que x e raiz de X
q
X, um polin omio de grau q; ora, os elementos
de Z/(q) nos dao q razes e estas s ao portanto as unicas razes. Em outras
palavras, f(x) = x se e somente se x Z/(q).
Observe que f(xy) = f(x)f(y). Alem disso,
f(x +y) = (x +y)
q
=
= x
q
+
_
q
1
_
x
q1
y + +
_
q
k
_
x
qk
y
k
+ +y
q
=
= x
q
+y
q
= f(x) +f(y)
pois
_
q
k
_
e m ultiplo de q para 0 < k < q.
Temos
f(g) =
_
_

0<j<p
_
j
p
_

j
_
_
q
=

0<j<p
_
j
p
_
qj = g
q
=
_
q
p
_
g.
Isto completa a demonstra cao.
Lema 1.61. Sejam p e q primos mpares distintos. Ent ao
_
p

q
_
=
_
q
p
_
.
Demonstra c

ao: Sejam p, q, K, g K como acima. Pelo Lema 1.59,


g Z/(q) se e somente se
_
p

q
_
= 1. Pelo Lema 1.60, g Z/(q) se e
somente se
_
q
p
_
= 1. Assim
_
p

q
_
= 1 se e somente se
_
q
p
_
= 1, como
queramos.
A Lei da Reciprocidade Quadratica segue diretamente deste ultimo
lema:
_
p
q
__
q
p
_
=
_
(1)
p1
2
q
__
p

q
__
q
p
_
= (1)
(p1)(q1)
4
.
Podemos tambem considerar este ultimo lema como uma reformulacao
da Lei de Reciprocidade Quadratica.
98 CAP

ITULO 1. POT

ENCIAS E CONGRU

ENCIAS
Problemas Propostos
1.43. Calcular
_
44
103
_
,
_
60
1019
_
e
_
2010
1019
_
.
1.44. Prove que o polin omio x
4
16x
2
+ 4 e irredutvel em Z[x] mas
n ao e irredutvel em (Z/(p))[x] para nenhum primo p.
1.45. Sejam p um primo mpar e c um inteiro que n ao e m ultiplo de p.
Prove que
p1

a=0
_
a(a +c)
p
_
= 1.
1.46. Existem inteiros m e n tais que
5m
2
6mn + 7n
2
= 1985 ?
1.47. Demonstrar que a congruencia 6x
2
+ 5x + 1 0 (mod m) tem
solu cao para todo valor natural de m.
1.48. Demonstrar que existem innitos primos da forma 3k+1 e 3k1.
1.49. Demonstrar que se mdc(a, b) = 1 o n umero a
2
+ b
2
n ao pode ter
fatores primos da forma 4k 1 e se alem disso mdc(a, 3) = 1 ent ao o
n umero a
2
+ 3b
2
n ao pode ter fatores da forma 3k 1. Que podemos
dizer sobre os fatores primos de a
2
+pb
2
onde p e um primo?
1.50. Demonstrar que, para p = 1093,
2
p1
2
1 (mod p
2
)
1.51 (IMO1996). Sejam a, b inteiros positivos tais que 15a+16b e 16a
15b sejam quadrados perfeitos. Encontrar o menor valor que pode tomar
o menor destes quadrados.
1.52. Seja p um n umero primo mpar. Mostrar que o menor n ao resto
quadr atico positivo de p e menor que

p + 1.
1.53. Sejam M um n umero inteiro e p um n umero primo maior do que
25. Mostrar que a sequencia M, M + 1, , M + 3

p 1 contem um
resto n ao quadr atico modulo p.
1.54 (Putnam 1991). Seja p um primo mpar. Quantos elementos tem
o conjunto
{x
2
| x Z/pZ} {y
2
+ 1 | y Z/pZ}?
1.55 (IMO2008). Prove que existe um n umero innito de inteiros posi-
tivos n tais que n
2
+ 1 tem um divisor primo maior do que 2n +

2n.
Captulo 2
Funcoes Multiplicativas e
as formulas de inversao de
M

obius
Neste captulo estudaremos algumas funcoes aritmeticas importan-
tes. Muitas de tais funcoes satisfazem a propriedade de serem deter-
minadas por seus valores em um subconjunto adequado dos n umeros
naturais, por exemplo os primos, ou as potencias de primo. Uma classe
importante de tais funcoes e a das chamadas fun coes multiplicativas.
Uma funcao f : N

C e dita multiplicativa se f(mn) = f(m) + f(n)


para quaisquer m, n primos entre si. No caso que a propriedade anterior
e verdadeira para quaisquer inteiros positivos m e n dizemos que f e
uma fun cao totalmente multiplicativa.
Alguns exemplos de funcoes totalmente multiplicativas s ao as funcoes
f
a
(n) = n
a
, onde a e uma constante. De fato, estas funcoes s ao as
unicas que possuem as propriedades de serem monotonas e totalmente
multiplicativas, como mostra a seguinte proposicao
Proposi cao 2.1. Seja f : N

uma fun cao totalmente multiplica-


tiva e monotona, ent ao existe R tal que f(n) = n

.
Demonstra c

ao: Trocando f por 1/f, podemos supor sem perda de


generalidade que f e crescente, e seja = log
2
f(2). Vejamos que f(n) =
n

. Para isto observemos que, aplicando f, para todo m N

temos
2
mlog
2
n
n
m
< 2
mlog
2
n+1
= 2
mlog
2
n
(f(n))
m
< 2
(mlog
2
n+1)
Assim,
2
mlog
2
n
m
f(n) < 2
(mlog
2
n+1)
m
para todo m N

.
99
100CAP

ITULO 2. FUN C

OES MULTIPLICATIVAS E AS F

ORMULAS DE INVERS

A
Mas
lim
m
mlog
2
n
m
= lim
m
(mlog
2
n + 1)
m
= log
2
n,
donde conclumos que f(n) = 2
log
2
n
= n

.
Para uma extensao desse resultado para funcoes multiplicativas veja
o exerccio 2.27
Uma funcao multiplicativa que nao e totalmente multiplicativa e por
exemplo f(n) = 2
(n)
, onde (n) e o n umero de divisores primos de n.
O seguinte teorema nos mostra uma forma usual de construir funcoes
multiplicativas.
Teorema 2.2. Se f e uma fun cao multiplicativa ent ao a fun cao
F(n) =

d|n
f(d)
e tambem multiplicativa.
Demonstra c

ao: Sejam a e b inteiros tais que mdc(a, b) = 1 entao


F(ab) =

d|ab
f(d) =

d
1
|a,d
2
|b
f(d
1
d
2
) =

d
1
|a,d
2
|b
f(d
1
)f(d
2
)
=

d
1
|a

d
2
|b
f(d
1
)f(d
2
) =

d
1
|a
f(d
1
)

d
2
|b
f(d
2
)
= F(a)F(b).
Segue que F tambem e multiplicativa.
Exemplo 2.3. Seja f : N

N tal que f(1) = 1 e para todo n > 1,


f(n) = p
1
p
k
onde {p
1
, . . . , p
k
} e o conjunto dos primos que dividem
n, isto e, f(n) e o maior inteiro livre de quadrados que divide n. Mostre
que f e uma fun cao multiplicativa e calcule

d|n
f(d) em termos da
fatoracao prima de n.
Solu c

ao: Observe que se n = ab onde mdc(a, b) = 1 e a, b > 1 , entao


a = p

1
1
p

l
l
e b = p

l+1
l+1
p

s
s
onde p
i
= p
j
para todo i = j, logo
f(a) = p
1
p
l
, f(b) = p
l+1
p
s
e
f(ab) = p
1
p
l
p
l+1
p
s
= f(a)f(b)
e portanto a funcao e multiplicativa. Para a segunda parte do problema,
pelo teorema anterior basta calcular a soma para potencias de primos.
Temos

d|p
k
f(d) = 1 +
k vezes
..
p + +p = kp + 1,
101
e portanto

d|n
f(d) = (
1
p
1
+ 1) (
s
p
s
+ 1).
Exemplo 2.4. Determine quantas solu coes distintas possui a congruen-
cia
x
2
800x 0 (mod 10!).
Solu c

ao: Primeiro observemos que m := 10! = 2


8
3
4
5
2
7, assim
para cada x solucao da equa cao inicial, existe r divisor de 10!, tal que x
e solucao do sistema de equa coes lineares
x 0 (mod r)
x 800 (mod
m
r
).
Por outro lado, para cada r|m, temos que o sistema anterior possui
solucao se, e somente se, mdc(r,
m
r
)|800. Alem disso, caso o sistema seja
sol uvel, a solucao e unica m odulo m/ mdc(r,
m
r
) (e portanto ha mdc(r,
m
r
)
solucoes m odulo m). Se denotamos por f
m
(d) o n umero de inteiros
positivos r divisores de m tais que mdc(r,
m
r
) = d, temos que o n umero
de solucoes da equa cao e

d|800
d.f
m
(d).
Para calcular f
m
(d) observe que se d|r e d|
m
r
entao d
2
|m, e neste
casso f
m
(d) = f
m/d
2(1), e no caso que d
2
m temos que f
m
(d) = 0.
Alem disso, f
k
(1) = 2
(k)
onde (k) e o n umero de primos distintos
que dividem k, pois precisamos distribuir os divisores primos de k em
dois fatores r e
k
r
sem que eles possuam fator comum. Usando o fato
anterior temos que o n umero de solucoes da equa cao inicial e

d|800
d.f
m
(d) =

d
2
|800
d.f
m/d
2(1)
=

d|20
d.2
(m/d
2
)
= 2
(m)
+ 2.2
(
m
2
2
)
+ 4.2
(
m
4
2
)
+ 5.2
(
m
5
2
)
+ 10.2
(
m
10
2
)
+ 20.2
(
m
20
2
)
= 2
4
+ 2.2
4
+ 4.2
4
+ 5.2
3
+ 10.2
3
+ 20.2
3
= 392
Nas seguintes secoes mostraremos algumas funcoes multiplicativas
importantes em aritmetica.
102CAP

ITULO 2. FUN C

OES MULTIPLICATIVAS E AS F

ORMULAS DE INVERS

A
2.1 As funcoes d, e
Para todo inteiro positivo n denimos d(n) como o n umero de divi-
sores positivos de n.

E facil calcular d(n) para valores pequenos de n.
Por exemplo d(1) = 1, d(2) = 2, d(3) = 2, d(4) = 3, d(5) = 2, d(6) = 4 e
d(7) = 2. Em particular, para todo n umero primo p, como seus unicos
divisores s ao 1 e p, temos que d(p) = 2.
Mais geralmente, para toda potencia de um primo p
k
, vemos que os
unicos n umeros que a dividem s ao as potencia de p com expoente menor
ou igual a k, isto e, 1, p, p
2
, . . . , p
k
. Assim, d(p
k
) = k + 1.
Para calcular d(n) onde n e um n umero arbitr ario precisamos do
teorema fundamental da aritmetica, isto e, todo n umero natural n maior
que 1 pode ser escrito de forma unica como produto de primos, por
exemplo 91 = 7 17, 1000 = 2
3
5
3
, 1001 = 7 11 13 etc. Em geral
todo n umero n pode ser escrito como
n = p
k
1
1
p
k
2
2
. . . p
k
s
s
,
onde p
1
, . . . , p
s
s ao n umeros primos distintos, e k
1
, . . . , k
s
s ao inteiros
positivos. Agora observemos que se um n umero m e um divisor de n
entao m nao pode ter em sua fatora cao prima primos distintos aos que
aparecem na fatora cao de n. Alem disso, o expoente de cada primo na
fatora cao de m tem que ser menor ou igual ao expoente deste primo na
fatora cao de n. Assim todo divisor de n e da forma
p
l
1
1
p
l
2
2
. . . p
l
s
s
onde 0 l
j
k
j
para todo j = 1, . . . , s. Isto quer dizer que l
1
pode
asumir qualquer dos valores 0, 1, . . . , k
1
, l
2
pode asumir qualquer dos
valores 0, 1, . . . , k
2
e assim por diante. Assim ha k
1
+1 valores possveis
para l
1
, k
2
+ 1 valores possveis para l
2
, e assim por diante, ate l
s
, que
tem k
s
+ 1 valores possveis. Pelo principio multiplicativo da contagem,
conclumos que
d(n) = (k
1
+ 1)(k
2
+ 1) (k
s
+ 1).
Alternativamente, temos d(n) =

d|n
1, e como a funcao constante igual
a 1 e multiplicativa, d e multiplicativa pelo teorema 2.2, e, como d(p
k
j
j
) =
k
j
+ 1, d(n) = d(p
k
1
1
p
k
2
2
. . . p
k
s
s
) = (k
1
+ 1)(k
2
+ 1) (k
s
+ 1).
Por exemplo, 1000 tem d(1000) = 4 4 = 16 divisores: 1, 2, 4, 5, 8,
10, 20, 25, 40, 50, 100, 125, 200, 250, 500 e 1000.

E facil ver que a funcao d, apesar de ser multiplicativa, nao e total-


mente multiplicativa. Por exemplo, d(4) = 3 = 2.2 = d(2).d(2).
2.1. AS FUN C

OES D, E 103
Exemplo 2.5. Determine todos os n umeros menores que 2012 que tem
exatamente 15 divisores.
Solu c

ao: Dado que 15 = 3 5, temos que os n umeros procurados


tem 1 ou 2 fatores primos. No caso em que somente haja um fator
primo p, este primo deve satisfazer 2
14
p
14
< 2012, o que e impossvel.
Assim os n umeros procurados tem exatamente 2 fatores primos. Isto
e, precisamos encontrar dois n umeros primos distintos p e q tais que
p
2
q
4
< 2012, portanto pq
2
<

2012 < 45 e assim q


_
44
2
< 4. Segue
que q e 2 ou 3. No caso em que q = 2, temos que p 11, donde obtemos
os n umeros 2
4
3
2
= 144, 2
4
5
2
= 400, 2
4
7
2
= 784, 2
4
11
2
= 1936, que
possuem 15 divisores. No caso que q = 3 temos p < 5, logo p = 3, e
obtemos o n umero 2
2
3
4
= 324, que tem 15 divisores.
Uma funcao da mesmanaturezaque d e a funcao soma dos divisores
de n, que denotaremos por (n). Seguindo as ideias anteriores, temos
que se n = p
k
1
1
p
k
2
2
. . . p
k
s
s
, entao a soma dos divisores de n e igual a

0l
j
k
j
1js
p
l
1
1
p
l
2
2
. . . p
l
s
s
=
_
k
1

l
1
=0
p
l
1
1
__
k
2

l
2
=0
p
l
2
2
_

_
k
s

l
s
=0
p
l
s
s
_
.
Como cada somatorio entre chaves e uma serie geometrica, conclumos
que
(n) =
p
k
1
+1
1
1
p
1
1

p
k
2
+1
2
1
p
2
1

p
k
s
+1
s
1
p
s
1
.
Em geral, se denotamos por
m
a soma das m-esimas potencias dos
divisores de n, por um processo identico se prova que

m
(n) =

d|n
d
m
=
p
(k
1
+1)m
1
1
p
m
1
1

p
(k
2
+1)m
2
1
p
m
2
1

p
(k
s
+1)m
s
1
p
m
s
1
.
Os detalhes da prova da igualdade anterior, assim como a prova do fato
de que cada uma das funcoes
m
e multiplicativa, s ao deixados como
exerccio para o leitor.
Exemplo 2.6. Determine um valor de n tal que (n) = 307.
Solu c

ao: Como 307 e primo, entao os unicos possveis valores para n


s ao as potencias de primo. Suponhamos que n = p
k
, assim
p
k+1
1
p1
= 307.
A equa cao anterior e equivalente a p(307 p
k
) = 306, assim p e um
divisor primo de 306 = 23
2
17, isto e, p pode ser 2, 3 ou 17. Observemos
que se p = 2 teramos 2
k
= 154, e no caso p = 3 obteramos 3
k
= 205,
que nao geram solucao. No caso p = 17 temos 17
k
= 289 = 17
2
, e assim
temos que n = 289 e a unica solucao do problema.
104CAP

ITULO 2. FUN C

OES MULTIPLICATIVAS E AS F

ORMULAS DE INVERS

A
Observemos que, para todo n maior que 1, (n) n + 1, e vale a
igualdade unicamente quando n e primo. No seguinte exemplo, encon-
tramos uma limitacao melhor que a anterior, e que relaciona as funcoes
d e .
Exemplo 2.7. Sejam n um n umero natural maior que 1. Mostrar que
(n) n + 1 + (d(n) 2)

n,
e que vale a igualdade se, e somente se, n e primo ou n e o quadrado de
um n umero primo.
Solu c

ao: Suponhamos que n nao e primo nem quadrado de n umero


primo, pois nestes casos se verica facilmente a igualdade, e vejamos
que neste caso se tem a desigualdade estrita. Observemos que para
todo divisor nao trivial d de n se tem que
n
d
tambem e divisor de n e
d +
n
d
2

n, onde a desigualdade e estrita se d =


n
d
. Assim, somando
sobre todos os divisores nao triviais de n temos que
2

d|n
d=1,n
d =

d|n
d=1,n
_
d +
n
d
_
>

d|n
d=1,n
2

n = 2(d(n) 2)

n.
Dividindo a desigualdade por 2, e somando n + 1 aos dois lados da
igualdade, obtemos a desigualdade desejada.
Podemos encontrar exemplos em que a soma dos divisores proprios
de um n umero (isto e, sem incluir ele mesmo), pode ser menor do que
n, maior do que n e igual a n. Os n umeros cuja soma dos divisores
proprios e n, ou, equivalentemente, com (n) = 2n s ao chamados de
n umeros perfeitos, nome que vem desde os gregos antigos. Observemos
que no tempo de Euclides ja eram conhecidos os n umeros perfeitos
6 = 2 3, 28 = 4 7, 496 = 16 31 e 8128 = 64 127
Todos estes n umeros s ao da forma 2
k1
(2
k
1) onde 2
k
1, e primo. De
fato, foi mostrado por Euler que todos os n umeros perfeitos pares s ao
desta forma como se mostra a seguir
Proposi cao 2.8. Se n um n umero par, ent ao (n) = 2n se, e somente
se, existe um inteiro k tal que M
k
:= 2
k
1 e primo e n = 2
k1
M
k
.
Os n umeros M
k
s ao chamados de N umeros de Mersenne em home-
nagem ao l osofo, te ologo e matem atico frances Marin Mersenne que
estudou estes n umeros, e conjecturou para que valores de k < 257 os
n umeros eram primos.
2.1. AS FUN C

OES D, E 105
Demonstra c

ao: Se M
p
= 2
p
1 e um n umero primo entao
(2
p1
M
p
) = (2
p1
) (M
p
) = (2
p
1)(M
p
+ 1) = 2 2
p1
M
p
.
Por outro lado, seja n = 2
k
b, com k > 0 e b mpar, um n umero perfeito
par. Temos
(n) = 2n = (2
k
)(b) donde 2
k+1
b = (2
k+1
1)(b).
Como mdc(2
k+1
1, 2
k+1
) = 1, temos que 2
k+1
1 e um divisor de b,
logo b = (2
k+1
1)c para algum inteiro mpar c e assim (b) = 2
k+1
c.
Mas 1, 2
k+1
1, c, b s ao divisores de b = (2
k+1
1)c; se c = 1 entao
2
k+1
c = (b) 1 +b +c = 1 + 2
k+1
c,
o que e contradit orio. Logo c = 1 e b = 2
k+1
1 e primo pois temos
(b) = 2
k+1
= b + 1.
Deixamos para o leitor mostrar que se M
k
e primo entao k tem que
ser primo. Na atualidade unicamente s ao conhecidos 47 valores de k pra
os quais M
k
e primo. O maior deles e k = 43112609, descoberto em 2008
(para mais informa coes ver o site http://www.mersenne.org).
Dado um n umero n, lembramos que (n) denota o n umero de n u-
meros naturais menores que n e primos relativos com n. Por exemplo,
se p e um n umero primo, entao todo n umero menor que p nao tem fator
comum com p, e assim (p) = p 1. Em geral, (p
k
) = p
k
p
k1
pois mdc(a, p
k
) = 1, se e somente se, a nao e m ultiplo de p e ha p
k1
m ultiplos de p no intervalo 1 a p
k
. Sabemos que a funcao e
multiplicativa, isto e, (nm) = (n)(m) para quaisquer m e n primos
entre si
Assim, se n = p

1
1
p

k
k
e a fatora cao de n em potencias de primos
distintos p
i
, temos que
(n) =

1ik
(p

i
i
) =

1ik
(p

i
i
p

i
1
i
) = n

1ik
_
1
1
p
i
_
.
Exemplo 2.9. Determine todas as solu coes de (n) = 2
7
.
Solu c

ao: Suponhamos que n = 2


a
m onde m = p

1
1
p

k
k
e mpar.
Temos (n) = (2
a
)(m), e (m) = p

1
1
1
p

k
1
k
(p
1
1) (p
k
1),
mas como (n) e uma potencia de 2, segue que
j
= 1 e p
j
1 e
uma potencia de 2 para todo j. Conclumos que os possveis primos na
fatora cao de m s ao 2
1
+ 1 = 3, 2
2
+ 1 = 5, 2
4
+ 1 = 17 (o proximo
primo da forma 2
n
+ 1 e 257 = 2
8
+ 1 > 2
7
+ 1). Portanto os possveis
valores de n s ao 2
8
= 256, 2
7
3 = 384, 2
6
5 = 320, 2
5
15 = 480,
2
4
17 = 272, 2
3
51 = 408, 2
2
85 = 340, 2 255 = 510 e 255.
106CAP

ITULO 2. FUN C

OES MULTIPLICATIVAS E AS F

ORMULAS DE INVERS

A
Exemplo 2.10. Mostrar que (n)
log 2
log(2n)
n para todo n > 2
Solu c

ao: Seja n = p

1
1
p

k
k
e a fatora cao de n. Pela identidade
anterior temos que
(n)
n
=

1ik
_
1
1
p
i
_

2ik+1
_
1
1
i
_
=
1
k + 1
.
Como n p
1
p
2
p
k
2
k
, assim 2n 2
k+1
e portanto
1
k+1

log 2
log(2n)
,
como queramos mostrar.
Exemplo 2.11. Encontrar todos os inteiros n para os quais (n) =
d(n).
Solu c

ao: Se p 3 e um primo, temos que


(p

) = (p 1)p
1
2(1 + 2)
1
2
_
1 + 2( 1)
_
+ 1 = d(p

),
onde a igualdade s o se da quando p = 3 e = 1. Portanto, pela
multiplicatividade das funcoes (n) e d(n), os unicos mpares que sa-
tisfazem (n) = d(n) s ao n = 1 e n = 3. Por outro lado, se > 3
temos (2

) = 2
1
> + 1 = d(2

); para = 3 obtemos as solucoes


n = 1 8 = 8 e n = 3 8 = 24.
Assim, s o nos falta resolver os casos (2n) = d(2n) (n) =
2d(n) e (4n) = d(4n) 2(n) = 3d(n) onde n e mpar. Temos
(5) = 4 = 2d(5), (15) = 8 = 2d(15) e (9) = 6 = 2d(9), donde
2 5 = 10, 2 9 = 18 e 2 15 = 30 tambem s ao solucoes da equa cao
inicial. Demonstremos agora que nao existem mais solucoes. Se n = p

e potencia de um primo mpar p entao para p = 3 e 3, ou para para


p = 5 e 2, ou para p 7, temos como acima que
(n) = p
1
(p 1) > 2 + 2 = 2d(n) >
3
2
d(n).
Por outro lado, ja sabemos que (n) d(n) para todo n mpar. Assim,
da multiplicatividade das funcoes (n) e d(n), obtemos que se n e divi-
svel por 3
3
, 5
2
ou por algum primo p 7, entao (n) > 2d(n) >
3
2
d(n)
e analisando os casos restantes obtemos apenas as solucoes apresentadas
anteriormente.
Em conclusao, as unicas solucoes de (n) = d(n) s ao 1, 3, 8, 10, 18,
24 e 30.
A seguinte proposicao relaciona as funcoes e .
Proposi cao 2.12. Seja n um n umero inteiro maior que 1. Mostrar que
(n)(n) < n
2
.
2.1. AS FUN C

OES D, E 107
Demonstra c

ao: Seja n = p
k
1
1
p
k
2
2
. . . p
k
s
s
a fatora cao em primos de n.
Observemos que
(n) =
p
k
1
+1
1
1
p
1
1

p
k
2
+1
2
1
p
2
1

p
k
s
+1
s
1
p
s
1
<
p
k
1
+1
1
p
k
2
+1
2
p
k
s
+1
s
(p
1
1)(p
2
1) (p
s
1)
=
np
1
p
2
p
s
(p
1
1)(p
2
1) (p
s
1)
=
n
2
(n)
,
como queramos mostrar.
Podemos usar a proposicao anterior para mostrar o seguinte resul-
tado sobre n umeros perfeitos e sobre os n umeros chamados de abundan-
tes, que s ao os n umeros menores que a soma dos seus divisores proprios,
isto e, com (n) > 2n.
Exemplo 2.13. Seja n um n umero mpar perfeito ou abundante. Ent ao
n tem 3 ou mais fatores primos distintos.
Solu c

ao: Pela proposicao anterior temos que


2n(n) (n)(n) < n
2
,
assim, dividindo por 2n
2
obtemos a desigualdade
(n)
n
=
_
1
1
p
1
__
1
1
p
2
_

_
1
1
p
s
_

1
2
.
Observe que se o n umero n tiver dois o menos fatores primos, entao
(n)
n

_
1
1
3
_ _
1
1
5
_
=
8
15
>
1
2
, o que e contradit orio. Portanto, n
tem tres ou mais fatores primos.
Ate o momento nao e conhecido nenhum n umero perfeito mpar. Se
conjectura que eles nao existam, mas caso existam, eles s ao maiores que
10
300
e tem no mnimo 9 fatores primos distintos.
Existem algumas condi coes necessarias para que um n umero mpar
seja perfeito (caso exista algum). O seguinte resultado e devido a Euler
Proposi cao 2.14. Seja n um n umero perfeito mpar. Ent ao existem p
um n umero primo, k e m inteiros tais que n = p
k
m
2
, onde p e k sao
congruentes a 1 modulo 4.
108CAP

ITULO 2. FUN C

OES MULTIPLICATIVAS E AS F

ORMULAS DE INVERS

A
Demonstra c

ao: Seja n = p
k
1
1
p
k
s
s
a fatora cao prima de n. Temos
2n = (n) = (p
k
1
1
) (p
k
s
s
).
Assim, somente um dos n umeros (p
k
j
j
) com 1 j s pode ser par.
Podemos supor que (p
k
1
1
) e par (mas nao divisvel por 4), e os outros
n umeros (p
k
j
j
) com 2 j s s ao mpares. Como
(p
k
j
j
) = 1 +p
j
+p
2
j
+ +p
k
j
j
k
j
+ 1 (mod 2),
temos que k
j
tem que ser par para todo j 2 assim p
k
2
2
p
k
s
s
= m
2
.
Observemos que m
2
1 (mod 4) logo (p
k
1
1
) 2 (mod 4), e portanto
k
1
e mpar. No caso que p
1
3 (mod 4) teramos que (p
k
1
1
) 0
(mod 4), o que e contradit orio, daqui que p
1
1 (mod 4) e
(p
k
1
1
) k
1
+ 1 2 (mod 4),
o que conclui a prova.
Problemas Propostos
2.1. Calcule (1001) e (1001).
2.2. Determine o n umero de divisores de 2008, 2009, 2010 e 2011.
2.3. Determine as solu coes de (n) = 2801.
2.4. Mostre que as fun coes
m
sao multiplicativas para todo m = 0.
Observe que para todo n > 1 se tem que lim
m0

m
(n) = d(n).
2.5. Determine as solu coes do sistema
_
(n) = 8784
d(n) = 12
.
2.6. Seja f uma fun cao multiplicativa tal que para todo n umero primo
p,
f(p
k
) =
_
1 se k = 2
s
para algum s N
0 caso contrario
.
Mostre que f e uma fun cao tal que f(n
2
) = f(n)
2
para todo n N, mas
n ao e totalmente multiplicativa.
2.7. Construa um exemplo de uma fun cao multiplicativa f, que n ao
seja totalmente multiplicativa, tal que f(n
k
) = f(n)
k
para todo k 10
e todo n N. Generalize o resultado anterior para k N (em lugar de
k 10), com N inteiro xo.
2.1. AS FUN C

OES D, E 109
2.8. Mostrar que para todo inteiro positivo n 5 temos que
n
d
2
(n)
4
+(n).
Para que valores de n se tem a igualdade?
2.9. Quantos n umeros com 35 divisores existem, tais que todos seus
fatores primos sejam menores que 20. Qual e o maior e o menor de tais
n umeros?
2.10. Seja n um n umero composto. Mostre que 2
n
1 tambem e com-
posto.
2.11. Mostre que um n umero n tem um n umero mpar de divisores se,
e somente se, n e um quadrado perfeito.
2.12. Determine a menor solu cao da equacao 2d(n
2
) = 17d(n).
2.13. Encontre innitos valores de n para os quais d(n) e um divisor de
n.
2.14. Denotemos por f(n) a soma dos divisores de n que sao quadrados
perfeitos. Mostre que f e uma fun cao multiplicativa, e determine uma
formula fechada para esta fun cao.
2.15. Determine todas as solu coes de (n) = 24.
2.16. Denotemos por n# o produto de todos os primos menores ou iguais
a n. Mostre que (n#) divide n! e usando este resultado mostre que a
equacao (x) = n! sempre possui solu cao x N.
2.17. Determine todos os valores de k N tais que n = k(n) possui
solu cao.
2.18. Mostre que para todo n existe um k tal que (x) = kn possui
solu cao.
2.19. Determinar todas as solu coes da equacao (n) = 2d(n).
2.20. Determinar todos os n umeros inteiros positivos n tais que
n = d(n)
2
.
2.21. Dois n umeros a e b sao amig aveis se (a) = b e (b) = a. Por
exemplo 1184 e 1210 sao amigaveis (vericar!). Encontrar outra dupla
de n umeros amigaveis.
2.22. Determine todas as solu coes de ((n)) = 2
13
3
2
5
4
.
110CAP

ITULO 2. FUN C

OES MULTIPLICATIVAS E AS F

ORMULAS DE INVERS

A
2.23. Mostre que para todo m e n inteiros vale a identidade
(mn) (mdc(m, n)) = mdc(m, n) (n) (m).
2.24. Mostre que 945 e o menor n umero abundante mpar.
2.25. Mostre que todo m ultiplo de um n umero abundante e abundante.
2.26. Determine qual e o n umero mnimo de divisores primos que um
n umero deve ter se ele e
abundante ou perfeito,
n ao divisvel por 2, nem por 3.
2.27. Seja f : N
+
R
+
uma fun cao multiplicativa e crescente.
(a) Prove que, para todo inteiro M > 1 e todo inteiro positivo n,
f(M
n+1
1) f(M
n
1)f(M) e f(M
n+1
+1) f(M
n
+1)f(M).
Conclua que
lim
n
n
_
f(M
n
) = f(M).
(b) Utilize o item anterior para M potencia de primo para concluir que
f(p
k
) = f(p)
k
para todo primo p.
(c) Conclua que f e totalmente multiplicativa, e portanto existe > 0
tal que f(n) = n

para todo inteiro positivo n.


2.2 Funcao de Mobius e F ormula de Inversao
Denimos a funcao de Mobius : N
>0
Z por
(n) =
_

_
1 se n = 1
0 se a
2
| n para algum a > 1
(1)
k
se n e produto de k primos distintos.
Facilmente se comprova que a funcao de Mobius e multiplicativa. Alem
disso
Lema 2.15. Para todo inteiro positivo n temos

d|n
(d) =
_
1 se n = 1
0 se n > 1.
2.2. FUN C

AO DE M

OBIUS E F

ORMULA DE INVERS

AO 111
Demonstra c

ao: No caso n = 1 nao temos nada para provar. Como a


funcao

d|n
(d) e multiplicativa pelo teorema 2.2, basta mostra o lema
para n = p
k
onde p e um n umero primo. De fato,

d|p
k
(d) =
k

j=0
(p
j
) = 1 1 = 0
como queramos demonstrar.
Teorema 2.16 (F ormula de inversao de Mobius). Seja f(n) uma fun cao
sobre os inteiros positivos e F(n) =

d|n
f(d), ent ao para todo n inteiro
positivo,
f(n) =

d|n
(d)F
_
n
d
_
.
Demonstra c

ao: Vejamos que

d|n
(d)F
_
n
d
_
=

d|n
(d)

d
1
|
n
d
f(d
1
)
=

d|n

d
1
|
n
d
(d)f(d
1
)
=

d
1
|n

d|
n
d
1
(d)f(d
1
)
=

d
1
|n
f(d
1
)

d|
n
d
1
(d) = f(n)(1) = f(n),
como queramos demonstrar.
Exemplo 2.17. Como vimos no lema 1.36,

d|n
(d) = n, n N.
Portanto, pelo teorema anterior, para todo inteiro positivo n, (n) =

d|n
(d)
n
d
. Da segue que

d|n
(d)
d
=
(n)
n
.
Exemplo 2.18. Uma pulseira e formada por pedras coloridas, de mesmo
tamanho, pregadas em volta de um crculo de modo a carem igualmente
espacadas. Duas pulseiras sao consideradas iguais se, e so se, suas con-
guracoes de pedras coincidem por uma rotacao. Se ha pedras disponveis
de k 1 cores distintas, mostre que o n umero de pulseiras diferentes
possveis com n pedras e dado pela expressao
1
n

d|n
(d) k
n/d
.
112CAP

ITULO 2. FUN C

OES MULTIPLICATIVAS E AS F

ORMULAS DE INVERS

A
Solu c

ao: No que segue o n umero k de cores de pedras estara sempre


xo. A cada pulseira podemos associar um perodo, que e denido como o
menor divisor positivo d de n tal que a sequencia das n pedras da pulseira
e obtida a partir de uma sequencia de d pedras repetida n/d vezes.
Se o problema fosse contar pulseiras xas, sem indenticar pulseiras
que coincidem por uma rota cao, a resposta seria claramente k
n
. Ao
considerarmos as n rota coes de uma pulseira de perodo d, obtemos d
pulseiras xas distintas (i.e., distintas como pulseiras xas, mas iguais a
menos de rota cao). Dizemos que uma pulseira com n pedras e primitiva
se seu perodo e n. Se denotarmos por g(n) o n umero de pulseiras
primitivas com n pedras, temos que, para cada divisor d de n, o n umero
de pulseiras com n pedras e perodo d e g(d) (se o perodo e d, podemos
tomar d pedras consecutivas e unir as pontas criando uma pulseira com
d pedras, que sera primitiva), e elas dao origem a d.g(d) pulseiras xas.
Assim, temos, para todo inteiro positivo n,

d|n
d.g(d) = k
n
, donde,
pelo teorema anterior, n.g(n) =

d|n
(d)k
n/d
.
O n umero de pulseiras que queremos contar, como no enunciado, e

d|n
g(d) =

d|n
1
d

s|d
(s)k
d/s
.
Fazendo t = d/s na ultima expressao, temos d = st, e d | n equivale a
s | n/t. Assim, podemos escrever a ultima expressao como

t|n

s|n/t
1
st
(s)k
t
=

t|n
k
t
t

s|n/t
(s)
s
,
que, pelo exemplo anterior, e igual a

t|n
k
t
t

t
n
(n/t) =

t|n
k
t
n
(n/t),
que, por sua vez (fazendo r = n/t), e igual a
1
n

r|n
(r) k
n/r
.
Agora, observemos que para todo n umero natural m, f e F denidas
como antes,
m

n=1
F(n) =
m

n=1

d|n
f(d) =
m

d=1

d|n
1nm
f(d)
Como f(d) e somado
_
m
d
_
vezes, entao
m

n=1
F(n) =
m

d=1
f(d)
_
m
d
_
.
No caso particular em que f(n) = (n) temos que F(n) = n pelo
lema 1.36 e assim
m(m+ 1)
2
=
m

n=1
(n)
_
m
n
_
.
2.2. FUN C

AO DE M

OBIUS E F

ORMULA DE INVERS

AO 113
Se f(n) = (n), entao F(n) = 0 se n > 1 e F(1) = 1 pelo lema 2.15,
portanto
1 =
m

n=1
(n)
_
m
n
_
.
A igualdade anterior nos permite resolver o seguinte
Exemplo 2.19. Demonstrar que, para todo inteiro m > 1,

k=1
(k)
k

< 1.
Solu c

ao: Como 1 < (k)


__
m
k
_

m
k
_
< 1 e
_
m
k
_

m
k
= 0 quando
k = 1, m, entao

k=1
(k)
_
m
k
_
m
m

k=1
(k)
k

< m1
Usando a identidade acima provada temos que

1 m
m

k=1
(k)
k

< m1,
logo

m
k=1
(k)
k

< m e simplicando m obtemos o que queramos


demonstrar.

E conhecido (Mangoldt 1897) que se m tende para innito,
entao a soma anterior converge para 0.
Teorema 2.20 (Segunda formula de inversao de Mobius). Sejam f, g
fun coes reais com domnio (0, +) tais que
g(x) =

k=1
f
_
x
k
_
para todo x, ent ao
f(x) =

k=1
(k)g
_
x
k
_
.
Demonstra c

ao: Observemos que


f(x) =

k=1
(k)
_

r=1
f
_
x
kr
__
=

m=1
_

k|m
(k)
_
f
_
x
m
_
= f(x),
como queramos demonstrar.
114CAP

ITULO 2. FUN C

OES MULTIPLICATIVAS E AS F

ORMULAS DE INVERS

A
A seguinte e uma das formulacoes da famosa hip otese de Riemann,
um dos problemas em aberto mais importantes da Matem atica. O Clay
Mathematics Institute oferece um premio de 1 milh ao de dolares para
a a primeira demonstra cao da Hipotese de Riemann (ver a p agina web
http://www.claymath.org/millennium/).
Conjetura 2.21 (Hipotese de Riemann). Se > 1/2, ent ao
lim
n
1
n

m=1
(m) = 0.
Podemos reenunciar esta conjectura assim: seja f : (0, +) R
denida por
_
f(t) = 0 se t < 1

k=1
f(t/k) = 1 se t 1.
Entao, para todo > 1/2,
lim
t
f(t)
t

= 0.
De fato, pela segunda formula de inversao de Mobius, temos
f(t) =
t

m=1
(m).
Problemas Propostos
2.28. Encontre formulas fechadas para as somas

r|n
(r)d(n/r)

d|n
(d)(n/d)

d|n
(d)
m
(n/d)
2.29. Seja f uma fun cao multiplicativa e n ao identicamente nula e n =
p

1
1
p

k
k
. Mostre que

d|n
(d)f(d) =
k

j=1
(1 f(p
j
)).
2.2. FUN C

AO DE M

OBIUS E F

ORMULA DE INVERS

AO 115
2.30. Encontre formulas fechadas para as somas

r|n
(r)d(r)

d|n
(d)(d)

d|n
(d)(d)

d|n
(d)
d
.
2.31. Seja r o n umero de fatores primos diferentes de n, demonstrar
que

d|n
|(d)| = 2
r
.
2.32. Seja n um inteiro positivo que n ao e primo e tal que (n) | n1.
Demonstrar que n possui ao menos quatro fatores primos distintos.
2.33. Dados dois n umeros reais e tais que 0 < 1, demons-
trar que existe um n umero natural m tal que
<
(m)
m
< .
2.34. Seja m um inteiro positivo. Dizemos que um inteiro m 1 e
superabundante se
k {1, 2, . . . , m1}
(m)
m
>
(k)
k
.
Demonstrar que existe um n umero innito de n umeros superabundantes.
2.35. Demonstrar que
(n)
d(n)

n.
2.36. Encontrar todos os valores de n para os quais (n) | n.
2.37. Demonstrar que m | (mn 1) para todo n se, e so se, m = 2, 3,
4, 6, 8, 12 ou 24.
2.38. Demonstrar que
(n!)
n!
> 1 +
1
2
+ +
1
n
.
116CAP

ITULO 2. FUN C

OES MULTIPLICATIVAS E AS F

ORMULAS DE INVERS

A
2.39. Demonstrar que existem innitos n umeros naturais n para os
quais (x) = n n ao tem solu cao.
2.40. Demonstrar que para todo m > 1

k=1
(k)
k

<
2
3
.
2.41. Encontrar todos os inteiros positivos n tais que
n = d
2
6
+d
2
7
1,
onde 1 = d
1
< d
2
< < d
k
= n sao todos os divisores positivos do
n umero n.
2.42 (IMO1998). Para cada inteiro positivo n, d(n) denota o n umero de
divisores de n. Determine todos os inteiros positivos k tais que d(n
2
) =
kd(n) para algum n.
2.43. Se n e composto, mostre que (n) n

n.
2.44. Mostrar que (n) +(n) 2n para todo inteiro positivo n.
2.45. Seja f : (0, +) R tal que f(x) = 0 se x (0, 1) e f(x) =
x

kx
(k)
k
, x 1. Prove que

k1
f(x/k) = x, x 1.
2.46. Demonstrar que
(n!)
n!
> 1 +
1
2
+ +
1
n
.
2.47. Dadas duas fun coes f, g : N
>0
C, denimos o produto de Diri-
chlet (ou convolu cao de Dirichlet) f g : N
>0
C de f e g por
f g(n)
def
=

d|n
f(d)g
_
n
d
_
=

d
1
d
2
=n
f(d
1
)g(d
2
).
(a) Prove que, se s R (ou s C) e as series

n1
f(n)
n
s
e

n1
g(n)
n
s
convergem absolutamente ent ao

n1
f(n)
n
s

n1
g(n)
n
s
=

n1
f g(n)
n
s
.
(b) Prove que, para quaisquer fun coes f, g, h : N
>0
C, temos f g =
g f e f (g h) = (f g) h (isto e, o produto de Dirichlet
e comutativo e associativo), e que a fun cao I : N
>0
C dada
por I(n) =
_
1 se n = 1
0 se n > 1
e o elemento neutro do produto , i.e.,
I f = f I = f, f : N
>0
C.
2.2. FUN C

AO DE M

OBIUS E F

ORMULA DE INVERS

AO 117
(c) Prove que se f e g sao multiplicativas ent ao f g e multiplicativa.
(d) Prove que, se f : N
>0
C e tal que f(1) = 0, ent ao existe uma
unica fun cao f
(1)
: N
>0
C tal que f f
(1)
= f
(1)
f = I, a
qual e dada recursivamente por f
(1)
(1) = 1/f(1) e, para n > 1,
f
(1)
(n) =
1
f(1)

d|n,d<n
f
_
n
d
_
f
(1)
(d).
(e) Prove que, se f e multiplicativa, ent ao a fun cao f
(1)
denida no
item anterior tambem e multiplicativa.
Captulo 3
Frac oes Contnuas
A teoria de fracoes contnuas e um dos mais belos assuntos da Ma-
tematica elementar, sendo ainda hoje tema de pesquisa.
Nas inclus oes N Z Q R, a passagem de Q para R e sem d uvida
a mais complicada conceitualmente e a representa cao de um n umero real
esta diretamente ligada `a propria nocao de n umero real.
De fato, o conceito de n umero natural e quase um conceito primitivo.
Ja um n umero inteiro e um n umero natural com um sinal que pode ser
+ ou , e um n umero racional e a razao entre um n umero inteiro e
um natural nao nulo. Por outro lado, dizer o que e um n umero real e
tarefa bem mais complicada, mas ha coisas que podemos dizer sobre eles.
Uma propriedade essencial de R e que todo n umero real pode ser bem
aproximado por n umeros racionais. Efetivamente, dado x R, existe
k = x Z tal que 0 x k < 1. Podemos escrever a representa cao
decimal de
x k = 0, a
1
a
2
. . . a
n
. . . , a
i
{0, 1, . . . , 9},
o que signica que se r
n
= a
n
+ 10 a
n1
+ 100 a
n2
+ + 10
n1
a
1
,
entao
r
n
10
n
x k <
r
n
+1
10
n
, e portanto k +
r
n
10
n
e uma boa aproximacao
racional de x, no sentido de que o erro

x
_
k +
r
n
10
n
_

e menor do que
1
10
n
, que e um n umero bem pequeno se n for grande. A representa cao
decimal de um n umero real fornece pois uma sequencia de aproximacoes
por racionais cujos denominadores s ao potencias de 10.
Dado qualquer x R e q natural nao nulo existe p Z tal que
p
q
x <
p+1
q
(basta tomar p = qx), e portanto

x
p
q

<
1
q
e

x
p+1
q


1
q
. Em particular ha aproximacoes de x por racionais com
denominador q com erro menor do que
1
q
. A representa cao decimal de x
equivale a dar essas aproximacoes para os denominadores q que s ao po-
tencias de 10, e tem meritos como sua praticidade para efetuar calculos
119
que a fazem a mais popular das representa coes dos n umeros reais. Por
outro lado, envolve a escolha arbitr aria da base 10, e oculta frequente-
mente aproximacoes racionais de x muito mais ecientes do que as que
exibe. Por exemplo, = 3, 141592653589793... e excepcionalmente bem
aproximado por
22
7
= 3, 142857142857... e
355
113
= 3, 14159292035398....
De fato,


22
7

<
1
700
<


314
100


355
113

<
1
3000000
<


3141592
1000000

mostram que
22
7
e
355
113
s ao melhores aproximacoes de que aproximacoes
decimais com denominadores muito maiores.
O objetivo desta secao e apresentar uma outra maneira de represen-
tar n umeros reais, a representa cao por fracoes contnuas, que sempre
fornece aproximacoes racionais surpreendentemente boas, e de fato for-
nece todas as aproximacoes excepcionalmente boas, alem de ser natural
e conceitualmente simples.
Denimos recursivamente

0
= x, a
n
=
n

e, se
n
/ Z,
n+1
=
1

n
a
n
para todo n N.
Se, para algum n,
n
= a
n
temos
x =
0
= [a
0
; a
1
, a
2
, . . . , a
n
]
def
= a
0
+
1
a
1
+
1
a
2
+ .
.
.
+
1
a
n
.
Se nao denotamos
x = [a
0
; a
1
, a
2
, . . . ]
def
= a
0
+
1
a
1
+
1
a
2
+ .
.
.
.
O sentido dessa ultima nota cao cara claro mais tarde. A representa cao
acima se chama representa cao por fracoes contnuas de x.
120 CAP

ITULO 3. FRA C

OES CONT

INUAS
A gura da uma interpretacao geometrica para a representa cao de
um n umero por fracoes contnuas. Enchemos um ret angulo 1 x com
quadrados de formagulosa, isto e, sempre colocando o maior quadrado
possvel dentro do espaco ainda livre. Os coecientes a
0
, a
1
, a
2
, . . . indi-
cam o n umero de quadrados de cada tamanho. Na gura, se os lados do
retangulo s ao c < d entao
d/c = [1; 2, 2, 1, ...]
pois temos a
0
= 1 quadrado grande, a
1
= 2 quadrados menores, a
2
= 2
quadrados ainda menores, a
3
= 1 quadrados ainda ainda menores, e um
n umero grande nao desenhado de quadrados ainda ainda ainda menores
(a
4
e grande). Deixamos a vericacao de que esta descricao geometrica
corresponde `a descricao algebrica acima a cargo do leitor.
Note que, se a representa cao por fra coes contnuas de x for nita
entao x e claramente racional.
Reciprocamente, se x Q, sua representa cao sera nita, e seus co-
ecientes a
n
vem do algoritmo de Euclides: se x = p/q (com q > 0)
temos
p = a
0
q +r
1
0 r
1
< q
q = a
1
r
1
+r
2
0 r
2
< r
1
r
1
= a
2
r
2
+r
3
0 r
3
< r
2
.
.
.
.
.
.
r
n1
= a
n
r
n
Temos entao
x = p/q = a
0
+r
1
/q = a
0
+
1
a
1
+r
2
/r
1
= a
0
+
1
a
1
+
1
a
2
+r
3
/r
2
121
= = a
0
+
1
a
1
+
1
a
2
+ .
.
.
+
1
a
n
= [a
0
; a
1
, a
2
, . . . , a
n
].
Isso ja e uma vantagem da representa cao por fra coes contnuas (alem
de nao depender de escolhas articiais de base), pois o reconhecimento
de racionais e mais simples que na representa cao decimal.
A representa cao decimal de n umeros reais esta intimamente ligada
`a funcao f : [0, 1) [0, 1) dada por f(x) = {10x} = 10x 10x, mais
precisamente, `a dinamica da funcao f. Por din amica da funcao f que-
remos dizer o estudo de suas composicoes sucessivas: para cada ponto
x [0, 1), estamos interessados na sequencia x, f(x), f(f(x)), [0, 1),
cujos termos s ao os chamados iterados sucessivos da f. De fato, se
x [0, 1) tem representa cao decimal 0, a
1
a
2
a
3
. . . , entao a
1
= 10x e
f(x) = 0, a
2
a
3
a
4
. . . . Assim, denindo f
1
= f e f
n+1
= f f
n
, temos
f
n
(x) = 0, a
n+1
a
n+2
a
n+3
. . . para todo n 1. Assim, por exemplo, se
x = 1/3 = 0, 333 . . . , temos f(x) = 0, 333 = x (nesse caso, dize-
mos que x = 1/3 e um ponto xo de f); se x = 4/33 = 0, 121212 . . . ,
temos f(x) = 0, 212121 . . . e f(f(x)) = 0, 121212 = x (nesse caso
dizemos que x = 4/33 e um ponto peri odico de perodo 2 de f) e, se
x [0, 1] e irracional, os seus iterados por f serao todos distintos, pois
sua representa cao decimal nao sera peri odica a partir de nenhum dgito.
Ja a representa cao em fra coes contnuas esta intimamente ligada
`a din amica da funcao g : (0, 1) [0, 1), dada por g(x) =
_
1
x
_
=
1
x

_
1
x
_
, tambem conhecida como transforma cao de Gauss: se =
[0; a
1
, a
2
, a
3
, . . . ] (0, 1), entao a
1
=
1

e g() = [0; a
2
, a
3
, a
4
, . . . ].
Assim, denindo, como antes g
1
= g e g
n+1
= g g
n
para todo n 1,
temos g
n
() = [0; a
n+1
, a
n+2
, a
n+3
, . . . ], para todo n 1.
Mais informa coes sobre a rela cao entre fra coes contnuas e a din amica
da transformacao de Gauss pode ser encontrada em [6].
Representamos abaixo os gr acos de f(x) = {10x} e g(x) = {
1
x
}.
1
y = f(x) = {10x}
y = g(x) =
_
1
x
_
122 CAP

ITULO 3. FRA C

OES CONT

INUAS
Seja x = [a
0
; a
1
, a
2
, . . . ]. Sejam p
n
Z, q
n
N
>0
primos entre si
tais que
p
n
q
n
= [a
0
; a
1
, a
2
, . . . , a
n
], n 0. Esta fra cao
p
n
q
n
e chamada de
n-esima reduzida ou convergente da fra cao contnua de x. O seguinte
resultado sera fundamental no que seguir a.
Proposi cao 3.1. Dada uma sequencia (nita ou innita) t
0
, t
1
, t
2
,
R tal que t
k
> 0, para todo k 1, denimos sequencias (x
m
) e (y
m
) por
x
0
= t
0
, y
0
= 1, x
1
= t
0
t
1
+ 1, y
1
= t
1
, x
m+2
= t
m+2
x
m+1
+ x
m
,
y
m+2
= t
m+2
y
m+1
+y
m
, para todo m 0. Temos ent ao
[t
0
; t
1
, t
2
, . . . , t
n
] = t
0
+
1
t
1
+
1
t
2
+ .
.
.
+
1
t
n
=
x
n
y
n
, n 0.
Alem disso, x
n+1
y
n
x
n
y
n+1
= (1)
n
, para todo n 0.
Demonstra c

ao: A prova sera por indu cao em n. Para n = 0 temos


[t
0
] = t
0
= t
0
/1 = x
0
/y
0
. Para n = 1, temos [t
0
; t
1
] = t
0
+ 1/t
1
=
t
0
t
1
+1
t
1
= x
1
/y
1
e, para n = 2, temos
[t
0
; t
1
, t
2
] = t
0
+
1
t
1
+ 1/t
2
= t
0
+
t
2
t
1
t
2
+ 1
=
t
0
t
1
t
2
+t
0
+t
2
t
1
t
2
+ 1
=
t
2
(t
0
t
1
+ 1) +t
0
t
2
t
1
+ 1
=
t
2
x
1
+x
0
t
2
y
1
+y
0
=
x
2
y
2
.
Suponha que a armacao seja valida para n. Para n + 1 em lugar de n
temos
[t
0
; t
1
, t
2
, . . . , t
n
, t
n+1
] = [t
0
; t
1
, t
2
, . . . , t
n
+
1
t
n+1
]
=
_
t
n
+
1
t
n+1
_
x
n1
+x
n2
_
t
n
+
1
t
n+1
_
y
n1
+y
n2
=
t
n+1
(t
n
x
n1
+x
n2
) +x
n1
t
n+1
(t
n
y
n1
+y
n2
) +y
n1
=
t
n+1
x
n
+x
n1
t
n+1
y
n
+y
n1
=
x
n+1
y
n+1

Vamos agora mostrar, por indu cao, a segunda armacao. Temos


x
1
y
0
x
0
y
1
= (t
0
t
1
+ 1) t
0
t
1
= 1 = (1)
0
e, se x
n+1
y
n
x
n
y
n+1
= (1)
n
para algum valor de n, entao
x
n+2
y
n+1
x
n+1
y
n+2
= (t
n+2
x
n+1
+x
n
)y
n+1
(t
n+2
y
n+1
+y
n
)x
n+1
= (x
n+1
y
n
x
n
y
n+1
) = (1)
n
= (1)
n+1
.
123
Nos proximos resultados, x = [a
0
; a
1
, a
2
, a
3
, . . . ] sera um n umero
real, e (
p
n
q
n
)
nN
,
p
n
q
n
= [a
0
; a
1
, a
2
, . . . , a
n
] sera a sequencia de reduzidas da
fra cao contnua de x.
Corolario 3.2. As sequencias (p
n
) e (q
n
) satisfazem as recorrencias
p
n+2
= a
n+2
p
n+1
+p
n
e q
n+2
= a
n+2
q
n+1
+q
n
para todo n 0, com p
0
= a
0
, p
1
= a
0
a
1
+ 1, q
0
= 1 e q
1
= a
1
. Alem
disso,
p
n+1
q
n
p
n
q
n+1
= (1)
n
para todo n 0.
Demonstra c

ao: As sequencias (p
n
) e (q
n
) denidas pelas recorrencias
acima satisfazem, pela proposicao anterior, as igualdades
p
n
q
n
= [a
0
; a
1
, a
2
, . . . , a
n
] e p
n+1
q
n
p
n
q
n+1
= (1)
n
, n 0.
Como p
n+1
q
n
p
n
q
n+1
= (1)
n
, para todo n N, temos que os p
n
, q
n
dados pelas recorrencias acima s ao primos entre si. Alem disso, tambem
segue da recorrencia que q
n
> 0, n 0. Esses fatos implicam que
(
p
n
q
n
)
nN
e a sequencia de reduzidas da fra cao contnua de x.
Corolario 3.3. Temos, para todo n N,
x =

n
p
n1
+p
n2

n
q
n1
+q
n2
e
n
=
p
n2
q
n2
x
q
n1
x p
n1
Demonstra c

ao: A primeira igualdade segue da proposicao anterior


pois x = [a
0
; a
1
, a
2
, . . . , a
n1
,
n
] e a segunda e consequencia direta da
primeira.
Proposi cao 3.4. Temos
x
p
n
q
n
=
(1)
n
(
n+1
+
n+1
)q
2
n
onde

n+1
=
q
n1
q
n
= [0; a
n
, a
n1
, a
n2
, . . . , a
1
].
Em particular,
1
(a
n+1
+ 2)q
2
n
<

x
p
n
q
n

=
1
(
n+1
+
n+1
)q
2
n
<
1
a
n+1
q
2
n
.
124 CAP

ITULO 3. FRA C

OES CONT

INUAS
Demonstra c

ao: Pelo corolario anterior temos


x
p
n
q
n
=

n+1
p
n
+p
n1

n+1
q
n
+q
n1

p
n
q
n
=
p
n1
q
n
p
n
q
n1
(
n+1
q
n
+q
n1
)q
n
=
(p
n
q
n1
p
n1
q
n
)
(
n+1
q
n
+q
n1
)q
n
=
(1)
n1
(
n+1
q
n
+q
n1
)q
n
=
(1)
n
(
n+1
q
n
+q
n1
)q
n
=
(1)
n
(
n+1
+q
n1
/q
n
)q
2
n
=
(1)
n
(
n+1
+
n+1
)q
2
n
.
Em particular,

x
p
n
q
n

=
1
(
n+1
+
n+1
)q
2
n
,
e, como
n+1
= a
n+1
e 0 <
n+1
< 1, segue que a
n+1
<
n+1
+
n+1
<
a
n+1
+ 2, o que implica a ultima armacao.
A expans ao de
n+1
como fra cao contnua segue de
q
n1
q
n
=
q
n1
a
n
q
n1
+q
n2
=
q
n1
q
n
=
1
a
n
+
q
n2
q
n1
aplicado recursivamente.
Observa cao 3.5. Como lim
n
q
n
= + (pois (q
n
) e estritamente
crescente), segue desta proposicao que
lim
n
p
n
q
n
= x,
o que permite recuperar x a partir de a
0
, a
1
, a
2
, . . . , e da sentido `a
igualdade x = [a
0
; a
1
, a
2
, . . . ] quando a fracao contnua de x e innita
(i.e., quando x e irracional).
Observa cao 3.6. A proposicao anterior implica que, para todo irra-
cional, a desigualdade | p/q| < 1/q
2
tem innitas solu coes racionais
p/q. Este fato e conhecido como o Teorema de Dirichlet.

E interessante notar que, se = r/s Q, a desigualdade |p/q| <


1/q
2
tem apenas um n umero nito de solu coes racionais p/q. De fato,
|r/s p/q| < 1/q
2
equivale a |qr ps| < s/q, o que implica que q s.
A seguinte proposicao mostra que os convergentes pares formam uma
sequencia crescente, e que os convergentes mpares formam uma sequen-
cia decrescente. Alem disso todos os convergentes mpares s ao maiores
do que todos os convergentes pares.
Proposi cao 3.7. Para todo k 0, temos
p
2k
q
2k

p
2k+2
q
2k+2
x
p
2k+3
q
2k+3

p
2k+1
q
2k+1
.
125
Demonstra c

ao: O resultado segue dos seguintes fatos gerais. Para


todo n 0, temos que
p
n+2
q
n+2

p
n
q
n
=
a
n+2
p
n+1
+p
n
a
n+2
q
n+1
+q
n

p
n
q
n
=
a
n+2
(p
n+1
q
n
p
n
q
n+1
)
q
n
(a
n+2
q
n+1
+q
n
)
=
(1)
n
a
n+2
q
n+2
q
n
e positivo para n par e negativo para n mpar. Alem disso, para todo
n 0, temos que x
p
n
q
n
=
(1)
n
(
n+1
q
n
+q
n1
)q
n
e positivo para n par e
negativo para n mpar.
Proposi cao 3.8. Sejam a
0
, a
1
, . . . , a
n
inteiros com a
k
> 0, k 1,
e seja (p
k
/q
k
)
k0
a sequencia de reduzidas da fracao contnua
[a
0
; a
1
, a
2
, . . . , a
n
]. Ent ao o conjunto dos n umeros reais cuja representa-
cao por fracoes contnuas come ca com a
0
, a
1
, . . . , a
n
e o intervalo
I(a
0
, a
1
, . . . , a
n
) =
_
p
n
q
n
_
{[a
0
, a
1
, . . . , a
n
, ], > 1}
=
_
_
_
_
p
n
q
n
,
p
n
+p
n1
q
n
+q
n1
_
se n e par
_
p
n
+p
n1
q
n
+q
n1
,
p
n
q
n
_
se n e mpar.
Alem disso, a fun cao G : (1, +) I(a
0
, a
1
, . . . , a
n
) dada por G() =
[a
0
; a
1
, a
2
, . . . , a
n
, ] e monotona, sendo crescente para n mpar e decres-
cente para n par.
Demonstra c

ao:

E suciente notar que, como na prova do corolario
anterior, G() = [a
0
; a
1
, a
2
, . . . , a
n
, ] =
p
n
+p
n1
q
n
+q
n1
=
p
n
q
n
+
(1)
n
(q
n
+q
n1
)q
n
,
e portanto G e crescente para n mpar e decrescente para n par. Assim,
como G(1) =
p
n
+p
n1
q
n
+q
n1
e lim
+
G() =
p
n
q
n
, temos
G((1, +)) =
_
(
p
n
q
n
,
p
n
+p
n1
q
n
+q
n1
) se n e par
(
p
n
+p
n1
q
n
+q
n1
,
p
n
q
n
) se n e mpar.
Portanto,
I(a
0
, a
1
, . . . , a
n
) =
_
p
n
q
n
_
{[a
0
, a
1
, . . . , a
n
, ], > 1}
=
_
p
n
q
n
_
G((1, +))
=
_
_
_
_
p
n
q
n
,
p
n
+p
n1
q
n
+q
n1
_
se n e par
_
p
n
+p
n1
q
n
+q
n1
,
p
n
q
n
_
se n e mpar.
126 CAP

ITULO 3. FRA C

OES CONT

INUAS
Proposi cao 3.9. Dados inteiros a
0
, a
1
, a
2
, . . . , com a
k
> 0, k 1,
existe um unico n umero real (que e irracional) cuja representa cao por
fracoes contnuas e [a
0
; a
1
, a
2
, . . . ].
Demonstra c

ao: Considere as sequencias (p


n
) e (q
n
) denidas pelas
recorrencias
p
n+2
= a
n+2
p
n+1
+p
n
e q
n+2
= a
n+2
q
n+1
+q
n
para todo n 0, com p
0
= a
0
, p
1
= a
0
a
1
+ 1, q
0
= 1 e q
1
= a
1
. Temos,
como na proposicao 3.7,
p
2k
q
2k

p
2k+2
q
2k+2

p
2k+3
q
2k+3

p
2k+1
q
2k+1
, k 0.
Assim, considerando os intervalos fechados
I
k
=
_
p
2k
q
2k
,
p
2k+1
q
2k+1
_
,
temos I
k+1
I
k
, k 0, e portanto, como
|I
k
| =
p
2k+1
q
2k+1

p
2k
q
2k
=
p
2k+1
q
2k
p
2k
q
2k+1
q
2k+1
q
2k
=
(1)
2k
q
2k+1
q
2k
=
1
q
2k+1
q
2k
tende a 0 quando k tende a innito, existe R tal que

k0
I
k
= {}.
Como, para todo k 0,
[a
0
; a
1
, a
2
, . . . , a
2k
] =
p
2k
q
2k

p
2k+1
q
2k+1
= [a
0
; a
1
, a
2
, . . . , a
2k
, a
2k+1
]
e, da proposicao anterior, [a
0
; a
1
, a
2
, . . . , a
2k
] e [a
0
; a
1
, a
2
, . . . , a
2k
, a
2k+1
]
pertencem a I(a
0
; a
1
, a
2
, . . . , a
2k
), que e um intervalo, segue que
I(a
0
; a
1
, a
2
, . . . , a
2k
), e portanto a fra cao contnua de comeca
com a
0
, a
1
, . . . , a
2k
, para todo k 0, donde a representa cao por fra coes
contnuas de e [a
0
; a
1
, a
2
, . . . ].
Note que, como a representa cao por fra coes contnuas de e innita,
e irracional.
Exemplo 3.10. Temos
= [3; 7, 15, 1, 292, 1, 1, 1, 2, 1, 3, 1, 14, 2, 1, . . . ], portanto
p
0
q
0
= 3,
p
1
q
1
=
22
7
,
p
2
q
2
=
333
106
,
p
3
q
3
=
355
113
. . .
3.1. REDUZIDAS E BOAS APROXIMA C

OES 127
e = [2; 1, 2, 1, 1, 4, 1, 1, 6, 1, 1, 8, . . . , 1, 1, 2n, . . . ]

2 = [1; 2, 2, 2, . . . ] pois

2 = 1 +
1

2 + 1
= 1 +
1
2 +
1

2 + 1
= 1 +
1
2 +
1
2 +
1

2 + 1
=

1+

5
2
= [1; 1, 1, 1, . . . ] pois
1 +

5
2
= 1 +
1
1+

5
2
= 1 +
1
1 +
1
1+

5
2
=
Isto prova em particular que

2 e
1+

5
2
sao irracionais, pois suas fracoes
contnuas sao innitas. Da segue tambem que

2 1 = [0; 2, 2, 2 . . . ] e

51
2
= [0; 1, 1, 1, . . . ] sao pontos xos da transforma cao de Gauss g.
3.1 Reduzidas e Boas Aproximacoes
Teorema 3.11. Temos, para todo n N,

x
p
n
q
n

1
q
n
q
n+1
<
1
q
2
n
Alem disso,

x
p
n
q
n

<
1
2q
2
n
ou

x
p
n+1
q
n+1

<
1
2q
2
n+1
.
Demonstra c

ao: O n umero x sempre pertence ao segmento de extre-


mos
p
n
q
n
e
p
n+1
q
n+1
cujo comprimento e

p
n+1
q
n+1

p
n
q
n

(1)
n
q
n
q
n+1

=
1
q
n
q
n+1
=

x
p
n
q
n

1
q
n
q
n+1
<
1
q
2
n

Alem disso, se

x
p
n
q
n

1
2q
2
n
e

x
p
n+1
q
n+1

1
2q
2
n+1
,
entao
1
q
n
q
n+1
=

x
p
n
q
n

x
p
n+1
q
n+1

1
2q
2
n
+
1
2q
2
n+1
= q
n+1
= q
n
,
absurdo.
128 CAP

ITULO 3. FRA C

OES CONT

INUAS
Observa cao 3.12. De fato

x
p
n
q
n

<
1
q
n
q
n+1
<
1
a
n+1
q
2
n
. Quanto maior
for a
n+1
melhor ser a a aproximacao
p
n
q
n
de x.
Teorema 3.13 (Hurwitz, Markov). Para todo irracional e todo inteiro
n 1, temos


p
q

<
1

5q
2
para pelo menos um racional
p
q

_
p
n1
q
n1
,
p
n
q
n
,
p
n+1
q
n+1
_
.
Em particular, a desigualdade acima tem innitas solu coes racionais
p/q.
Demonstra c

ao: Suponha que o teorema seja falso. Entao, pela pro-


posicao 3.4, existe irracional, n 1 com
n
+
n

5,
n+1
+
n+1

5 e
n+2
+
n+2

5. Devemos portanto ter a


n+1
= a
n+2
= 1 ja
que claramente a
k
2 para k = n, n + 1, n + 2 e se algum a
k
= 2 com
k = n + 1, n + 2, teramos a
k
+
k
2 +
1
3
>

5, absurdo.
Sejam x = 1/
n+2
e y =
n+1
. As desigualdades acima se traduzem
em
1
1 +x
+
1
y

5, 1 +x +y

5 e
1
x
+
1
1 +y

5.
Temos
1 +x +y

5 = 1 +x

5 y
=
1
1 +x
+
1
y

1

5 y
+
1
y
=

5
y(

5 y)
e portanto y(

5 y) 1 = y

51
2
. Por outro lado temos
x

5 1 y =
1
x
+
1
1 +y

1

5 1 y
+
1
1 +y
=

5
(1 +y)(

5 1 y)
e portanto (1 +y)(

5 1 y) 1 = y

51
2
, e portanto devemos
ter y =

51
2
, o que e absurdo pois y =
n+1
=
q
n1
q
n
Q.
3.2. BOAS APROXIMA C

OES S

AO REDUZIDAS 129
Observa cao 3.14. Em particular provamos que


p
q

<
1

5q
2
tem
innitas solu coes racionais
p
q
, para todo irracional. O n umero

5 e o
maior com essa propriedade. De fato, se
> 0, =
1 +

5
2
e


p
q

<
1
(

5 +)q
2
,
temos

q
_
1 +

5
2
_
p

<
1
(

5 +)q
=

q
_
1 +

5
2
_
p

q
_
1

5
2
_
p

<

5
2

p
q

5 +
,
ou seja,
|p
2
pq q
2
| <

1 +

5
2

p
q

_
(

5 +).
Se q e grande, 1/q
2
e pequeno, e
1+

5
2

p
q
e muito pr oximo de 0, donde
o lado direito da desigualdade e muito pr oximo de

5+
< 1, absurdo,
pois |p
2
pq q
2
| 1, de fato se p
2
pq q
2
= 0 teramos
_
p
q
_
2

_
p
q
_
1 = 0 =
p
q

_
1 +

5
2
,
1

5
2
_
,
o que e absurdo, pois
p
q
Q.
Outra maneira de ver que, para todo > 0,

1+

5
2

p
q

<
1
(

5+)q
2
tem apenas um n umero nito de solu coes
p
q
Q e observar que as me-
lhores aproximacoes racionais de
1+

5
2
sao as reduzidas
p
n
q
n
de sua fra-
cao contnua [1; 1, 1, 1, . . . ] (ver pr oxima se cao), para as quais temos

1+

5
2

p
n
q
n

=
1
(
n+1
+
n+1
)q
2
n
, com
n+1
+
n+1
se aproximando cada vez
mais de
[1; 1, 1, 1, . . . ] + [0; 1, 1, 1, . . . ] =
1 +

5
2
+

5 1
2
=

5.
3.2 Boas Aproximacoes sao Reduzidas
O proximo teorema (e seu corolario 3.17) caracteriza as reduzidas
em termos do erro reduzido da aproximacao de x por p/q, o qual e, por
denicao, |qxp|, a razao entre |xp/q| e o erro m aximo da aproximacao
por falta com denominador q, que e 1/q.
130 CAP

ITULO 3. FRA C

OES CONT

INUAS
Teorema 3.15. Para todo p, q Z, com 0 < q < q
n+1
temos
|q
n
x p
n
| |qx p|.
Alem disso, se 0 < q < q
n
a desigualdade acima e estrita.
Demonstra c

ao: Como mdc(p


n
, q
n
) = 1, temos que se
p
q
=
p
n
q
n
entao
p = kp
n
e q = kq
n
para algum inteiro k = 0 e neste caso o resultado e
claro. Assim, podemos supor que
p
q
=
p
n
q
n
de modo que

p
q

p
n
q
n

1
qq
n
>
1
q
n
q
n+1
ja que q < q
n+1
. Assim,
p
q
esta fora do intervalo de extremos
p
n
q
n
e
p
n+1
q
n+1
e portanto

x
p
q

min
_

p
q

p
n
q
n

p
q

p
n+1
q
n+1

1
qq
n+1
o que implica
|qx p|
1
q
n+1
|q
n
x p
n
|.
Alem disso, a igualdade s o pode ocorrer se x =
p
n+1
q
n+1
, donde a
n+1
2,
e q
n+1
> 2q
n
, pois numa fracao contnua nita, como no algoritmo de
Euclides, o ultimo coeciente a
n
e sempre maior que 1. Nesse caso, se
q < q
n
, teremos

x
p
q

p
q

p
n
q
n

p
n+1
q
n+1

p
n
q
n

1
qq
n

1
q
n
q
n+1
=
q
n+1
q
qq
n
q
n+1
>
1
qq
n+1
o que implica
|qx p| >
1
q
n+1
|q
n
x p
n
|.
Corolario 3.16. Para todo q < q
n
,

x
p
n
q
n

<

x
p
q

Corolario 3.17. Se |qx p| < |q

x p

|, para todo p

e q

q tais que
p
q
=
p

, ent ao p/q e uma reduzida da fracao contnua de x.


Demonstra c

ao: Tome n tal que q


n
q < q
n+1
. Pelo teorema,
|q
n
x p
n
| |qx p|, e portanto p/q = p
n
/q
n
.
3.2. BOAS APROXIMA C

OES S

AO REDUZIDAS 131
Teorema 3.18. Se

x
p
q

<
1
2q
2
ent ao
p
q
e uma reduzida da fracao
contnua de x.
Demonstra c

ao: Seja n tal que q


n
q < q
n+1
. Suponha que
p
q
=
p
n
q
n
.
Como na demonstra cao do teorema anterior,

x
p
q


1
qq
n+1
e assim
p
q
esta fora do intervalo de extremos
p
n
q
n
e
p
n+1
q
n+1
. Temos duas possibilidades:
(a) Se q
q
n+1
2
entao

x
p
q

1
qq
n+1

1
2q
2
, absurdo.
(b) Se q <
q
n+1
2
,

x
p
q

p
n
q
n

p
q

p
n+1
q
n+1

p
n
q
n

1
qq
n

1
q
n
q
n+1
=
q
n+1
q
qq
n
q
n+1
>
1
2qq
n

1
2q
2
o que tambem e um absurdo.
Exemplo 3.19. Seja =[a
0
; a
1
, a
2
, . . . ]R. Prove que, se q
n
q <
q
n+1
, mdc(p, q) = 1 e p/q = p
n
/q
n
ent ao | p/q| | p
n
/q
n
| se,
e somente se,
p
q
=
p
n+1
rp
n
q
n+1
rq
n
, onde r N e tal que 0 < r < a
n+1
/2 ou
(r = a
n+1
/2 e
n+2

n+1
1).
Solu c

ao: Como antes,

p
q

p
n
q
n


1
qq
n
>
1
q
n
q
n+1
, e portanto
p
q
esta
fora do intervalo de extremos
p
n
q
n
e
p
n+1
q
n+1
. Se |
p
q
| | p
n
/q
n
|,
p
q
esta do mesmo lado que
p
n+1
q
n+1
em rela cao a , e portanto
p
q

p
n
q
n
=
pq
n
qp
n
qq
n
tem o mesmo sinal de
p
n+1
q
n+1

p
n
q
n
=
(1)
n
q
n
q
n+1
; por outro lado, se
tivessemos |pq
n
qp
n
| 2, teramos |
p
q
| |
p
q

p
n
q
n
| |
p
n
q
n
|
2
qq
n

1
q
n
q
n+1
>
1
q
n
q
n+1
| p
n
/q
n
|, absurdo. Assim, devemos ter
pq
n
qp
n
= (1)
n
= p
n+1
q
n
q
n+1
p
n
, e portanto, como q < q
n+1
,
devemos ter p = p
n+1
rp
n
= (a
n+1
r)p
n
+ p
n1
e q = q
n+1
rq
n
=
(a
n+1
r)q
n
+q
n1
, para algum inteiro positivo r.
Temos entao


p
q

n+1
p
n
+p
n1

n+1
q
n
+q
n1

(a
n+1
r)p
n
+p
n1
(a
n+1
r)q
n
+q
n1

=
=

n+1
a
n+1
+r
(
n+1
q
n
+q
n1
)((a
n+1
r)q
n
+q
n1
)
,
132 CAP

ITULO 3. FRA C

OES CONT

INUAS
e como |
p
n
q
n
| =
1
(
n+1
q
n
+q
n1
)q
n
, temos | p/q| | p
n
/q
n
| se, e
somente se,
n+1
a
n+1
+r
(a
n+1
r)q
n
+q
n1
q
n
= a
n+1
r+
n+1
. Se 0 <
r < a
n+1
/2, temos
n+1
a
n+1
+r < 1+r a
n+1
r < a
n+1
r+
n+1
.
Se r > a
n+1
/2,
n+1
a
n+1
+r r a
n+1
+ 1 r > a
n+1
r +
n+1
.
Finalmente, se r = a
n+1
/2,
n+1
a
n+1
+r a
n+1
r +
n+1
equivale
a
1
n+2
=
n+1
a
n+1

n+1
, ou seja, a
n+2

n+1
1.
Exemplo 3.20. Seja =[a
0
; a
1
, a
2
, . . . ]R. Prove que, se q
n
q <
q
n+1
, mdc(p, q) = 1 e p/q = p
n
/q
n
ent ao |p/q| < 1/q
2
se, e somente
se, (a
n+1
2,
p
q
=
p
n+1
p
n
q
n+1
q
n
e a
n+1
2 +
n+1
<
n+2
) ou (a
n+1
2,
p
q
=
p
n
+p
n1
q
n
+q
n1
e (
n+1
2)
n+1
< 1).
Solu c

ao: Como antes,

p
q

p
n
q
n


1
qq
n
>
1
q
n
q
n+1
, e portanto
p
q
esta
fora do intervalo de extremos
p
n
q
n
e
p
n+1
q
n+1
. Se | p/q|
1
q
2
, como
1
q
2

1
qq
n
,
p
q
esta do mesmo lado que
p
n+1
q
n+1
em rela cao a , e portanto
p
q

p
n
q
n
=
pq
n
qp
n
qq
n
tem o mesmo sinal de
p
n+1
q
n+1

p
n
q
n
=
(1)
n
q
n
q
n+1
; por outro
lado, se tivessemos |pq
n
qp
n
| 2, teramos |
p
q
| |
p
q

p
n
q
n
|
|
p
n
q
n
|
2
qq
n

1
q
n
q
n+1
>
1
qq
n

1
q
2
, absurdo. Assim, devemos ter
pq
n
qp
n
= (1)
n
= p
n+1
q
n
q
n+1
p
n
, e portanto, como q < q
n+1
,
devemos ter p = p
n+1
rp
n
= (a
n+1
r)p
n
+ p
n1
e q = q
n+1
rq
n
=
(a
n+1
r)q
n
+q
n1
, para algum inteiro positivo r.
Temos entao, como antes,


p
q

=

n+1
a
n+1
+r
(
n+1
q
n
+q
n1
)((a
n+1
r)q
n
+q
n1
)
,
donde | p/q| <
1
q
2
=
1
((a
n+1
r)q
n
+q
n1
)
2
se, e somente se,
(
n+1
a
n+1
+r)((a
n+1
r)q
n
+q
n1
) <
n+1
q
n
+q
n1
.
Se essa desigualdade ocorre, devemos ter
1
n+2
+r(a
n+1
r) (
1
n+2
+
r)(a
n+1
r) (
n+1
a
n+1
+r)((a
n+1
r)q
n
+q
n1
)/q
n
q
n1
/q
n
<
(
n+1
q
n
+q
n1
)/q
n
q
n+1
/q
n
=
1
n+2
+a
n+1
, donde r = 1 ou r = a
n+1
1
(caso contrario, teramos a
n+1
4 e r(a
n+1
r) 2(a
n+1
2) a
n+1
,
absurdo).
Se r = 1 (e a
n+1
2), temos (
n+1
a
n+1
+r)((a
n+1
r)q
n
+q
n1
) <

n+1
q
n
+q
n1
equivalente a (
n+1
a
n+1
+1)(a
n+1
1+
n+1
) <
n+1
+

n+1
, que, escrevendo
n+1
= a
n+1
+
1
n+2
, equivale a a
n+1
2+
n+1
<

n+2
. Finalmente, se r = a
n+1
1, temos (
n+1
a
n+1
+r)((a
n+1
r)q
n
+
q
n1
) <
n+1
q
n
+q
n1
equivalente a (
n+1
1)(1+
n+1
) <
n+1
+
n+1
,
que equivale a (
n+1
2)
n+1
< 1.
3.2. BOAS APROXIMA C

OES S

AO REDUZIDAS 133
Dado R, denimos a ordem de como o unico n umero real
ord = > 0 (se existir) tal que, para todo > 0


p
q

<
1
q

tem innitas solucoes


p
q
Q e
|
p
q
| <
1
q
+
tem apenas um n umero nito de solucoes racionais p/q
(caso nao exista um tal , i.e., caso |
p
q
| <
1
q

tenha innitas solucoes


racionais p/q para todo > 0, denimos ord = +).
Observemos que, se = r/s Q, com r e s inteiros, S > 0, entao
ord = 1. De fato, para todo racional p/q = , temos
|
p
q
| = |
r
s

p
q
| =
|qr ps|
sq

1
sq
=
c
q
, onde c =
1
s
> 0, e portanto
|
p
q
| <
1
q
1+
tem apenas um n umero nito de solucoes racionais p/q.
Por outro lado, pelo teorema 3.11, para todo irracional, temos
ord 2. A ordem de todo n umero irracional pode ser calculada (por
causa de 3.18 e usando a proposicao 3.4) a partir de sua fra cao contnua.
Vejamos por exemplo a seguinte
Proposi cao 3.21. Para todo [2, +] existe irracional com ord =
.
Demonstra c

ao: Sabemos que as melhores aproximacoes por racio-


nais s ao dadas pelos convergentes da fra cao contnua. Se < +, toma-
mos = [0; a
1
, a
2
, a
3
, ...], onde a
1
= 2 e, para cada n 1, a
n+1
= q
2
n

e o menor inteiro que e maior ou igual a q
2
n
. Temos entao, para todo
n 1, pela proposicao 3.4,
1
4q

1
(q
2
n
+ 3)q
2
n
<
1
(a
n+1
+ 2)q
2
n
< |
p
n
q
n
| <
1
a
n+1
q
2
n

1
q
2
n
q
2
n
=
1
q

n
,
o que claramente implica ord = .
Por outro lado, no caso = +, podemos tomar = [0; a
1
, a
2
, a
3
, ...],
onde a
1
= 2 e, para cada n 1, a
n+1
= q
n2
n
. Temos entao, para todo
n 1, pela proposicao 3.4,
|
p
n
q
n
| <
1
a
n+1
q
2
n

1
q
n2
n
q
2
n
=
1
q
n
n
,
e portanto ord = +.
134 CAP

ITULO 3. FRA C

OES CONT

INUAS
3.3 Fracoes Contnuas Periodicas
Nesta secao provaremos que os n umeros reais com fra cao contnua
peri odica s ao exatamente as razes de equa coes do segundo grau com
coecientes inteiros.
Lembramos que na representa cao de x por fra cao contnua, a
n
,
n
s ao denidos por recurs ao por

0
= x, a
n
=
n
,
n+1
=
1

n
a
n
e temos

n
=
p
n2
q
n2
x
q
n1
x p
n1
, n N.
Isso da uma prova explcita do fato de que se a fra cao contnua de
x e peri odica, entao x e raiz de uma equa cao do segundo grau com
coecientes inteiros. De fato, se
n+k
=
n
, n N, k N
>0
segue que
p
n2
q
n2
x
q
n1
x p
n1
=
p
n+k2
q
n+k2
x
q
n+k1
x p
n+k1
,
entao Ax
2
+Bx +C = 0, onde
A = q
n1
q
n+k2
q
n2
q
n+k1
B = p
n+k1
q
n2
+p
n2
q
n+k1
p
n+k2
q
n1
p
n1
q
n+k2
C = p
n1
p
n+k2
p
n2
p
n+k1
.
Note que o coeciente de x
2
e nao-nulo, pois
q
n1
q
n2
e uma fra cao irredutvel
de denominador q
n2
, pois p
n1
q
n2
p
n2
q
n1
= (1)
n
, e
q
n+k1
q
n+k2
e uma
fra cao irredutvel de denominador q
n+k2
> q
n2
, donde
q
n1
q
n2
=
q
n+k1
q
n+k2
,
logo q
n1
q
n+k2
q
n2
q
n+k1
= 0.
Vamos provar agora um resultado devido a Lagrange segundo o qual
se x e uma irracionalidade quadr atica, isto e, se x e um irracional do
tipo r +

s, r, s Q, s > 0, entao a fra cao contnua de x e peri odica,


i.e., existem n N e k N
>0
com
n+k
=
n
. Neste caso, existem a,
b, c inteiros tais que ax
2
+ bx + c = 0, com b
2
4ac > 0 e

b
2
4ac
irracional. Como x =
p
n1

n
+p
n2
q
n1

n
+q
n2
, temos
ax
2
+bx +c = 0
= a
_
p
n1

n
+p
n2
q
n1

n
+q
n2
_
2
+b
_
p
n1

n
+p
n2
q
n1

n
+q
n2
_
+c = 0
= A
n

2
n
+B
n

n
+C
n
= 0,
3.4. O ESPECTRO DE LAGRANGE 135
onde
A
n
= ap
2
n1
+bp
n1
q
n1
+cq
2
n1
B
n
= 2ap
n1
p
n2
+b(p
n1
q
n2
+p
n2
q
n1
) + 2cq
n1
q
n2
C
n
= ap
2
n2
+bp
n2
q
n2
+cq
2
n2
.
Note que C
n
= A
n1
. Vamos provar que existe M > 0 tal que
0 < |A
n
| M para todo n N, e portanto 0 < |C
n
| M, n N:
A
n
= ap
2
n1
+bp
n1
q
n1
+cq
2
n1
= aq
2
n1
_
x
p
n1
q
n1
__
x
p
n1
q
n1
_
,
onde x e x s ao as razes de aX
2
+bX +c = 0, mas

x
p
n1
q
n1

<
1
q
2
n1
1 = |A
n
| = aq
2
n1

x
p
n1
q
n1

x
p
n1
q
n1

a
_
| x x| +

x
p
n1
q
n1

_
M
def
= a(| x x| + 1).
Notemos agora que, para qualquer n N,
B
2
n
4A
n
C
n
= (p
n1
q
n2
p
n2
q
n1
)
2
(b
2
4ac) = b
2
4ac.
Portanto
B
2
n
4A
n
C
n
+b
2
4ac 4M
2
+b
2
4ac
= B
n
M

def
=
_
4M
2
+b
2
4ac.
Provamos assim que A
n
, B
n
e C
n
estao uniformemente limitados,
donde ha apenas um n umero nito de possveis equa coes A
n
X
2
+B
n
X+
C
n
= 0, e portanto de possveis valores de
n
. Assim, necessariamente

n+k
=
n
para alguma escolha de n N, k N
>0
.
3.4 O espectro de Lagrange
Seja um n umero irracional. Denimos k() como o unico k > 0
(se existir) tal que, para todo > 0, |
p
q
| <
1
(k +)q
2
tem innitas
solucoes racionais p/q e |
p
q
| <
1
(k )q
2
tem apenas um n umero nito
de solucoes racionais p/q (caso nao exista um tal k, i.e., caso |
p
q
| <
136 CAP

ITULO 3. FRA C

OES CONT

INUAS
1
kq
2
tenha innitas solucoes racionais p/q para todo k > 0, denimos
k() = +). Pelo Teorema de Hurwitz-Markov, temos k()

5,
R \ Q. Por outro lado, e possvel provar que k(
1+

5
2
) =

5.
Estaremos interessados nos R tais que k() < +, e, mais
particularmente, na imagem da funcao k, isto e, no conjunto L = {k() |
R\Q e k() < +}. Este conjunto e conhecido como o espectro de
Lagrange.
O conjunto L encodica uma serie de propriedades diofantinas (isto
e, relativas `as boas aproximacoes por n umeros racionais) de n umeros
reais, e vem sendo estudado ha bastante tempo. Talvez o primeiro re-
sultado nao-trivial sobre ele se deva a Markov, que provou em 1879 que
L (, 3) =
_
k
1
=

5 < k
2
= 2

2 < k
3
=

221
5
<
_
,
onde (k
n
) e uma sequencia convergente a 3 tal que k
n
/ Q mas k
2
n
Q
para todo n. Assim, ocomecodo espectro de Lagrange e discreto. Essa
armacao nao e verdadeira para todo o conjunto L. Marshall Hall provou
em 1947 que L contem toda uma semi-reta (por exemplo [6, +)), e G.
Freiman determinou em 1975 a maior semi-reta que esta contida em L,
que e
_
2221564096 + 283748

462
491993569
, +
_
.
Uma apresenta cao detalhada destes e de outros resultados sobre o
espectro de Lagrange pode ser encontrada em [3].
Na referencia [9] s ao provados resultados sobre propriedades geome-
tricas (relativas a geometria fractal) dos espectros de Markov e Lagrange,
que envolvem resultados delicados sobre somas de conjuntos de Cantor
regulares.
Problemas Propostos
3.1. Determine a fracao contnua de

7. Mostre que ela e peri odica a


partir de um certo ponto, e determine o perodo.
3.2. Escreva na forma r +

s, com r, s Q, s 0, os n umeros reais
cujas representa coes em fracoes contnuas sao as seguintes:
(a) [0; 3, 6, 3, 6, 3, 6, . . . ].
(b) [0; k, k, k, . . . ], onde k e um inteiro positivo dado.
(c) [0; 1, 1, 2, 2, 1, 1, 2, 2, 1, 1, 2, 2, . . . ].
3.4. O ESPECTRO DE LAGRANGE 137
3.3. (a) Sabendo que 3, 14 < x < 3, 15, determine o maior natural n e
inteiros a
0
, a
1
, . . . , a
n
para os quais e possvel garantir que a repre-
senta cao em fracoes contnuas de x come ca por [a
0
; a
1
, . . . , a
n
].
(b) Sabendo que 3, 141592 < x < 3, 141593, determine o maior natural
n e inteiros a
0
, a
1
, . . . , a
n
para os quais e possvel garantir que a
representa cao em fracoes contnuas de x come ca por [a
0
; a
1
, . . . , a
n
].
(c) Sabendo que 3, 1415926 < x < 3, 1415927, determine o maior natu-
ral n e inteiros a
0
, a
1
, . . . , a
n
para os quais e possvel garantir que a
representa cao em fracoes contnuas de x come ca por [a
0
; a
1
, . . . , a
n
].
3.4. (a) Determine as primeiros 6 reduzidas da fracao continua de

5.
(b) Denimos a sequencia a
n
= n

5 n

5. Determine os valores de
n 2011 tais que a
n
seja respectivamente maximo e mnimo.
3.5. Demonstrar que, para todo inteiro positivo a, temos as seguintes
expansoes em fracoes contnuas peri odicas:
(a)

a
2
+ 1 = [a, 2a].
(b)

a
2
1 = [a 1, 1, 2a 2].
(c)

a
2
2 = [a 1, 1, a 2, 1, 2a 2].
(d)

a
2
a = [a 1, 2, 2a 2].
3.6. Encontrar as fracoes contnuas de

a
2
+ 4 e

a
2
4.
3.7. Prove que, para quaisquer inteiros p, q com q > 0, temos
|

2
p
q
| >
1
3q
2
. Determine todos os pares de inteiros (p, q) com q > 0
tais que |

2
p
q
| <
1
q
3
.
3.8. Prove que, para qualquer R\Q, e quaisquer s, t R com s < t,
existem inteiros m, n com n > 0 tais que s < n +m < t.
3.9. Seja
p
n
q
n
=
1
1 +
1
2
2 +
3
2
2 +
5
2
.
.
.
2 +
(2n 3)
2
2
138 CAP

ITULO 3. FRA C

OES CONT

INUAS
a n-esima convergente da fracao contnua
1
1 +
1
2
2 +
3
2
2 +
5
2
2 +
7
2
.
.
.
Demonstrar que
p
n
q
n
= 1
1
3
+
1
5

1
7
+ + (1)
n1 1
2n1
.
3.10. Dizemos que dois n umeros irracionais e sao GL
2
(Z)-equiva-
lentes se existem inteiros a, b, c, d com |ad bc| = 1 tais que =
a+b
c+d
.
Mostre que, se as fracoes contnuas de e sao = [a
0
; a
1
, a
2
, . . . ]
e = [b
0
; b
1
, b
2
, . . . ] ent ao e sao GL
2
(Z)-equivalentes se, e somente
se, existem r Z e n
0
N tais que b
n
= a
n+r
, n n
0
.
Captulo 4
Equacoes diofantinas nao
lineares
4.1 Teorema de Pitagoras e triplas Pitagoricas
Um dos teoremas basicos e elementares da geometria euclidiana e o
conhecido Teorema de Pit agoras, que relaciona o comprimento dos lados
de um tri angulo quando um dos angulos e reto. Nesta secao estudaremos
os tri angulos ret angulos com a restricao adicional que o comprimento dos
lados s ao n umeros inteiros. Antes de impor tal restricao vejamos uma
prova, entre as muitas conhecidas, deste teorema.
Teorema 4.1 (Pit agoras). Sejam a, b e c os comprimentos dos lados
de um tri angulo. O angulo oposto ao lado c e reto se e somente se
a
2
+b
2
= c
2
.
Demonstra c

ao: Nossa prova se baseia no seguinte desenho, no qual


temos colocado quatro tri angulos ret angulos de catetos a e b e hipotenusa
c ao redor de um quadrado de lado c.
b
a
b
a
b
a
b a
c
c
c
c
139
140 CAP

ITULO 4. EQUA C

OES DIOFANTINAS N

AO LINEARES
Observe que a gura externa formada tambem e um quadrado (com-
prove!) de lado a+b. A area deste quadrado pode ser calculada de duas
formas diferentes: uma e a forma natural, elevando o comprimento do
lado ao quadrado, isto e (a + b)
2
, e a outra calculando a area como a
soma das areas dos quatro tri angulos mais a area do quadrado de lado
c, isto e,
(a +b)
2
= 4
_
ab
2
_
+c
2
= 2ab +c
2
.
Subtraindo 2ab dos dois lados da igualdade obtemos a rela cao desejada.
Claramente esta rela cao mostra a dependencia de um lado com res-
peito aos outros dois. Desta forma e facil encontrar exemplos de tri an-
gulos que tem exatamente dois lados de comprimento inteiro, como por
exemplo (1, 1,

2), (1, 2,

5), (3, 2

10, 7) etc. Menos evidente e encon-


trar tri angulos com todos seus lados inteiros, fato que pode ser ilustrado
no seguinte
Exemplo 4.2. Encontre todos os tri angulos ret angulos com lados intei-
ros e um cateto igual a 30.
Solu c

ao: Precisamos encontrar inteiros positivos b e c tais que 900 +


b
2
= c
2
. Esta igualdade pode ser reescrita da seguinte forma
2
2
3
2
5
2
= (c +b)(c b).
Observe que os n umeros c+b e cb tem a mesma paridade, logo os dois
s ao pares, e como c +b e maior que c b, as unicas formas de distribuir
os fatores de 900 entre estes fatores s ao
_
c +b = 450
c b = 2
_
c +b = 150
c b = 6
_
c +b = 90
c b = 10
e
_
c +b = 50
c b = 18
que gera as solucoes (30, 224, 226), (30, 72, 78), (30, 40, 50) e (30, 16, 34).
N ao temos um metodo an alogo para determinar os lados de um tri an-
gulo dado o comprimento da hipotenusa, mas eles podem ser encontrados
por uma procura ordenada como e mostrado a seguir
Exemplo 4.3. Encontre todos os tri angulos ret angulos com lados intei-
ros e a hipotenusa igual a 65.
Neste caso, precisamos encontrar dois n umeros inteiros a e b, que
podemos supor, sem perda de generalidade, satisfazerem a < b, tais que
4.2. TRI

ANGULOS RET

ANGULOS DE PIT

AGORAS E PLAT

AO141
a
2
+b
2
= 65
2
= 4225. Como b
2
+(b1)
2
4225, temos que 64 b 47.
Assim temos 18 possveis valores de b que precisamos testar se vericam
que 4225 b
2
e um quadrado perfeito. De fato, este n umero e quadrado
perfeito quando b e igual a 52, 56, 60 e 63, que gera os tri angulos com
lados de comprimento (39, 52, 65), (33, 56, 65), (25, 60, 65) e (16, 63, 65).
O caso geral do problema anterior sera resolvido por outro metodo
na secao 4.12.
No caso em que os tres n umeros (a, b, c) sejam inteiros e sejam lados
de um tri angulo ret angulo, diremos que estes tres n umeros formam uma
Tripla Pitagorica, que sera o objeto de estudo nas seguintes secoes.
Problemas Propostos
4.1. Determine todos os tri angulos ret angulos com lados inteiros e um
de seus catetos com comprimento igual a
a) 60
b) 825
4.2. Determine todos os tri angulos ret angulos com lados inteiros e hi-
potenusa de comprimento 105.
4.3. a) Mostre que o quadrado de um n umero mpar sempre deixa resto
1 quando dividido por 8.
b) Existe algum tri angulo ret angulo com lados inteiros e catetos mpa-
res?
4.4. Observem que usando o tri angulo ret angulo (1, 1,

2), e possvel
construir

2 com regua e compasso. Mostrar que para todo inteiro po-


sitivo n, o n umero

n e construtvel com regua e compasso.
4.5. Existe algum tri angulo ret angulo com lados inteiros e permetro
igual a uma potencia de 2?
4.2 Triangulos retangulos de Pitagoras e Platao
Observemos que podemos gerar innitas triplas pit agoricas a partir
de uma. De fato, como 3
2
+ 4
2
= 5
2
, temos que (3, 4, 5) e uma tripla
Pitagorica, e a partir dela podemos gerar innitas multiplicando por um
inteiro positivo:
(6, 8, 10), (3, 12, 15), . . . , (3k, 4k, 5k).
142 CAP

ITULO 4. EQUA C

OES DIOFANTINAS N

AO LINEARES
As triplas nesta lista s ao chamadas triplas pitagoricas N

AO primitivas,
porque s ao obtidas multiplicado (3, 4, 5) por um n umero inteiro maior
que 1, enquanto (3, 4, 5) e chamada tripla pitagorica primitiva, isto e,
uma tripla pitagorica (a, b, c) e chamada primitiva se ela nao e obtida a
partir de outra multiplicando por uma constante inteira maior ou igual
a 2.
Pit agoras observou que existe uma famlia innita de triplas pitago-
ricas primitivas: (5, 12, 13), (7, 24, 25) (9, 40, 41), etc. Em todos os casos
estudados por ele, um dos catetos e a hipotenusa s ao inteiros consecuti-
vos. Para gerar uma formula para todas as triplas do tipo (a, b, b + 1)
(chamadas triplas pitagoricas cl assicas de primeiro tipo) e suciente ob-
servar que
a
2
= (b + 1)
2
b
2
= 2b + 1,
logo a e um n umero mpar, portanto a = 2k + 1 com k um n umero
natural, assim substituindo na formula anterior temos que 4k
2
+4k+1 =
2b +1, e assim b = 2k
2
+2k e c = 2k
2
+2k +1, isto e, obtemos a famlia
de triplas pitagoricas (2k + 1, 2k
2
+ 2k, 2k
2
+ 2k + 1).
Por outro lado, Platao observou outra famlia de triplas primitivas
em que a diferen ca entre a hipotenusa e um cateto e 2 (chamadas triplas
pitagoricas cl assicas de segundo tipo), isto e, triplas da forma (a, b, b+2).
Seguindo o argumento anterior temos que a
2
= (b + 2)
2
b
2
= 4b + 4.
Assim a e par. Escrevendo a = 2s e substituindo na equa cao anterior
obtemos que s
2
= b + 1, como estamos interessados somente em triplas
Pitagoricas primitivas, entao b nao pode ser par, assim s nao pode ser
mpar, assim s = 2k com k inteiro, e neste caso obtemos a famlia de
triplas pitagoricas (4k, 4k
2
1, 4k
2
+ 1).
A seguinte tabela mostra as triplas obtidas a partir das equa coes de
Pit agoras e Platao
k (2k + 1, 2k
2
+ 2k, 2k
2
+ 2k + 1) (4k, 4k
2
1, 4k
2
+ 1)
1 (3, 4, 5) (4, 3, 5)
2 (5, 12, 13) (8, 15, 17)
3 (7, 24, 25) (12, 35, 37)
4 (9, 40, 41) (16, 63, 65)
5 (11, 60, 61) (20, 99, 101)
6 (13, 84, 85) (24, 143, 145)
7 (15, 112, 113) (28, 195, 197)
Problemas Propostos
4.6. Existem triplas pitagoricas primitivas, tais que a diferen ca entre a
hipotenusa e um cateto seja 3, 4, 5, 6, 7 ou 8? Caso existam em algum
dos casos, determine a formula geral delas.
4.3. TRIPLAS PITAG

ORICAS PRIMITIVAS 143


4.7. Dado r natural, encontre a famlia de todas as triplas pitagoricas
primitivas tais que a diferen ca entre a hipotenusa e um cateto seja r.
4.8. Encontre todas as triplas pitagoricas tais que os lados estao em
progressao aritmetica.
4.3 Triplas Pitagoricas Primitivas
Como falamos anteriormente, se a, b e c s ao os comprimentos dos
lados de um tri angulo ret angulos e queremos caracterizar os tri angulos
ret angulos com lados inteiros, o problema e equivalente a encontrar todas
as triplas de n umeros inteiros (a, b, c) tais que se verique a rela cao
a
2
+ b
2
= c
2
. Como ja foi observado anteriormente, tais triplas s ao
denominadas triplas pitagoricas.
Para isto, suponhamos que temos um tripla pitagorica (a, b, c). Po-
demos supor que os tres n umeros nao tem fatores em comum dois a dois,
porque, por exemplo, no caso em que a e b tem um fator comum d entao
d
2
divide a
2
+b
2
= c
2
, portanto d tambem divide c, assim (
a
d
,
b
d
,
c
d
) tam-
bem e tripla pitagorica. Uma tripla pitagorica na qual seus termos nao
tem fator em comum dois a dois se denomina tripla pitagorica primitiva.
Observemos que a e b nao podem ser simultaneamente mpares, ja
que se temos um n umero mpar 2k + 1, ao elev a-lo ao quadrado temos
que
(2k + 1)
2
= 4k
2
+ 4k + 1 = 4k(k + 1) + 1,
e logo o quadrado de um n umero mpar e sempre congruente a 1 m odulo
4. Assim, se somamos dois quadrados de n umeros mpares obteremos um
n umero que e congruente a 2 m odulo 4, mas o quadrado de um n umero
par e sempre divisvel por por 4. Portanto, temos que a e b nao podem
ser simultaneamente mpares. Podemos supor entao que a e mpar e b e
par, e, entao, c e mpar. Temos entao
b
2
= c
2
a
2
= (c a)(c +a).
mas c e a nao tem fator em comum, donde u =
c+a
2
e v =
ca
2
nao podem
ter fator em comum (caso contrario, se d divide simultaneamente u e v,
entao d tambem dividira a u + v = c e u v = a). Daqui segue que
b
2
= uv, isto e, o produto uv e o quadrado de um n umero inteiro. Mas
como u e v nao tem fator em comum, entao cada um deles tem que ser
o quadrado de um n umero inteiro. Assim
c +a
2
= u = m
2
,
c a
2
= v = n
2
, b = 2mn,
144 CAP

ITULO 4. EQUA C

OES DIOFANTINAS N

AO LINEARES
onde m e n nao tem fator comum. Conclumos entao que todas as triplas
pitagoricas primitivas estao dadas pelas formulas
a = m
2
n
2
, b = 2mn e c = m
2
+n
2
com m e n sem fator comum e m+nmpar e todas as triplas pitagoricas
se encontram a partir de uma tripla pitagorica primitiva, multiplicando
por uma constante.
Observemos que o resultado encontrado na secao anterior, isto e, as
triplas pitagoricas encontradas por Pit agoras, correspondem ao caso em
que m = k + 1 e n = k, e as triplas pitagoricos encontradas por Platao
se reduzem ao caso em que m = 2k e n = 1. A seguinte tabela ilustra
algumas solucoes novas nao encontradas pelo metodo anterior.
m n (a, b, c)
5 2 (21, 20, 25)
7 2 (45, 28, 53)
7 4 (33, 56, 65)
8 3 (55, 48, 73)
8 5 (39, 80, 89)
9 2 (77, 36, 85)
9 4 (65, 72, 97)
Como uma aplicacao do resultado anterior, consideremos o seguinte
Exemplo 4.4. Encontrar todas as triplas de n umeros (a, b, c) tais que
a
2
, b
2
e c
2
estao em progressao aritmetica.
Observemos que a tripla (1, 5, 7) e solucao do problema. Sabemos
que, em uma progress ao aritmetica, a diferen ca de dois termos conse-
cutivos e constante; assim, o problema se reduz a encontrar todas as
triplas de n umeros (a, b, c) tais que
b
2
a
2
= c
2
b
2
,
isto e, a
2
+ c
2
= 2b
2
. Daqui segue que a e c tem a mesma paridade e,
portanto, existem inteiros r e s tais que c = r + s e a = r s (basta
fazer r =
c+a
2
e s =
ca
2
). Deste modo, substituindo teremos que
2b
2
= a
2
+c
2
= (r s)
2
+ (r +s)
2
= 2(r
2
+s
2
)
donde (r, s, b) e uma tripla pitagorica, portanto existem inteiros m e n
tais que r = m
2
n
2
, s = 2mn e b = m
2
+n
2
, e se conclui que
a = |m
2
n
2
2mn|, b = m
2
+n
2
, c = m
2
n
2
+ 2mn,
com m > n e m+nmpar, e uma tripla que satisfaz o pedido. Alem disso
todas as triplas primitivas s ao desta forma. A seguinte tabela mostra os
primeiros valores de a, b e c com a
2
, b
2
e c
2
em progress ao aritmetica
4.3. TRIPLAS PITAG

ORICAS PRIMITIVAS 145


m n (a, b, c)
2 1 (1, 5, 7)
3 2 (7, 13, 17)
4 1 (7, 17, 23)
4 3 (17, 25, 31)
Usando a mesma tecnica podemos resolver o seguinte
Exemplo 4.5. Determine todas as ternas (a, b, c) de inteiros positivos
tais que a
2
= 2
b
+c
4
.
Solu cao. Seja (a, b, c) uma de tais solucoes. Dado que
(a c
2
)(a +c
2
) = 2
b
,
existem dois naturais m > n tais que a c
2
= 2
n
e a + c
2
= 2
m
.
Substraindo tais equa coes, obtemos que 2c
2
= 2
m
2
n
, assim c
2
=
2
n1
(2
mn
1). Segue que n 1 e par e que 2
mn
1 = k
2
, mas
k
2
+ 1 2 (mod 4), portanto mn = 1. Assim, todas as solucoes s ao
da forma (3 2
2r
, 4r + 3, 2
r
) com r N.
Exemplo 4.6. Encontrar todas as solu coes inteiras de x
2
+y
2
+z
2
= t
2
.
Solu cao. Observemos inicialmente (considerando congruencias m o-
dulo 4) que pelo menos dois de tais n umeros tem que ser pares. Sejam
y = 2l e z = 2m. Como t > x, podemos supor que t = x + 2n, com n
inteiro positivo. Substituindo, teremos que (x + 2n)
2
= x
2
+ 4l
2
+ 4m
2
,
isto e, nx + n
2
= l
2
+ m
2
. Obtemos assim que x =
l
2
+m
2
n
2
n
. Como x
e inteiro positivo, escolhemos n como sendo um divisor de l
2
+ m
2
me-
nor que

l
2
+m
2
. Conclumos que todas as solucoes da equa cao estao
dadas por
x =
l
2
+m
2
n
n, y = 2l, z = 2m, t =
l
2
+m
2
n
+n,
onde n e um divisor de l
2
+m
2
menor que

l
2
+m
2
.
Outra representa cao das solucoes desta equa cao e proposta no pro-
blema 4.35.
Problemas Propostos
4.9. Determine todas as solu coes inteiras da equacao 2x
2
+y
2
= z
2
.
4.10. Existe algum tri angulo ret angulo com lados inteiros e permetro
2p
k
com p um n umero primo e k inteiro?
4.11. Determine todos os tri angulos ret angulos com lados inteiros e pe-
rmetro 120.
146 CAP

ITULO 4. EQUA C

OES DIOFANTINAS N

AO LINEARES
4.4 Triangulos pitagoricos e o metodo geome-
trico
Nesta secao mostraremos outra forma de encontrar a forma geral das
triplas pitagoricas com o uso de um pouco de geometria analtica. Para
isto basta observar que a
2
+ b
2
= c
2
e equivalente `a equa cao (
a
c
)
2
+
(
b
c
)
2
= 1, isto e, o ponto (
a
c
,
b
c
) e um ponto com coordenadas racionais
da circunferencia C com centro na origem e raio 1, ou, equivalentemente,
(
a
c
,
b
c
) e uma solucao da equa cao x
2
+ y
2
= 1 com coordenadas que s ao
n umeros racionais.
Note que os pontos (1, 0), (0, 1) s ao pontos que pertencem `a cir-
cunferencia. Agora, se temos outro ponto sobre a circunferencia que tem
coordenadas racionais, digamos (r, s), entao a reta L que passa pelos
pontos (0, 1) e (r, s) corta a circunferencia exatamente nesses pontos e
tem inclina cao
s+1
r
, que e um n umero racional.
Agora pensemos ao contrario, peguemos uma reta L que passa por
(0, 1) e tem inclina cao racional
m
n
. Ela cortara a circunferencia C no
ponto (0, 1), em algum outro ponto que pode ser calculado resolvendo
o sistema de equa coes
_
x
2
+y
2
= 1
y =
m
n
x 1,
x
y
y =
m
n
x 1
(0, 1)
(
a
c
,
b
c
)
Assim, substituindo y por
m
n
x 1, obtemos
1 = x
2
+
_
m
n
x + 1
_
2
= x
2
+
m
2
n
2
x
2
2
m
n
x + 1.
A equa cao anterior possui uma solucao esperada x = 0, A segunda
solucao e dada por (1 +
m
2
n
2
)x = 2
m
n
, isto e, x =
2mn
m
2
+n
2
, e portanto
a
c
=
2mn
m
2
+n
2
e
b
c
=
m
n
2mn
m
2
+n
2
1 =
m
2
n
2
m
2
+n
2
,
4.4. TRI

ANGULOS PITAG

ORICOS E O M

ETODO GEOM

ETRICO147
agora como as fracoes s ao iguais, supondo que mdc(m, n) = 1, segue-se
que, no caso em que m+n e mpar, existe um inteiro k tal que
a = 2mnk, b = (m
2
n
2
)k, c = (m
2
+n
2
)k.
E, no caso em que m + n e par, existe um inteiro k tal que a = mnk,
b =
(m
2
n
2
)
2
k e c =
(m
2
+n
2
)
2
k.
O interessante deste metodo geometrico, e que permite resolver equa-
coes mais gerais de tipo quadratico quando conhecemos um ponto, como
se mostra no seguinte exemplo
Exemplo 4.7. Determinar todas as solu coes inteiras da equacao a
2
+
2b
2
= 11c
2
.
Dividindo por c
2
obtemos (
a
c
)
2
+2(
b
c
)
2
= 11, assim queremos encon-
trar as solucoes racionais da equa cao x
2
+ 2y
2
= 11 que e uma elipse
centrada na origem do plano cartesiano. Por inspecao direta temos que
os pontos (3, 1) s ao solucoes. A equa cao da reta que passa pelo ponto
(3, 1) e tem inclina cao
m
n
e y =
m
n
(x + 3) 1 =
m
n
x +
3mn
n
.
y =
m
n
x +
3mn
n
(3, 1)
(
a
c
,
b
c
)
Os pontos de interseccao da reta com a elipse, s ao as solu coes obtidas
quando resolvemos o sistema de equa coes
_
x
2
+ 2y
2
= 11
y =
m
n
x +
3mn
n
,
agora, substituindo y na equa cao da elipse obtemos que
11 = x
2
+ 2
_
m
n
x +
3mn
n
_
2
=
n
2
+ 2m
2
n
2
x
2
+ 4
m(3mn)
n
2
x + 2
9m
2
6mn +n
2
n
2
,
148 CAP

ITULO 4. EQUA C

OES DIOFANTINAS N

AO LINEARES
assim, x e solucao da equa cao quadratica
x
2
+ 4
m(3mn)
n
2
+ 2m
2
x 3
3n
2
+ 4mn 6m
2
n
2
+ 2m
2
= 0.
Como (3, 1) e um dos pontos de interseccao da elipse com a reta, o
leitor pode comprovar diretamente que x = 3 deve ser solucao desta
equa cao. Agora usando o fato que o coeciente independente de uma
equa cao quadratica e o produto das razes, temos que a outra raiz e
a
c
= x =
3n
2
+ 4mn 6m
2
n
2
+ 2m
2
,
e assim
b
c
= y =
m
n
_
3n
2
+ 4mn 6m
2
n
2
+ 2m
2
_
+
3mn
n
=
2m
2
+ 6mn n
2
n
2
+ 2m
2
.
Como as fracoes que aparecem em cada igualdade s ao iguais, entao exis-
tir a um inteiro k tal que
a =
k
d
(3n
2
+4mn6m
2
), b =
k
d
(2m
2
+6mnn
2
) e c =
k
d
(n
2
+2m
2
)
(onde d = mdc(3n
2
+ 4mn 6m
2
, 2m
2
+ 6mn n
2
, n
2
+ 2m
2
), que
pertence a {1, 2, 11}, caso mdc(m, n) = 1) e usando a simetria da elipse
podemos considerar os valores positivos de a, b e c. A tabela seguinte
ilustra algumas solucoes obtidas a partir das equa coes anteriores supondo
k = 1
m n (a, b, c)
1 -1 (7, 5, 3)
0 1 (3, 1, 1)
1 1 (1, 7, 3)
2 1 (13, 19, 9)
3 1 (39, 37, 19)
4 1 (77, 55, 33)
4 3 (21, 95, 25)
Problemas Propostos
4.12. Determine todas as solu coes inteiras da equacao a
2
+ 3b
2
= 13c
2
.
4.13. Mostre que a equacao x
2
+ y
2
= 6z
2
n ao possui solu coes inteiras
positivas.
Dica: Mostre que o quadrado de todo mpar deixa resto 1 quando di-
vidido por 8, assim os quadrados quando divididos por 8 somente podem
deixar resto 0, 1 ou 4.
4.5. TRI

ANGULOS COM LADOS INTEIROS E



ANGULOS EM PROGRESS

AO
4.14. Demonstre que a equacao x
2
+y
2
= 3z
2
n ao tem solu coes inteiras
positivas.
4.15. Encontrar todas as solu coes inteiras da equacao x
2
+y
2
= 5z
2
.
4.16. Um bloco retangular e chamado Bloco de Euler se o comprimento
de seus lados e inteiro e o comprimento das diagonais em cada face e
inteira. Um bloco de Euler e chamado primitivo se o mdc dos compri-
mentos dos lados e 1. Mostrar que um bloco com medidas 44, 117 e 240
e um bloco de Euler.
4.17. Mostrar que se um bloco com medidas a, b e c e de Euler, ent ao
um bloco com medidas (bc, ab, ac) e de Euler.
4.18. Encontre um bloco de Euler cuja aresta de menor comprimento e
85.
4.19. Mostrar que existem innitos blocos de Euler primitivos .
Obs.:

E um problema aberto a existencia de um bloco de Euler com
diagonal inteira.
4.5 Triangulos com lados inteiros e angulos em
progressao aritmetica
Como uma aplicacao do resultado da secao anterior, pretendemos
caracterizar todos os tri angulos ABC com lados de comprimentos in-
teiros a, b e c que tenham seus angulos em progress ao aritmetica. Como
a soma dos angulos internos de um tri angulo e 180
o
, se a razao da pro-
gress ao e , entao os angulos ter ao medidas 60

, 60

e 60

+.
Suponhamos que o angulo em B mede 60

e o angulo em A e o maior,
e seja D o pe da altura desde o vertice A. Observemos que o tri angulo
BAC e de fato, meio tri angulo equil atero, assim BD =
c
2
.
Como os tri angulos ADB e ADC s ao ret angulos, aplicando o
teorema de Pit agoras temos que
c
2

_
c
2
_
2
= AD
2
= b
2

_
a
c
2
_
2
que equivale a
3
4
c
2
= b
2
a
2
+ac
1
4
c
2
,
150 CAP

ITULO 4. EQUA C

OES DIOFANTINAS N

AO LINEARES
D
60

B C
A
c
2
a
c
2
c
b
ou seja, os tri angulos que tem os lados em progress ao aritmetica s ao os
tri angulos que tem lados que satisfazem a rela cao
a
2
ac +c
2
= b
2
,
que equivale usando o metodo geometrico, a determinar as solucoes ra-
cionais da equa cao x
2
xy +y
2
= 1, que representa no plano uma elipse
rotacionada. Observemos que esta equa cao possui a solucao (0, 1).
Consideremos a interse cao da elipse com a reta y =
m
n
x 1.
y =
m
n
x 1
(1, 1)
(0, 1)
(
a
b
,
c
b
)
x
2
xy +y
2
= 1
A interse cao da reta com a elipse e formada por pontos cuja coorde-
nada x satisfaz
(m
2
mn +n
2
)x
2
(2mn)nx = 0.
Assim,
a
b
=
2mmn
2
m
2
mn +n
2
,
c
b
=
m
2
n
2
m
2
mn +n
2
.
4.6. OUTRA RELA C

AO DE

ANGULOS 151
Conclumos que os comprimentos dos lados dos tri angulos com um an-
gulo de 60

s ao
a = k(2mn n
2
) b = k(m
2
mn +n
2
) c = k(m
2
n
2
), k Z.
Fica como exerccio para o leitor completar a seguinte tabela, conside-
rando k = 1.
m n (a, b, c)
2 1 (3, 3, 3)
3 1 (5, 7, 8)
3 2
4 2
4 3
5 1
5 2
Problemas Propostos
4.20. Mostre que n ao existem tri angulos com lados inteiros de tal forma
que um de seus angulos seja 30

, 45

ou 72

.
4.21. Encontre todos os tri angulos com lados inteiros e com um angulo
tal que cos =
2
5
.
4.22. Seja 0 < < 180

, tal que cos e um n umero racional. Mostre


que existem innitos tri angulos n ao semelhantes e com lados inteiros
tais que um de seus angulos mede .
4.6 Outra relacao de angulos
Nesta secao mostraremos como determinar todos os tri angulos com
lados inteiros, tais que um angulo e o dobro do outro. Para isto, supo-
nhamos que temos um tri angulo ABC com lados de comprimentos a,
b e c e tal que A = 2B = 2. Tracemos a bissetriz desde o vertice A
e denotemos por D o pe da bissetriz no lado BC.
Pelas condi coes acima, segue que o tri angulo BDA e isosceles, e,
assim, BD = DA = x. Por outro lado, o angulo ADC e angulo externo
ao tri angulo BDA, logo ele e igual `a soma dos angulos nao adjacentes,
donde ADC = 2.
152 CAP

ITULO 4. EQUA C

OES DIOFANTINAS N

AO LINEARES
A
B
C
D
Assim, os tri angulos ABC e DCA s ao semelhantes, ja que seus
angulos s ao iguais, de onde temos as rela coes entre os lados
b
a
=
x
c
=
a x
b
.
Da primeira igualdade temos que x =
bc
a
e substituindo na segunda
igualdade obtemos que
b
a
=
a
bc
a
b
=
a
2
bc
ab
, assim b
2
+bc = a
2
.
Podemos resolver esta equa cao pelo metodo geometrico ou simplesmente,
no caso em que mdc(b, c) = 1, observando que b(b + c) e um quadrado
perfeito, e que b e b + c nao tem fator em comum, donde b e b + c tem
que ser quadrados de n umeros inteiros, logo
_
b = n
2
b +c = m
2
com m e n
sem fator comum. Portanto, o conjunto de tri angulos procurados e o
dos que satisfazem as rela coes
a = kmn, b = kn
2
e c = k(m
2
n
2
)
com n e m sem fator comum e k inteiro arbitr ario.
Problemas Propostos
4.23. Determine todos os tri angulos ABC com lados inteiros, A =
2B e c = 40.
4.24. Encontre uma formula para o n umero de tri angulos ABC com
lados inteiros, A = 2B e lado c um inteiro dado (esta formula deve
depender do n umero de divisores de c).
4.7. CONTANDO TRI

ANGULOS PITAG

ORICOS COM UM CATETO DADO153


4.7 Contando triangulos pitagoricos com um ca-
teto dado
Dado um n umero natural M, uma pergunta que podemos fazer e
quantos tri angulos ret angulos com lados inteiros tem M como um de
seus catetos. Para responder esta pergunta basta contar o n umero de
formas que podemos escrever M como k(m
2
n
2
) ou 2kmn satisfazendo
as condi coes: m e n nao tem fator comum e m + n e mpar. Primeiro
vamos supor que k = 1, isto e, contaremos somente triplas pitagoricas
primitivas. Podemos separar o problema em dois casos:
Se M e mpar, entao precisamos encontrar o n umero de soluc oes
de
M = m
2
n
2
= (mn)(m+n),
isto e, o n umero de formas de fatorar M como produto de dois
fatores distintos, mas isto e exatamente
C(M) =
_
1
2
d(M), se M nao e um quadrado perfeito
1
2
(d(M) 1), se M e um quadrado perfeito,
onde d e a funcao n umero de divisores estudada no captulo 2.
Por exemplo se M = 15, temos que d(15) = 4, logo existem dois tri-
angulos pitagoricos primitivos com 15 como lado. Para determinar
estes tri angulos, precisamos resolver os sistemas
_
m+n = 15
mn = 1
e
_
m+n = 5
mn = 3
, que tem como solucoes (m, n) = (8, 7) e (m, n) =
(4, 1), e que geram os tri angulos com lados (15, 112, 113) e (15, 8, 17).
Observemos que desta forma nao geramos os tri angulos com lados
(15, 20, 25) e (15, 36, 39) porque estes tri angulos nao s ao primitivos.
Se M e par, precisamos contar de quantas forma podemos escrever
M = 2mn comme n de paridades diferentes. Assim, se
M
2
empar,
nao existe nenhum tri angulo pitagorico primitivo que tenha M
como lado. Escrevamos M = 2
a+1
N onde N empar e a 1, assim
o n umero de tri angulos pitagoricos primitivos com M = 2
a+1
N e
C(2
a+1
N) =
_
0, se a = 0
d(N), se a > 0.
Por exemplo se M = 60, entao N = 15 e d(N) = 4, isto e, 15 =
15 1 = 5 3, assim (m, n) pode tomar os valores (30, 1), (15, 2),
154 CAP

ITULO 4. EQUA C

OES DIOFANTINAS N

AO LINEARES
(10, 3), ou (6, 5), que geram os tri angulos com lados (899, 60, 901),
(221, 60, 229), (91, 60, 109) e (11, 60, 61).
Agora, para contar o n umero de triplas pitagoricas que existem com um
cateto igual a M, consideraremos dois casos:
Se M e impar, para cada m um divisor de M e para cada tripla
pitagorica primitiva com um cateto igual a m e possvel gerar uma
tripla pitagorica com um cateto igual a M, multiplicando esta
termo da tripla por
M
m
, assim o total de triplas pitagoricas e

m|M
C(m) =

m|M
m=n
2
C(m) +

n
2
|M
C(n
2
)
=
1
2

m|M
m=n
2
d(m) +
1
2

n
2
|M
(d(n
2
) 1)
=
1
2

m|M
d(m)
1
2

n
2
|M
1.
No caso que M = p

1
1
p

2
2
p

k
k
, usando que a funcao d e multi-
plicativa temos que

i
j=0
d(p
j
i
) =

i
j=0
(j + 1) =
(
j
+2)(
j
+1)
2
e
portanto, o n umero de tri angulos pitagoricos com um cateto igual
a M e
1
2
_
k

j=1
(
j
+ 2)(
j
+ 1)
2

k

j=1
_

j
+ 2
2
_
_
.
Se M e par, isto e, M = 2
a+1
N (coma 0) onde N = p

1
1
p

2
2
p

k
k
e mpar, o n umero de triplas pitagoricas com um cateto igual a M
e

m|M
C(m) =

m|N
C(m) +
a

j=1

m|N
C(2
j+1
m).
O primeiro somatorio ja foi calculado no item anterior e o segundo
e igual a
a

j=1

m|N
d(m) = a
k

j=1
(
j
+ 2)(
j
+ 1)
2
,
Portanto, o n umero de tri angulos pitagoricos com um cateto igual
a M = 2
a+1
N e
_
1
2
+a
_
k

j=1
(
j
+ 2)(
j
+ 1)
2

1
2
k

j=1
_

j
+ 1
2
_
.
4.8. N

UMEROS QUE S

AO SOMAS DE DOIS QUADRADOS 155


Problemas Propostos
4.25. Determine o n umero de tri angulos pitagoricos com um cateto de
comprimento 60. Observe que existem exatamente 4 triplas pitagoricas
primitivas em que um dos catetos vale 60.
4.26. Quantos tri angulos pitagoricos tem 2 3
k
como um de seus cate-
tos?
4.27. Quantos tri angulos pitagoricos tem 3
k
5
l
como um de seus catetos?
4.8 N umeros que sao somas de dois quadrados
Vamos mostrar algumas condi coes sobre os divisores primos de um
n umero que e soma de dois quadrados. Notemos, inicialmente, que 2 =
1
2
+ 1
2
. Quanto aos primos mpares, temos a
Proposi cao 4.8. Sejam a e b dois n umeros inteiros tais que mdc(a, b) =
1 e seja p um n umero primo mpar tal que p divide a
2
+b
2
, ent ao p deixa
resto 1 quando dividido por 4.
Demonstra c

ao: Primeiro observemos que p nao divide nem a nem


b, e que a
2
b
2
(mod p). Elevando ` a potencia
p1
2
temos que a
p1

(1)
(p1)/2
b
p1
(mod p), e pelo pequeno teorema de Fermat obtemos
que (1)
(p1)/2
1 (mod p). Portanto
p1
2
e um n umero par, isto e,
p 1 (mod 4).
A seguinte identidade mostra que para determinar quais n umeros
s ao somas de dois quadrados, basta identicar quais primos s ao soma de
dois quadrados.
Proposi cao 4.9. Sejam m e n n umeros que sao somas de dois quadra-
dos, ent ao mn tambem e soma de dois quadrados.
Demonstra c

ao: Suponhamos que m = a


2
+b
2
e n = c
2
+d
2
. Entao
mn = (a
2
+b
2
)(c
2
+d
2
) = a
2
c
2
+a
2
d
2
+b
2
c
2
+b
2
d
2
= (ac)
2
+ (ad)
2
+ (bc)
2
+ (bd)
2
+ 2abcd 2abcd
= ((ac)
2
+ 2abcd + (bd)
2
) + ((bc)
2
2abcd + (ad)
2
)
= (ac +bd)
2
+ (bc ad)
2
.
Portanto, seu produto tambem sera soma de dois quadrados.
156 CAP

ITULO 4. EQUA C

OES DIOFANTINAS N

AO LINEARES
Observa cao 4.10. Pelo mesmo processo podemos obter que
mn = (ac +bd)
2
+ (bc ad)
2
= (ac bd)
2
+ (bc +ad)
2
,
isto e, se m, n = 2 ent ao mn tem no mnimo duas representa coes como
soma de dois quadrados.
Em resumo, mostramos que todos os n umeros primos mpares que
dividem a
2
+b
2
com mdc(a, b) = 1 s ao da forma 4k+1 e se dois n umeros
s ao soma de dois quadrados entao seu produto tambem e. Assim, seria
interessante provar o recproco desta armacao; mas, para isto, precisa-
mos que todos os primos da forma 4k+1 sejam soma de dois quadrados.
Pesquisando os primeiros casos, vemos que
5 = 2
2
+ 1
2
, 13 = 3
2
+ 2
2
, 17 = 4
2
+ 1
2
, 29 = 5
2
+ 2
2
,
37 = 6
2
+ 1
2
, 41 = 5
2
+ 4
2
, 53 = 7
2
+ 2
2
, 61 = 6
2
+ 5
2
, etc
Assim, mostrando que todo primo da forma 4k+1 e soma de dois primos,
obteremos uma classicacao completa dos n umeros que s ao hipotenusa
de um tri angulo ret angulo com lados inteiros. Nessa sec ao mostraremos
usando ferramentas elementares que esta caracteriza cao e verdadeira. Na
secao 4.12.1 mostraremos este mesmo fato usando os chamados inteiros
de Gau.
De fato, para mostrar esta armacao precisamos antes dos seguintes
resultados
Lema 4.11 (Lema de Thue). Se m e um n umero natural e a e um
inteiro primo relativo com m, ent ao existem n umeros naturais x e y n ao
nulos menores que

m e tais que algum dos n umeros ax +y ou ax y
e divisvel por m.
Demonstra c

ao: No caso em que m = 1, para qualquer valor de a


teremos que x = y = 1 satisfazem as condi coes. Suponhamos que m seja
um n umero natural maior que 1. Seja q =

m. Entao q + 1 >

m
e portanto (q + 1)
2
> m. Consideremos todos os n umeros da forma
ax y onde x e y tomam os valores 0, 1, . . . , q. Observe que estamos
considerando (q + 1)
2
n umeros. Como s o existem m restos possveis ao
dividir um n umero por m, entao existem dois desses n umeros que tem
o mesmo resto quando divididos por m. Sejam ax
1
y
1
e ax
2
y
2
tais
n umeros. Portanto sua diferen ca a(x
1
x
2
) (y
1
y
2
) e divisvel por
m. Como
0 x
1
<

m e 0 x
2
<

m, entao

m < x
1
x
2
<

m ,
e da mesma forma
0 y
1
<

m e 0 y
2
<

m, entao

m < y
1
y
2
<

m.
COMPRIMENTO DA HIPOTENUSA DE UM TRI

ANGULO 157
Se x
1
x
2
= 0, entao y
1
y
2
sera divisvel por m, o que implicara
que y
1
= y
2
, mas os pares (x
1
, y
1
) e (x
2
, y
2
) s ao diferentes, logo isso
nos leva a uma contradi cao. Da mesma forma, se y
1
y
2
= 0, entao
a(x
1
x
2
) sera divisvel por m, mas a e m s ao primos relativos, assim
m divide (x
1
x
2
) e teremos x
1
= x
2
, contradi cao. Assim, x
1
x
2
= 0 e
y
1
y
2
= 0. Podemos supor, sem perda de generalidade, que x
1
x
2
> 0
e, neste caso, tomamos x = x
1
x
2
e y = |y
1
y
2
|, e conclumos a prova
do lema.
Agora sim podemos enunciar e mostrar o teorema central desta secao
Teorema 4.12. Todo primo da forma 4k + 1 pode-se escrever como
soma de dois quadrados de inteiros positivos.
Demonstra c

ao: Seja p um n umero primo da forma 4k + 1. Pelo


teorema de Wilson temos que
(p 1)! 1 (mod p),
mas
12
p 1
2
(1)(2)
_

p 1
2
_
(p1)(p2)
p + 1
2
(mod p)
ja que
p1
2
e par, portanto
__
p 1
2
_
!
_
2
1 (mod p).
Tomando a = (
1
2
(p 1))! entao p divide a
2
+ 1 e aplicando o lema de
Thue existem inteiros 0 < x, y <

p tais que algum dos n umeros axy
e divisvel por p, portanto o n umero (ax + y)(ax y) = a
2
x
2
y
2
e
divisvel por p. Daqui, segue que
x
2
+y
2
= a
2
x
2
+y
2
0 (mod p),
mas 0 < x, y <

p, logo 0 < x
2
+ y
2
< 2p e portanto se conclui que
p = x
2
+y
2
.
Podemos classicar, usando este teorema e a proposicao 4.8, os n u-
meros que podem ser comprimento da hipotenusa de um tri angulo re-
tangulo com lados inteiros. Observemos que para N ser hipotenusa de
um tri angulo temos que N deve ser da forma N = k(a
2
+ b
2
) onde a
e b s ao inteiros sem fator comum e de diferente paridade. Assim, os
n umeros que podem ser comprimento da hipotenusa de um tri angulo
ret angulo com lados inteiros s ao exatamente os n umeros que tem algum
fator primo congruente a 1 m odulo 4.
O seguinte resultado mostra que a decomposicao de um primo como
soma de dois quadrados de fato e unica:
158 CAP

ITULO 4. EQUA C

OES DIOFANTINAS N

AO LINEARES
Teorema 4.13. Seja n um n umero primo da forma 4k + 1. Ent ao n
pode ser escrito de forma unica como soma de dois quadrados.
Demonstra c

ao:
Suponhamos, por outro lado, que n seja primo e que n = a
2
+
b
2
= c
2
+ d
2
com a, b, c, d <

n. Note que necessariamente temos
mdc(a, b) = mdc(c, d) = 1. Armamos que, a menos de ordem, estas
somas s ao iguais. De fato, se a
2
b
2
(mod n) e d
2
c
2
(mod n),
entao (ad)
2
(bc)
2
(mod n), e como n e primo, temos que n divide
ad bc ou ad +bc.
No primeiro caso, sabemos que |ad bc| max{ad, bc} < n. Por-
tanto, adbc = 0, isto e,
a
b
=
c
d
; mas como as duas fra coes s ao reduzidas,
temos que a = c e b = d. No caso em que n divide ad + bc, observemos
que n
2
divide
(a
2
+b
2
)(c
2
+d
2
) = (ad +bc)
2
+ (ac bd)
2
.
Portanto n divide ac bd e, pelo mesmo argumento, obtemos que a = d
e b = c.
Como conclusao nal desta secao, temos que os unicos n umeros que
podem ser expressos como soma de dois quadrados de inteiros s ao os da
forma n = 2
s
d
2
l onde s e d s ao naturais e l e um produto de primos da
forma 4k + 1.
Problemas Propostos
4.28. Escrever 73, 89 e 97 como somas de dois quadrados.
4.29. Escrever 145 e 187 como somas de dois quadrados de duas formas
distintas.
4.30. De quantas formas distintas pode-se escrever 1001
2
como soma de
dois quadrados?
4.31. Seja p um n umero primo tal que a congruencia c
2
2 mod p
possui solu cao. Mostre que existem inteiros a e b tais que p = a
2
+ 2b
2
.
4.32. Demonstre que todo primo da forma 6k + 1 pode se expressar de
forma unica como x
2
+ 3y
2
, com x e y inteiros positivos.
4.33. Mostre que os n umeros da forma 4
k
(8n+7) n ao podem ser escritos
como soma de tres quadrados.
4.34 (Scholz). Prove a seguinte generalizacao do lema de Thue: Sejam
n um n umero natural positivo e r e s n umeros naturais tais que rs > n
com 1 < r, s < n. Ent ao, para todo a com (a, n) = 1, a congruencia
ay x (mod n) tem solu cao inteira com 0 < x < r e 0 < y < s.
4.9. TRI

ANGULOS PITAG

ORICOS COM CATETOS CONSECUTIVOS159


4.35. Demonstrar que todas as solu coes inteiras de x
2
+ y
2
+ z
2
= t
2
,
estao dadas pelas equacoes
x = d(m
2
n
2
p
2
+q
2
), y = d(2mn 2pq),
z = d(2mp + 2nq), t = d(m
2
+n
2
+p
2
+q
2
),
com d, m, n, p, q inteiros.
4.9 Triangulos pitagoricos com catetos conse-
cutivos
O problema de determinar todos os tri angulos com a hipotenusa e
um cateto consecutivos foi resolvido totalmente por Pit agoras. Agora
observemos que os tri angulos com lados (3, 4, 5) e (20, 21, 25) tem seus
catetos consecutivos. O prop osito desta secao e determinar todos os
tri angulos com os catetos consecutivos. Se os lados tem comprimentos
a, a + 1 e c, temos a equa cao
2a
2
+ 2a + 1 = c
2
,
a qual, multiplicando por 2 e completando quadrados, equivale a
(2a + 1)
2
2c
2
= 1.
Assim, o problema e equivalente a encontrar todas as solucoes inteiras
positivas da equa cao x
2
2y
2
= 1. Antes de encontrar as solucoes
desta equa cao estudaremos uma equa cao mais geral, conhecida como
equa cao de Pell.
4.9.1 Equa cao de Pell
Uma equa cao de Pell e uma equa cao da forma x
2
dy
2
= 1, onde
d e um n umero natural que nao e um quadrado perfeito. Por exemplo,
algumas solucoes da equa cao x
2
3y
2
= 1 s ao
(2, 1), (7, 4), (26, 15), (101, 56), . . . .
As solucoes desta equa cao s ao exatamente os pontos de coordenadas
inteiras da hiperbole x
2
3y
2
= 1, como mostra a gura
160 CAP

ITULO 4. EQUA C

OES DIOFANTINAS N

AO LINEARES
x
y
Suponhamos que conhecemos uma solucao(x
1
, y
1
) com x
1
e y
1
intei-
ros positivos (encontrada testando alguns valores ou pelo metodo apre-
sentado na seguinte secao). Vejamos como gerar innitas solucoes da
equa cao de Pell a partir desta solucao (x
1
, y
1
). Como x
2
1
dy
2
1
= 1,
fatorando a parte da direita e elevando `a potencia n temos
(x
1
y
1

d)
n
(x
1
+y
1

d)
n
= (x
2
1
dy
2
1
)
n
= (1)
n
.
Cada um destes fatores pode ser expandido usando o bin omio de Newton
(x
1
y
1

d)
n
=
n

i=0
_
n
i
_
x
ni
1
(y
1

d)
i
=

0in
i par
_
n
i
_
x
ni
1
(y
1

d)
i

0in
i mpar
_
n
i
_
x
ni
1
(y
1

d)
i
.
Portanto, denindo (x
n
, y
n
) Z
2
de forma que x
n
+ y
n

d = (x
1
+
y
1

d)
n
, ou seja, tomando
x
n
=

n
2

i=0
_
n
2i
_
x
n2i
1
d
i
y
2i
1
e y
n
=

n1
2

i=0
_
n
2i + 1
_
x
n2i1
1
d
i
y
2i+1
1
,
temos tambem x
n
y
n

d = (x
1
y
1

d)
n
, e obtemos que x
2
n
dy
2
n
=
(1)
n
. Assim, a partir de uma solucao nao trivial da equa cao de Pell
podemos gerar innitas solucoes.
Estas mesmas solucoes podem ser obtidas a partir de uma formula
recursiva. Para chegar a tal formula vejamos que
x
n+1
+y
n+1

d = (x
1
+y
1

d)
n+1
= (x
1
+y
1

d)
n
(x
1
+y
1

d)
= (x
n
+y
n

d)(x
1
+y
1

d),
4.9. TRI

ANGULOS PITAG

ORICOS COM CATETOS CONSECUTIVOS161


e igualando parte racional com parte racional e parte irracional com
parte irracional, temos as rela coes
_
x
n+1
= x
1
x
n
+dy
1
y
n
y
n+1
= x
1
y
n
+y
1
x
n
.
Por exemplo, a equa cao x
2
5y
2
= 1 tem como solucao (2, 1), as-
sim usando as equa coes recursivas
_
x
n+1
= 2x
n
+ 5y
n
y
n+1
= 2y
n
+x
n
, obtemos o par
(x
2
, y
2
) = (9, 4), que e solucao da equa cao x
2
5y
2
= (1)
2
= 1 e o
par (x
3
, y
3
) = (38, 17) que e solucao da equa cao original. Em outras
palavras, os termos de ordem mpar desta sequencia, (2, 1), (38, 17),
(682, 305), (12238, 5473) etc, s ao solucoes da equa cao x
2
5y
2
= 1 e os
termos de ordem par da sequencia, que s ao (9, 4), (161, 72), (2889, 1292),
(51841, 23184) etc, s ao solucoes da equa cao x
2
5y
2
= 1.
Por outro lado, dentre todas as solucoes (x, y) N
2
da equa cao
de Pell x
2
dy
2
= 1 com x + y

d > 1, existe uma solu cao mnima


ou fundamental, i.e., com x + y

d mnimo e como consequencia x e


y mnimos. Denote por (x
1
, y
1
) esta solucao mnima. Se, como antes,
denimos (x
n
, y
n
) N
2
pela rela cao x
n
+ y
n

d = (x
1
+ y
1

d)
n
, temos
que (x
n
, y
n
), n 1, s ao todas as solucoes inteiras positivas da equa cao
de Pell x
2
dy
2
= 1: de fato, ja vimos que (x
n
, y
n
) s ao solucoes, e se
(a, b) e uma outra solucao, entao como x
1
+ y
1

d > 1, existe n 0 tal


que
(x
1
+y
1

d)
n
a +b

d < (x
1
+y
1

d)
n+1
.
Multiplicando por x
n
y
n

d = (x
1
+y
1

d)
n
> 0, obtemos
1 (a +b

d)(x
n
y
n

d) = (ax
n
by
n
d) + (bx
n
ay
n
)

d
< x
1
+y
1

d.
Como
(ax
n
dby
n
)
2
d(bx
n
ay
n
)
2
= (a
2
db
2
)(x
2
n
dy
2
n
) = 1,
temos que (ax
n
dby
n
, bx
n
ay
n
) tambem e uma solucao da equa cao
de Pell, menor que a solucao mnima. Temos que ax
n
dby
n
0, pois
caso contrario ax
n
dby
n
< 0
a
b
x
n
y
n
< d, porem
x
2
n
dy
2
n
= 1 =
_
x
n
y
n
_
2
= d +
1
y
2
n
> d =
x
n
y
n
>

d
162 CAP

ITULO 4. EQUA C

OES DIOFANTINAS N

AO LINEARES
e analogamente
a
b
>

d, o que contradiz
a
b
x
n
y
n
< d. Da mesma forma,
bx
n
ay
n
0, pois caso contrario,
x
n
y
n
<
a
b
= d +
1
y
2
n
=
_
x
n
y
n
_
2
<
_
a
b
_
2
= d +
1
b
2
= b < y
n
= a < x
n
o que contradiz o fato de x
n
+ y
n

d = (x
1
+ y
1

d)
n
a + b

d. Re-
sumindo, temos que (ax
n
dby
n
, bx
n
ay
n
) N
2
e uma solucao menor
do que a solucao mnima, logo ax
n
dby
n
= 1 e bx
n
ay
n
= 0, ou
seja, (a + b

d)(x
1
y
1

d)
n
= 1 a + b

d = x
n
+ y
n

d, donde
(a, b) = (x
n
, y
n
), como queramos.
Voltando ao problema inicial de encontrar todos os tri angulos ret an-
gulos com lados inteiros tal que os catetos sejam inteiros consecutivos,
isto e, com lados a, a + 1 e hipotenusa c tinhamos obtido a rela cao
(2a + a)
2
2c
2
= 1. Portanto, este problema se reduz a estudar a
equa cao x
2
2y
2
= 1 que tem solucao fundamental 1 +

2, assim
todas as solucoes s ao da forma
x
n
=
(1 +

2)
2n+1
+ (1

2)
2n+1
2
e y
n
=
(1 +

2)
2n+1
(1

2)
2n+1
)
2

2
.
Conclumos que todas as solucoes s ao da forma
a
n
=
(1 +

2)
2n+1
+ (1

2)
2n+1
2
4
,
b
n
= a
n
+ 1,
c
n
=
(1 +

2)
2n+1
(1

2)
2n+1
)
2

2
para todo n N. As primeiras solucoes s ao mostradas na tabela a seguir
n a
n
b
n
c
n
1 3 4 5
2 20 21 29
3 119 120 169
4 696 697 985
5 4059 4060 5741
6 23660 23661 33461
Exemplo 4.14. Mostre que existem innitos valores de n tais que a
soma dos primeiros n inteiros positivos e igual ao quadrado de um in-
teiro.
Solu cao. Observemos que 1+2+ +n =
n(n+1)
2
. O problema se re-
duz a encontrar inteiros (n, m) tais que
n(n+1)
2
= m
2
, o que e equivalente
4.9. TRI

ANGULOS PITAG

ORICOS COM CATETOS CONSECUTIVOS163


a n(n + 1) = 2m
2
. Multiplicando a equa cao anterior por 4 e comple-
tando quadrados obtemos a equa cao (2n + 1)
2
8m
2
= 1. Assim basta
encontrar as solucoes da equa cao de Pell x
2
8y
2
= 1. Esta equa cao tem
como solucao fundamental (3, 1), e portanto possui innitas solucoes. As
seguintes tres solucoes podem ser calculadas facilmente, pois
(3 +

8)
2
= 17 + 6

8,
(3 +

8)
3
= 99 + 35

8
(3 +

8)
2
= 577 + 204

8.
Assim 2n + 1 pode ser 3, 17, 99, 577, . . . , logo n = 1, 8, 49, 288, . . . .
A seguinte proposicao mostra uma recorrencia simples de segunda
ordem que gera todas as solucoes da equa cao de Pell a partir das duas
primeiras.
Proposi cao 4.15. Seja a + b

d a solu cao fundamental da equacao de


Pell x
2
dy
2
= 1. Se {(x
n
, y
n
)}
nN
e a sequencia de solu coes da equacao,
ent ao
x
n+2
= 2ax
n+1
x
n
y
n+2
= 2ay
n+1
y
n
,
para todo n 1.
Demonstra c

ao: Como ja foi mostrado anteriormente temos que a


sequencia de solucoes satisfaz a recorrencia
_
x
n+1
= ax
n
+dy
n
y
n+1
= bx
n
+ay
n
.
. As-
sim temos que
x
n+2
= ax
n+1
+dby
n+1
= ax
n+1
+db(bx
n
+ay
n
) = ax
n+1
+db
2
x
n
+a(dby
n
)
= ax
n+1
+db
2
x
n
+a(x
n+1
ax
n
) = 2ax
n+1
(a
2
db
2
)x
n
= 2ax
n+1
x
n
,
o que prova a primeira recorrencia. A segunda recorrencia e obtida de
modo an alogo e e deixada como exerccio ao leitor.
Se aplicarmos estas formulas no problema anterior, podemos compro-
var diretamente que 577 = 2 3 99 17 e 204 = 2 3 35 6, assim nao
e necessario calcular a expans ao dos bin omios para calcular as solucoes
de forma recursiva.
164 CAP

ITULO 4. EQUA C

OES DIOFANTINAS N

AO LINEARES
Problemas Propostos
4.36. Observe que 8
3
7
3
= 13
2
. Mostre que existem innitos pares de
cubos consecutivos tais que sua diferen ca e um quadrado perfeito.
4.37. Mostre que a soma de tres quadrados consecutivos n ao pode ser
igual a um quadrado. Pode a soma de cinco quadrados consecutivos ser
igual a um quadrado?
4.10 Solu cao fundamental da equacao de Pell
Para determinar uma solucao da equa cao x
2
Ay
2
= 1, vamos
considerar a fracao contnua de

A +

A = [a
0
; a
1
, a
2
, . . .]. Mos-
traremos que existem duas sequencias de inteiros positivos b
i
e c
i
de
modo que

A+c
i
b
i
= [a
i
; a
i+1
, a
i+2
. . .]
para todo i 0, onde os b
i
e c
i
s ao limitados por uma constante que
depende unicamente de A, assim em algum momento tais sequencias
comecam a se repetir. Come camos denindo b
0
= 1 e c
0
=

A. Em
geral, denimos recursivamente c
i+1
= a
i
b
i
c
i
e b
i+1
= (Ac
2
i+1
)/b
i
.
Mostremos inicialmente por indu cao que b
i
e c
i
s ao inteiros com
b
i
= 0 e tais que b
i
| A c
2
i
para todo i. Isto e claramente verdade
para i = 0. Por hip otese de indu cao, temos que b
i
e c
i
s ao inteiros, logo
c
i+1
= a
i
b
i
c
i
tambem sera inteiro e A c
2
i+1
= 0 ja que A nao e
quadrado perfeito. Alem disso,
Ac
2
i+1
= A(a
i
b
i
c
i
)
2
= Ac
2
i
b
i
(a
2
i
b
i
2a
i
c
i
)
sera m ultiplo de b
i
ja que b
i
| A c
2
i
por hip otese de indu cao. Assim
b
i+1
= (Ac
2
i+1
)/b
i
sera um inteiro nao nulo tal que b
i+1
| Ac
2
i+1
.
Desta forma, temos

A+c
i
b
i
= a
i
+

Ac
i+1
b
i
= a
i
+
b
i+1

A+c
i+1
= a
i
+
1

A+c
i+1
b
i+1
de modo que

A+c
i+1
b
i+1
= [a
i+1
; a
i+2
, a
i+3
. . .], e portanto sera valida para
todo i. Resta provar que b
i
e c
i
s ao positivos. Para isto, vamos provar
por indu cao que b
i
> 0 e 0 < c
i
<

A, o que e verdadeiro para i = 0


pois c
0
=

A e A nao e quadrado perfeito. Alem disso, pela denicao


de a
i
temos
a
i
<

A+c
i
b
i
= [a
i
; a
i+1
, a
i+2
. . .] < a
i
+ 1
4.10. SOLU C

AO FUNDAMENTAL DA EQUA C

AO DE PELL 165
donde obtemos a
i
b
i
<

A+c
i
< a
i
b
i
+b
i
(ja que b
i
> 0 por hip otese de
indu cao) e portanto
c
i+1
= a
i
b
i
c
i
<

A < a
i
b
i
c
i
+b
i
= c
i+1
+b
i
e assim c
i+1
<

A, o que implica b
i+1
= (A c
2
i+1
)/b
i
> 0 tambem.
Agora suponha por absurdo que c
i+1
0. Neste caso teramos b
i
>

Ac
i+1

A, mas como

A > c
i
por hip otese de indu cao, teramos
b
i
> c
i
, donde c
i+1
= a
i
b
i
c
i
b
i
c
i
> 0, o que e uma contradi cao.
Portanto c
i+1
> 0, completando a indu cao.
Como 0 < c
i
<

A e b
i
| A c
2
i
, temos que as sequencias {c
i
}
e {b
i
} s o assumem um n umero nito de valores. Alem disso, como
b
i
= (A c
2
i+1
)/b
i+1
e c
i
= a
i
b
i
c
i+1
podemos recuperar os valores de
b
i
e c
i
a partir dos de b
i+1
e c
i+1
. Portanto estas duas sequencias, assim
como a fracao contnua

A +

A = [a
0
; a
1
, a
2
, . . .], s ao peri odicas
puras, digamos de perodo k. Em particular b
k
= 1 e c
k
= a
0
.
Note que como a
0
= 2

A, temos que a expans ao em fra cao cont-


nua de

A e [a
0
/2; a
1
, a
2
, . . .]. Logo, para i 1, denotando por p
i
/q
i
a
i-esima convergente desta fra cao contnua, temos

A =

A+c
i+1
b
i+1
p
i
+p
i1

A+c
i+1
b
i+1
q
i
+q
i1
,
e portanto
Aq
i
+c
i+1

Aq
i
+

Ab
i+1
q
i1
=

Ap
i
+c
i+1
p
i
+b
i+1
p
i1
.
Separando parte racional da parte irracional obtemos as equa coes
Aq
i
= c
i+1
p
i
+b
i+1
p
i1
e p
i
= c
i+1
q
i
+b
i+1
q
i1
.
Isolando c
i+1
nas equa coes anteriores e igualando obtemos
Aq
i
b
i+1
p
i1
p
i
=
p
i
b
i+1
q
i1
q
i
Aq
2
i
b
i+1
p
i1
q
i
= p
2
i
b
i+1
q
i1
p
i
p
2
i
Aq
2
i
= b
i+1
(p
i
q
i1
p
i1
q
i
)
p
2
i
Aq
2
i
= (1)
i+1
b
i+1
donde obtemos uma solucao da equa cao x
2
Ay
2
= (1)
i+1
b
i+1
. Se k e
o perodo teremos que b
k
= 1 e portanto a equa cao x
2
Ay
2
= 1 tem
solucao se k e mpar, enquanto que x
2
Ay
2
= 1 sempre tem solucao
(tomando i + 1 = 2k).
Exemplo 4.16. Determine a solu cao mnima da equacao x
2
13y
2
= 1.
166 CAP

ITULO 4. EQUA C

OES DIOFANTINAS N

AO LINEARES
Solu c

ao: Sejam [a
0
; a
1
, a
2
, a
3
, . . . ] a fra cao contnua de
0
=

13+3.
Os valores a
j
podem ser calculados recursivamente usando as rela coes
a
j
=
j
,
j+1
=
1

j
a
j
. Assim neste caso
j a
j

j
0 6

13 + 3
1 1

13+3
4
2 1

13+1
3
3 1

13+2
3
4 1

13+1
4
5 6

13 + 3
Observemos que a linha 0 e igual `a linha 5, assim os n umeros vao se
repetir, portanto

13 + 3 = [6; 1, 1, 1, 1] e

13 = [3; 1, 1, 1, 1, 6], assim


podemos calcular os primeiro 5 convergentes de

13 obtendo
p
0
q
0
=
3
1
,
p
1
q
1
=
4
1
,
p
2
q
2
=
7
2
,
p
3
q
3
=
11
3
,
p
4
q
4
=
18
5
logo p
2
4
13q
2
4
= 18
2
135
2
= 1 e a solucao fundamental da equa cao
x
2
13y
2
= 1. Para obter a solucao fundamental de x
2
13y
2
= 1
basta calcular o decimo convergente, de fato
p
5
q
5
=
119
33
,
p
6
q
6
=
137
38
,
p
7
q
7
=
256
71
,
p
8
q
8
=
393
109
,
p
9
q
9
=
649
180
,
segue que 649
2
13 180
2
= 1 e a solucao mnima desta equa cao.
Observemos que podemos chegar a este mesmo valor pegando a solucao
18+5

13 e elevando ao quadrado, isto e, (18+5

13)
2
= 649+180

13.
Problemas Propostos
4.38. Calcule a a solu cao fundamental da equacao de Pell x
2
dy
2
= 1,
onde d = a
2
2 e d = a
2
a para a inteiro arbitr ario.
4.39. Determine a fracao contnua de

a
2
+ 2, e determine para que
valores de a a equacao x
2
(a
2
+ 2)y
2
= 1 tem solu cao.
4.40. Determine a solu cao fundamental da equacao x
2
31y
2
= 1.
4.41. Determine a solu cao fundamental da equacao x
2
41y
2
= 1.
4.11. OUTRAS EQUA C

OES DO TIPO X
2
AY
2
= C 167
4.11 Outras equacoes do tipo x
2
Ay
2
= c
Foi mostrado que a equa cao x
2
Ay
2
= 1 sempre possui solucao.
Por outro lado a equa cao x
2
Ay
2
= 1 s o possui solucao se o perodo
da fracao contnua de

A tem comprimento mpar. O seguintes teore-


mas respondem de forma simples quando esta equa cao tem ou nao tem
solucao para alguns valores particulares de A.
Teorema 4.17. Se A tem um divisor primo da forma 4k + 3, ent ao a
equacao x
2
Ay
2
= 1 n ao tem solu cao inteira positiva.
Demonstra c

ao: Seja p um n umero primo da forma 4k +3 que divide


A. Se esta equa cao tiver solucao, entao a congruencia x
2
x
2
Ay
2

1 (mod p) teria solucao. Mas elevando `a potencia


p1
2
, obtemos que
x
p1
(1)
(p1)/2
1 (mod p), logo pelo teorema de Fermat obte-
mos que 1 1 (mod p), o que e contradit orio.
Teorema 4.18. Seja p um primo da forma 4k + 1. Ent ao a equacao
x
2
py
2
= 1 sempre possui solu cao.
Demonstra c

ao: Suponhamos que (a, b) e solucao fundamental da


equa cao x
2
py
2
= 1. Como a
2
pb
2
a
2
b
2
1 (mod 4) temos que
a e mpar e b e par, e como a
2
1 = pb
2
temos que
a + 1
2
a 1
2
= p
_
b
2
_
2
.
Como os n umeros
a+1
2
e
a1
2
s ao primos entre si, temos duas possibili-
dades:
a + 1 = 2f
2
e a 1 = 2pg
2
. Subtraindo estas duas equa coes
obtemos f
2
pg
2
= 1, isto e, o par (f, g) tambem e solucao de
x
2
py
2
= 1 menor que a solucao (a, b), o que e impossvel ja que
esta e a solucao fundamental, logo este caso nao pode acontecer.
a + 1 = 2pf
2
e a 1 = 2g
2
. Subtraindo estas duas equa coes
obtemos g
2
pf
2
= 1, como queramos mostrar.
Exemplo 4.19. Sabendo que x
2
29y
2
= 1 possui como solu cao fun-
damental x = 9801 e y = 1820, determine a solu cao fundamental de
x
2
29y
2
= 1.
Solu c

ao: Pela prova do teorema anterior sabemos que a equa cao


x
2
29y
2
= 1 possui uma solucao (g, f) com 2g
2
= 9801 1 = 9800 e
229f
2
= 9801+1, portanto g =

4900 = 70 e f =
_
9802
58
=

169 = 13.
168 CAP

ITULO 4. EQUA C

OES DIOFANTINAS N

AO LINEARES
A seguinte proposicao ajuda a reduzir o trabalho necessario para
decidir se a equa cao mais geral x
2
Ay
2
= c possui solucoes inteiras
positivas.
Proposi cao 4.20. Seja = x
1
+ y
1

A > 1 onde (x
1
, y
1
) e a solu cao
mnima de x
2
Ay
2
= 1. Dado c Z n ao nulo, se existem x, y N
com x
2
Ay
2
= c, ent ao existem u, v N com u + v

A
_
|c|
e u
2
Av
2
= c (em particular, para esta solu cao 0 u
_
|c| e
0 v
_
|c|/A).
Demonstra c

ao: Se = r+s

A com r, s Q, denimos = rs

A
e N() = N( ) = = r
2
As
2
.
Seja = x+y

A > 0, com N() = x


2
Ay
2
= c. Entao N(
k
) =
c para todo k Z. Em particular podemos escolher um k Z tal
que
_
|c| <
k

_
|c|. No caso que
_
|c| <
k

_
|c|
denimos =
k
e no caso que
_
|c| <
k

_
|c|, podemos
denir = |c|/(
k
) =

1k
onde =
c
|c|
{1, 1}, assim
N() = N(

) = N() = c e
_
|c| <
_
|c|. Logo, sem perda de
generalidade, podemos supor que
_
|c|
_
|c|.
Assim temos que = u + v

A com u, v Z e ainda precisamos


vericar que u, v s ao naturais, mas
c = N() = u
2
Av
2
= (u +v

A)(u v

A).
Temos entao
|u v

A| =
|c|
u +v

|c|
_
|c|
=
_
|c| u +v

A.
Temos assim u v

A u + v

A, donde v 0 e simultaneamente
u +v

A u +v

A, e logo u 0.
Exemplo 4.21. Determine se a equacao x
2
10y
2
= 39 possui solu cao.
Solu c

ao: A equa cao x


2
10y
2
= 1 possui como solucao fundamental
19 + 6

10. Como
_
39(19 + 6

10) < 39 e
_
39(19+6

10)
10
< 13, pela
proposicao anterior, e do fato que x
2
> 39, temos que uma possvel
solucao satisfaz 7 x 39 com x mpar e y 12, assim s o precisamos
testar 12 valores para y. De fato com y = 1, obtemos x = 7, e portanto
a equa cao possui solucao.
Exemplo 4.22. Mostre que a equacao x
2
10y
2
= 11 n ao possui solu cao
inteira.
4.12. CONTANDO TRI

ANGULOS PITAG

ORICOS COM HIPOTENUSA FIXAD


Solu c

ao: Pelo mesmo processo anterior temos que


_
11(19 + 6

10) < 21 e

11(19 + 6

10)
10
< 7.
Considerando a equa cao m odulo 4 obtemos x
2
+ 2y
2
3 (mod 4), mas
esta rela cao somente e possvel quando x e y s ao mpares, portanto, se
a equa cao tem solucao, uma das solucoes deve satisfazer 4 x 20,
y 6 com x e y mpares, assim s o precisamos testar y = 1, 3, 5, mas
como nenhum dos n umeros 11+10 = 21, 11+90 = 101 e 11+250 = 261
e um quadrado perfeito, conclumos que a equa cao nao possui solucoes
inteiras.
Problemas Propostos
4.42. Determine se a equacao x
2
19y
2
= 21 possui solu cao inteira.
4.43. Determine todos os |c| < 10 tais que x
2
17y
2
= c possui solu cao
inteira.
4.44. Usando o fato de que a equacao x
2
91y
2
= 1 tem como solu cao
fundamental o par (1574, 165), mostre que a equacao 7x
2
13y
2
= 1 n ao
possui solu coes inteiras.
4.12 Contando triangulos pitagoricos com hipo-
tenusa xada
Para todo c n umero natural, denotemos por S
2
(c) o n umero de pares
de naturais (m, n) com mdc(m, n) = 1, m < n e m + n mpar tais que
c = m
2
+ n
2
(note que, se S
2
(c) = 0, entao n e mpar) e denotemos
por T
2
(c) o n umero de triplas de naturais (k, m, n) com mdc(m, n) = 1,
1 m < n e m + n mpar tais que c = k(m
2
+ n
2
), isto e S
2
(c) e
o n umero de triplas pitagoricas primitivas com hipotenusa c e T
2
(c) e
o n umero de triplas pitagoricas com hipotenusa c. Denotemos ainda
por U
2
(c) o n umero de pares de naturais (m, n) com m n tais que
c = m
2
+ n
2
. Pelo que foi mostrado anteriormente, se c possui um
fator primo congruente a 3 (mod 4) entao S
2
(k) = 0. Por outro lado,
se p e um primo congruente a 1 (mod 4) pelo teorema 4.13 temos que
T
2
(p) = S
2
(p) = 1.
Para determinar uma formula fechada para S
2
(c) e T
2
(c), precisare-
mos introduzir alguns resultados sobre inteiros de Gau. Antes de fazer
isso, resolvamos o seguinte problema:
170 CAP

ITULO 4. EQUA C

OES DIOFANTINAS N

AO LINEARES
Exemplo 4.23. Determine todas as triplas pitagoricas com hipotenusa
325.
Solu c

ao: Sabemos que cada tripla de inteiros k,me n com mdc(m, n) =


1 e m+nmpar gera uma unica tripla pitagorica (a, b, c) com a = 2kmn,
b = k(m
2
n
2
) e c = k(m
2
+ n
2
). Assim precisamos encontrar triplas
(k, m, n) tais que k(m
2
+ n
2
) = 325 = 5
2
13. Como m
2
+ n
2
e um
divisor de 325 distinto de 1, precisamos encontrar as solucoes de
m
2
+n
2
{5, 13, 25, 65, 325}.
De fato, 1
2
+2
2
= 5, 2
2
+3
2
= 13, 3
2
+4
2
= 25, 1
2
+8
2
= 65, 4
2
+7
2
= 65,
6
2
+ 17
2
= 325 e 1
2
+ 18
2
= 325, s ao as unicas solucoes, que geram as
triplas pitagoricas
(195, 260, 325), (125, 300, 325), (91, 312, 325),
(80, 315, 325), (165, 280, 325), (204, 253, 325), (36, 323, 325)
4.12.1 Inteiros de Gau
Os inteiros de Gau s ao os elementos do conjunto
Z[i]
def
= {a +bi C | a, b Z}
subconjunto dos n umeros complexos (onde i
2
= 1). Este conjunto e
fechado para as operacoes soma e produto de n umeros, isto e, soma de
dois interios de Gau e um inteiro de Gau e produto de inteiros de Gauss
e um inteiro de Gau. Alem disso e com respeito a soma, todo elemento
tem inverso aditivo, assim este conjunto junto com estas operacoes e
chamado anel dos inteiros de Gau. Este anel cumpre propriedades
equivalentes ao anel de inteiros Z como mostraremos nesta secao. Em
particular, podemos denir um algoritmo da divisao, e todo n umero
pode-se fatorar como produto de primos, assim neste anel podemos
desenvolver uma aritmetica similar `a aritmetica dos n umeros inteiros.
Denimos a norma de um inteiro de Gau como
N: Z[i] Z
z = a +bi |z|
2
= zz = a
2
+b
2
.
Como foi vericado anteriormente a norma cumpre que N(wz) = N(w)N(z)
para todo w, z Z[i], isto e, N e uma funcao multiplicativa.
4.12. CONTANDO TRI

ANGULOS PITAG

ORICOS COM HIPOTENUSA FIXAD


Inteiros de Gau possuem propriedades aritmeticas muito similares
`as dos inteiros. Por exemplo, podemos denir divisibilidade da maneira
usual:
| existe Z[i] tal que = .
Assim, por exemplo, temos que 1 +i | 5 +3i pois 5 +3i = (1 +i)(4 i).
Note que | = N() | N(). Mais geralmente, temos
Lema 4.24 (Divisao Euclidiana). Sejam , Z[i] com = 0. Ent ao
existem inteiros de Gau q, r Z[i] tais que
= q +r com N(r) < N().
Demonstra c

ao: Escreva

= x + yi com x, y Q. Agora sejam


m, n Z os inteiros mais proximos de x e y, ou seja, m e n s ao tais que
|x m|
1
2
e |y n|
1
2
. Agora basta tomar q = m+ni e r = q,
pois temos

2
= |(x m) + (y n)i|
2
= (x m)
2
+ (y n)
2

1
4
+
1
4
< 1.
Multiplicando por ||
2
, temos portanto
N(r) = | q|
2
< ||
2
= N().
Note que, ao contrario da divisao euclidiana em inteiros, o quociente e
o resto na divisao em Z[i] nem sempre estao unicamente determinados.
Por exemplo, dividindo-se = 5 por = 1 + i, temos mais de uma
possibilidade:
5 = (1 +i)(2 2i) + 1 com 1 = N(1) < N(1 +i) = 2
5 = (1 +i)(2 3i) +i com 1 = N(i) < N(1 +i) = 2.
Exemplo 4.25. Escreva 21 i como produto de dois inteiros de Gau
n ao triviais.
Solu c

ao: Queremos encontrar a, b, c e d tais que (a + ib)(c + id) =


21 i, logo utilizando a funcao norma temos que (a
2
+ b
2
)(c
2
+ d
2
) =
21
2
+1
2
= 442 = 17 26. Fazendo a
2
+b
2
= 17 e c
2
+d
2
= 26, obtemos
solucoes para este problema. Dentre estas solucoes, temos a +ib = 4 i
e c + id = 5 + i que fornecem uma solucao do problema original, pois
21 i = (4 i)(5 +i).
172 CAP

ITULO 4. EQUA C

OES DIOFANTINAS N

AO LINEARES
Ja que existe um algoritmo de divisao em Z[i] podemos denir mdc
de dois elementos de Z[i]. Em particular, dizemos que dois n umeros
em Z[i] s ao primos entre si se os seus unicos divisores comuns s ao os
elementos invertveis de Z[i]. Em geral temos a seguinte
Denicao 4.26. Seja A um domnio, isto e, um anel em que (ab =
0 = a = 0 ou b = 0). Dizemos que um elemento u A e uma
unidade (ou um elemento invertvel) se ele possui inverso multiplicativo
em A, isto e, existe v A tal que uv = 1. O conjunto de todas as
unidades de A com a operacao de produto e um grupo multiplicativo, o
grupo de unidades de A, que denotamos por A

.
Por exemplo, em A = Z, as unidades s ao 1 e se A = Z[i] os unicos
elementos invertveis s ao {1, i} como se mostrara no lema 4.31.
Por outro lado, dado que em Z[i] temos um algoritmo de divisao,
entao tambem temos um equivalente ao teorema de Bachet-Bezout como
se mostra a seguir.
Teorema 4.27 (Bachet-Bezout). Sejam e dois elementos em Z[i]
primos entre si, isto e, cujos unicos divisores comuns sao unidades. En-
t ao existem x, y Z[i] tais que
x +y = 1.
Demonstra c

ao: Consideremos o conjunto C = {x +y|x, y Z[i]}


e sejam = x
0
+ y
0
C nao nulo e com norma mnima (observe
que o conjunto das normas e um subconjunto de N e portanto esta bem
ordenado). Vejamos que divide e . Caso contrario, suponhamos
que nao divide : assim existem , Z[i] tais que = + , com
0 < N() < N(). Portanto
= = (1 x
0
) (y
0
)
tambem pertence a C, o que e contradiz a minimalidade de N(), e logo
divide . Simetricamente temos que divide . Como e s ao
primos entre si, temos que N() = 1, e assim, multiplicando por caso
necessario, podemos supor que = 1, como queriamos mostrar.
Exemplo 4.28. Calcule mdc(17 + 51i, 210 + 214i)
Solu c

ao: Aplicando o algoritmo extendido da divisao para inteiros de


Gau temos
210 + 214i = (2 + 5i) (17 + 51i) + (11 + 27i)
17 + 51i = (2 + 0i) (11 + 27i) + (5 3i)
11 + 27i = (4 3i) (5 3i).
Poranto mdc(17 + 51i, 210 + 214i) = 5 + 3i.
4.12. CONTANDO TRI

ANGULOS PITAG

ORICOS COM HIPOTENUSA FIXAD


Problemas Propostos
4.45. Calcule um resto da divisao de 38 + 65i por 7 5i.
4.46. Determine mdc(232 + 156i, 371 + 223i).
4.47. Use o algarismo extendido da divisao para determinar as solu coes
x, y Z[i] de (13 + 10i)x + (7 + 20i)y = 1.
Podemos tambem denir congruencias para inteiros de Gau:
(mod ) | .
As mesmas demonstra coes do caso de Z mostram que congruencias m o-
dulo denem uma rela cao de equivalencia em Z[i] compatvel com a
soma, a subtracao e o produto. Podemos portanto formar o anel quo-
ciente Z[i]/(), cujos elementos s ao as classes de congruencia m odulo
.
Exemplo 4.29. Mostre que (1 +i)
2009
+1 e divisvel por 2 +i em Z[i].
Solu c

ao: Temos que 2 + i 0 (mod 2 + i) i 2 (mod 2 +


i) 1 +i 1 (mod 2 +i). Logo (1 +i)
2009
(1)
2009
(mod 2 +
i) (1 +i)
2009
+ 1 0 (mod 2 +i).
O proximo passo e generalizar o conceito de primo:
Denicao 4.30. Dizemos que A\ {0} e irredutvel se ele n ao pode
ser escrito como produto de dois elementos em A\A

. Dois irredutveis

1
e
2
sao ditos associados se eles diferem por multiplicacao por uma
unidade:
1
= u
2
com u A

.
Por exemplo, em A = Z, os n umeros da forma p, onde p e um
n umero primo s ao primos associados. Intuitivamente, elementos associ-
ados devem ser vistos nao como primos distintos mas como um unico
primo para efeitos de fatora cao.
Primos emZ nao necessariamente s ao irredutveis emZ[i]. Por exem-
plo, temos que 5 = (2 +i)(2 i). Por outro lado, 2 +i e 2 i possuem
norma N(2 i) = 5 prima e logo s ao irredutveis pelo seguinte
Lema 4.31. 1. Z[i]

= {1, i}. Em particular u Z[i]


N(u) = 1.
2. Se Z[i] e tal que N() e um n umero primo, ent ao e irredu-
tvel.
3. Se p Z e um primo p 3 (mod 4), ent ao p e irredutvel em Z[i].
174 CAP

ITULO 4. EQUA C

OES DIOFANTINAS N

AO LINEARES
Demonstra c

ao:

E facil vericar que 1, i s ao unidades. Por outro
lado, se u Z[i]

, entao existe v Z[i] tal que uv = 1, logo N(u)N(v) =


1. Como N(u), N(v) s ao inteiros positivos, temos N(u) = N(v) = 1.
Escrevendo u = a+bi com a, b Z, temos que N(u) = 1 a
2
+b
2
=
1 (a, b) = (1, 0) ou (a, b) = (0, 1), ou seja, u {1, i}.
Agora suponha que N() seja primo. Se = com , Z[i]
entao N() = N()N(). Como N() e primo, ou N() = 1 ou N() =
1, ou seja, ou ou e uma unidade e portanto e irredutvel.
Finalmente, seja p 3 (mod 4). Se p pode ser fatorado como p =
com , Z[i] \ Z[i]

, temos p
2
= N(p) = N()N(). Como e nao
s ao unidades, N() = 1 e N() = 1, logo N() = N() = p. Porem,
escrevendo = a +bi com a, b Z, temos que a
2
+b
2
= p 3 (mod 4),
o que e impossvel, visto que um quadrado perfeito e congruente a 0 ou
a 1 m odulo 4, logo a
2
+b
2
e congruente a 0, 1 ou 2 m odulo 4, mas nunca
a 3 m odulo 4.
Note que, como no caso de Z, o teorema de Bachet-Bezout implica
Lema 4.32. Seja Z[i] um elemento irredutvel. Ent ao
| = | ou |
para , Z[i].
Como corolario, obtemos a fatora cao unica:
Teorema 4.33 (Fatoracao unica). Qualquer elemento = 0 de Z[i]
admite uma fatoracao
=
1

2
. . .
n
em elementos irredutveis
i
. Tal fatoracao e unica a menos da ordem
dos fatores e de multiplicacao por unidades (isto e, a menos de associa-
dos).
Demonstra c

ao: A prova da unicidade da fatora cao e identica `a dos


inteiros, utilizando o lema anterior. A prova da existencia da fatora cao
e tambem similar, mas agora utilizamos indu cao em N(): se N() = 2
(base) entao e irredutvel (ver lema) e se e irredutvel, nao ha nada a
fazer; caso contrario, existe uma fatora cao = onde nem nem s ao
unidades, isto e, N() = 1 e N() = 1. Como N() = N()N(), temos
que e possuem norma estritamente menor do que N(). Por hip otese
de indu cao, e podem ser fatorados em irredutveis, e combinando as
duas fatora coes temos uma fatora cao de .
4.12. CONTANDO TRI

ANGULOS PITAG

ORICOS COM HIPOTENUSA FIXAD


Exemplo 4.34. Escreva 50 como produto de irredutveis em Z[i].
Solu c

ao: Como 50 = 2 5
2
e ja sabemos fatorar 5 = (2 + i)(2 i)
em irredutveis, basta agora fatorar 2. Temos que 2 = (1 + i)(1 i) =
i(1 i)
2
e 1 i e irredutvel pois sua norma N(1 i) = 2 e prima. Logo
50 = i(1 i)
2
(2 +i)
2
(2 i)
2
e a fatora cao em irredutveis de 50.
Exemplo 4.35. Determine todas as solu coes inteiras da equacao
x
2
+y
2
= z
3
, com mdc(x, y) = 1.
Solu c

ao: Fatorando em inteiros de Gau temos que (x+iy)(xiy) =


z
3
. Alem disso, temos
mdc(x+iy, xiy) = mdc(2x, xiy)| mdc(2x, 2x2iy) = 2 mdc(x, y) = 2.
O mdc destes dois n umeros nao pode ser 2, porque nesse caso x e y
seriam divisveis por 2. Assim temos dois possveis casos
Se mdc(x+iy, xiy) = 1, entao cada um e o cubo de um inteiro de
Gau, isto, e x+iy = (a+ib)
3
, portanto x = a
2
3ab
2
, y = 3abb
3
e z = a
2
+ b
2
com mdc(a, b) = 1 e com a + b mpar (de fato, se
a + b fosse par, entao a + ib e x + iy seriam m ultiplos de 1 + i, e
teramos 1 +i | mdc(x +iy, x iy), contradi cao).
Se mdc(x +iy, x iy) = 1 +i teramos x e y da mesma paridade,
logo x e y seriam mpares, donde x
2
+ y
2
2 (mod 4), e logo
x
2
+y
2
nao pode ser o cubo de um inteiro.
Problemas Propostos
4.48. Determine todas as solu coes inteiras da equacao x
2
+y
2
= 2z
3
.
4.49. Mostre que se p um primo da forma 4k + 1 ent ao x
2
+ y
2
= pz
4
possui innitas solu coes com mdc(x, y) = 1.
4.50. Mostre que a equacao 25x
2
+ 14xy + 2y
2
= z
5
possui innitas
solu coes com mdc(x, y) = 1.
Agora sim voltemos a nosso problema original de calcular S
2
(c) e
T
2
(c).
176 CAP

ITULO 4. EQUA C

OES DIOFANTINAS N

AO LINEARES
Teorema 4.36. Sejam c um inteiro positivo e c = p

1
1
p

k
k
sua fato-
racao em fatores primos. Ent ao
S
2
(c) =
_

_
0 se c e par
0 se existe j com 1 j k tal que p
j
3 (mod 4)
2
k1
caso contrario.
Demonstra c

ao: Se c e par, nao pode ser escrito como soma de qua-


drados de paridades distintzas. No caso que c possui um divisor primo
da forma 4t +3 ja foi provado na proposicao 4.8 que c nao pode-se escre-
ver como soma de dois quadrados primos entre si. Assim vamos supor
que todos os divisores primos de c deixam resto 1 quando divididos por
4. Neste caso cada p
k
pode-se fatorar no anel de inteiros de Gau com
p
j
= p
j
p
j
, onde p
j
= a
j
+ ib
j
e p
j
= a
j
ib
j
s ao primos no anel dos
inteiros de Gau. Assim c se fatora nos inteiros de Gau como
c = p

1
1
p

1
1
p

2
2
p

2
2
p

k
1
p

k
1
.
Como queremos saber de quantas formas podemos escrever c como m
2
+
n
2
, isto e c = (m + in)(m in), com mdc(m, n) = 1, entao queremos
contar de quantas formas podemos distribuir os fatores de c em dois
fatores com a propridedade de que um sejam conjugado do outro, assim,
automaticamente no momento que escolhemos um primo para estar no
primeiro fator, seu conjugado tem que estar no segundo fator. Obser-
vemos tambem que um primo p
j
e seu conjugado p
j
nao podem estar
simultaneamente no mesmo fator, porque isso implicaria que p
j
dividi-
ria simultaneamente m + in e m in, e logo m e n nao seriam primos
entre si. Assim, o n umero de formas de escolher os fatores de m + in e
2
k
. Como a ordem dos fatores m + in e m in nao interessa, entao o
n umero de formas distintas de escrever c como soma de dois quadrados
e 2
k1
.
Agora estamos em condi coes de estudar o casos geral, isto e, contar
todas as triplas pitagoricas com hipotenusa xa.
Teorema 4.37. Sejam c um n umero inteiro positivo e
c = 2
n
p

1
1
p

k
k
q

1
1
q

l
l
sua fatoracao em fatores primos, onde os fatores p
i
sao congruentes a 1
modulo 4 e os fatores q
j
sao congruentes a 3 modulo 4. Ent ao
T
2
(c) =
1
2
(2
1
+ 1) (2
k
+ 1)
1
2
.
4.12. CONTANDO TRI

ANGULOS PITAG

ORICOS COM HIPOTENUSA FIXAD


Demonstra c

ao: Observemos que a funcao S


2
nao e multiplicativa,
mas e facil comprovar que a funcao

S denida por

S(n) =
_
1 se n = 1
2S
2
(n) se n > 1
e multiplicativa. Assim
T
2
(c) =

d|c
d=c
S
2
_
c
d
_
=
1
2
_

d|c

S
_
c
d
__

1
2
.
Usando o fato que

S e multiplicativa, temos que para todo primo p se
tem que
t

j=0

S(p
j
) =
_
1 se p = 2 ou p 3 (mod 4)
2t + 1 se p 1 (mod 4)
.
Destes dois fatos obtemos o que queramos mostrar.
No exemplo 4.23 exibimos 7 tri angulos com hipotenusa 325. De fato,
como 325 = 5
2
13, aplicando a formula obtida no teorema anterior
temos que T(325) =
1
2
(2 2 + 1)(2 1 + 1)
1
2
= 7, que esta em
concord ancia com o exemplo.
Vamos agora mostrar uma formula para U
2
(c):
Teorema 4.38. Sejam c um n umero inteiro positivo e
c = 2
n
p

1
1
p

k
k
q

1
1
q

l
l
sua fatoracao em fatores primos, onde os fatores p
i
sao congruentes a 1
modulo 4 e os fatores q
j
sao congruentes a 3 modulo 4. Se algum
j
e
mpar ent ao U
2
(c) = 0. Caso contrario, temos duas possibilidades:
Caso algum
i
seja mpar,
U
2
(c) =
1
2
(
1
+ 1) (
k
+ 1),
e caso todos os
i
sejam pares,
U
2
(c) =
1
2
((
1
+ 1) (
k
+ 1) + 1).
Demonstra c

ao: Cada p
k
pode-se fatorar no anel de inteiros de Gau
com p
j
= p
j
p
j
, onde p
j
= a
j
+ib
j
e p
j
= a
j
ib
j
s ao primos no anel dos
inteiros de Gau. Assim c se fatora nos inteiros de Gau como
c = i
n
(1 i)
n
p

1
1
p

1
1
p

2
2
p

2
2
p

k
1
p

k
1
q

1
1
q

l
l
.
178 CAP

ITULO 4. EQUA C

OES DIOFANTINAS N

AO LINEARES
Queremos saber de quantas formas podemos escrever c como u
2
+ v
2
,
isto e c = (u + iv)(u iv), com 0 u v, entao queremos contar
de quantas formas podemos distribuir os fatores de c em dois fatores
com a propridedade de que um sejam conjugado do outro. Para cada
i k, temos as
i
+ 1 escolhas 0, 1, ...,
1
para o expoente
1
de p
1
no
primeiro fator (sendo o expoente de p
1
no primeiro fator igual a
1

1
),
o que nos da (
1
+ 1) (
k
+ 1) escolhas. Mas se permutarmos esses
dois fatores conjugados de c obtemos a mesma solucao. Assim, devemos
dividir o n umero de escolhas por 2, exceto no caso em que algum fator
seja conjugado dele mesmo (m odulo multiplica cao por invertvel), mas
isso s o e possvel quando todos os
i
s ao pares, e nesse caso ha um unico
fator com essa propriedade, o que nos da a formula do enunciado.
Problemas Propostos
4.51. Encontre todos os tri angulos ret angulos com hipotenusa 330.
4.52. Quantos triplas pitagoricas tem hipotenusa igual a 5525?
4.53. Encontre o menor valor para c de tal forma que existam exata-
mente 28 tri angulos com hipotenusa igual a c.
4.13 Descenso Innito de Fermat
Ate agora foram estudadas essencialmente equa coes que principal-
mente possuem termos quadraticos. Existem metodos semelhantes ao
metodo geometrico, por exemplo em curvas do tipo y
2
= x
3
+ ax + b,
as chamadas curvas elpticas, mas, em geral, nao existe um metodo ge-
nerico para determinar se uma equa cao diofantina possui ou nao possui
solucao. De fato, em 1970 o matem atico russo Yuri Matiyasevich, com-
pletando trabalhos de Martin Davis, Julia Robinson e Hilary Putnam,
mostrou que nao pode existir um algoritmo para determinar se uma
dada equa cao diofantina possui ou nao possui solucao, dando resposta
negativa ao decimo problema de Hilbert.
Apesar disso, em alguns casos, e possvel mostrar que algumas equa-
coes diofantinas
f(x
1
, . . . , x
n
) = 0,
nao possuem solucao. O metodo do descenso innito (quando aplicavel)
permite mostrar que esta equa cao nao possui solucoes inteiras positivas
ou, sob certas condi coes, ate mesmo encontrar todas as suas solucoes
inteiras. Se o conjunto de solucoes de f
A = {(x
1
, . . . , x
n
) Z
n
| f(x
1
, . . . , x
n
) = 0}
4.13. DESCENSO INFINITO DE FERMAT 179
e diferente de vazio, entao gostaramos de considerar a solucao mnima
em certo sentido. Em outras palavras, queremos construir uma funcao
: A N e considerar a solucao (x
1
, . . . , x
n
) A com (x
1
, . . . , x
n
)
mnimo. O descenso consiste em obter, a partir desta solucao mnima,
uma ainda menor, o que nos conduz claramente a uma contradi cao,
provando que A de fato e vazio.
Para ilustrar este metodo consideremos o seguinte
Exemplo 4.39. Sejam a e b inteiros positivos tais que b < 2a + 1.
Mostrar que a equacao bx
2
2axy y
2
= 0 n ao possui solu coes inteiras
positivas.
Solu c

ao: Suponhamos que a equa cao possui solucao nao nula (m, n)
com m, n N, isto e, bm
2
2amn n
2
= 0 e escolhemos esta solucao
de tal forma que n seja mnimo. Como n
2
+2amn = bm
2
, somando aos
dois lados a
2
m
2
obtemos que
(n +am)
2
= (b +a
2
)m
2
< (a
2
+ 2a + 1)m
2
= (a + 1)
2
m
2
,
e assim n+am < am+m, isto, e n < m. Observemos que m(bm2an) =
n
2
, isto e, m e um divisor de n
2
, portanto n
2
= mq onde 0 < q < n < m
e q + 2an = bm. Por outro lado, multiplicando a equa cao original por b
obtemos
0 = bn
2
+2an(bm)(bm)
2
= bn
2
+2an(q+2an)(q+2an)
2
= bn
2
2anqq
2
,
logo o par (n, q) e tambem solucao da equa cao, o que contradiz a mini-
malidade da solucao anterior.
Como consequencia imediata do problema anterior, podemos mostrar
que se k e um inteiro positivo que nao e um quadrado perfeito, ent ao

k
e irracional. De fato, suponhamos por contradi cao que

k e racional e
seja a sua parte inteira. Logo existem inteiros nao nulos m, n tais que

k = a +
n
m
. Elevando ao quadrado e multiplicando por m
2
obtemos
(k a
2
)m
2
2amn n
2
= 0,
isto e, a equa cao (k a
2
)x
2
2axy y
2
= 0 possui uma solucao inteira
positiva, mas k < (a+1)
2
, portanto k a
2
< 2a+1, mas pelo problema
anterior a equa cao nao possui solucoes inteiras positivas, portanto

k
nao pode ser racional.
O seguinte e um exemplo dado por Fermat para aplicacao de seu
metodo, que em particular tem como consequencia que a equa cao de
Fermat nao possui solucoes nao triviais para expoentes m ultiplos de 4.
180 CAP

ITULO 4. EQUA C

OES DIOFANTINAS N

AO LINEARES
Exemplo 4.40 (Fermat). Demonstrar que a equacao x
4
+y
4
= z
2
n ao
possui solu coes inteiras positivas, isto e, n ao existem tripas pitagoricas
em que os dois catetos sejam quadrados perfeitos.
Solu c

ao: Suponhamos que x


4
+ y
4
= z
2
possui uma solucao inteira
com x, y, z > 0. Logo existe uma tal solucao (a, b, c) na qual c e m-
nimo. Em particular, temos que a e b s ao primos entre si, pois se
d = mdc(a, b) > 1 poderamos substituir (a, b, c) por (
a
d
,
b
d
,
c
d
2
) e ob-
ter uma solucao com c menor. De (a
2
)
2
+ (b
2
)
2
= c
2
temos portanto
que (a
2
, b
2
, c) e uma tripla pitagorica primitiva e assim existem inteiros
positivos m e n primos relativos tais que
a
2
= m
2
n
2
, b
2
= 2mn e c = m
2
+n
2
.
Da primeira equa cao temos que (a, n, m) e uma tripla pitagorica primi-
tiva e portanto m e mpar. Assim, de b
2
= 2mn conclumos que b, e
portanto n, e par. Observando ainda que b
2
= (2n)m e um quadrado
perfeito e mdc(2n, m) = 1, conclumos que tanto 2n como m s ao qua-
drados perfeitos, donde podemos encontrar inteiros positivos s e t tais
que
2n = 4s
2
e m = t
2
.
Por outro lado, dado que a
2
+n
2
= m
2
, entao existir ao inteiros positivos
i e j, primos entre si, tais que
a = i
2
j
2
, n = 2ij e m = i
2
+j
2
.
Portanto s
2
=
n
2
= ij, logo i e j serao quadrados perfeitos, digamos
i = u
2
e j = v
2
.
Logo temos que m = i
2
+j
2
, i = u
2
, j = v
2
e m = t
2
, assim
t
2
= u
4
+v
4
,
isto e, (u, v, t) e outra solucao da equa cao original. Porem
t t
2
= m m
2
< m
2
+n
2
= c
e t = 0 porque m e diferente de 0. Isto contradiz a minimalidade de c,
o que conclui a demonstra cao.
Observemos alem disso que, uma vez que esta equa cao nao possui so-
lu coes inteiras positivas, entao a equa cao x
4
+y
4
= z
4
e, mais geralmente
x
4n
+y
4n
= z
4n
, nao possuem solucoes inteiras positivas.
Exemplo 4.41. Mostre que a equacao x
4
2y
2
= 1 somente possui
solu coes triviais.
4.13. DESCENSO INFINITO DE FERMAT 181
Solu c

ao: Suponhamos que (a, b) e uma solucao da equa cao, assim


b
2
=
a
4
1
2
. Elevando ao quadrado obtemos
b
4
=
_
a
4
1
2
_
2
=
a
8
+ 2a
4
+ 1 4a
4
4
=
_
a
4
+ 1
2
_
2
a
4
,
logo a
4
+b
4
=
_
a
4
+1
2
_
2
e solucao da equa cao x
4
+y
4
= z
2
, que como foi
mostrado anteriormente, somente possui solucoes com xyz = 0, assim
a = 1 e b = 0 e a unica solucao.
No seguinte exemplo e usado o metodo do descenso de Fermat para
achar todas as solucoes a partir de uma solucao mnima.
Exemplo 4.42. Determine todos os pares de inteiros positivos (m, n)
tais que m|n
2
+ 1 e n|m
2
+ 1.
Solu c

ao: Observemos que qualquer solucao satisfaz mdc(m, n) =


mdc(n
2
+ 1, n) = 1, e assim a unica solucao com m = n e a solucao
(1, 1). Suponhamos que temos outra solucao com m = n e pela simetria
das condi coes do problema, podemos supor que m > n. Como m e n
dividem m
2
+ n
2
+ 1 e nao tem fator comum, entao
m
2
+n
2
+1
mn
= k N.
Assim temos a rela cao m
2
+n
2
+1kmn = 0, isto e, m e raiz do polin o-
mio quadratico X
2
knX+(n
2
+1) = 0. A outra solucao desta equa cao
quadratica pode ser calculada de duas formas distintas, j a que kn e a
soma das razes e n
2
+1 e o produto das razes. Assim esta segunda raiz
m

por um lado e igual a kn m, e portanto inteira, e por outro lado


tambem e igual a
n
2
+1
m
o que implica que e positiva. Observe alem disso
que
m

=
n
2
+ 1
m

n
2
+ 1
n + 1
= n 1 +
2
n + 1
n.
Por tanto se o par (m, n) e solucao do problema comm > n, entao (n, m

)
e solucao de
m

2
+n
2
+1
nm

= k com n m

, onde esta desigualdade e estrita


se n > 1. Assim, neste caso poderamos de novo aplicar este processo
para de novo encontrar uma solucao menor. Este processo s o nao pode
ser aplicado novamente quando n = 1 = m, e neste caso k = 3. Para
gerar todas as solucoes basta fazer o processo inverso. Assim, partindo
do par (1, 1) podemos construir todas as solucoes usando os termos da
recorrencia x
1
= 1, x
2
= 1 e x
j+1
= 3x
j
x
j1
, onde os pares
(1, 1), (2, 1), (5, 2), (13, 5), (34, 13), . . . , (x
j+1
, x
j
), . . .
s ao todas as solucoes do problema.
182 CAP

ITULO 4. EQUA C

OES DIOFANTINAS N

AO LINEARES
O metodo do descenso tambem pode ser usado para encontrar as
solucoes da equa cao de Markov.
Exemplo 4.43. Mostrar que a equacao diofantina
x
2
+y
2
+z
2
= 3xyz.
possui innitas solu coes inteiras positivas, e descreve-las.
Esta equacao e conhecida como equacao de Markov.
Solu c

ao: Por vericacao direta temos que (1, 1, 1) e (1, 1, 2) s ao so-


lu coes da equa cao. Alem disso, como a equa cao e simetrica, podemos
considerar, sem perda de generalidade, somente as solucoes com as co-
ordenadas x y z ordenadas de forma nao decrescente. Se z = y,
teremos x
2
= 3xy
2
2y
2
= (3x 2)y
2
y
2
x
2
, valendo as igualdades
se e s o se z = y = x = 1.
Assim suponhamos que (x, y, z) e uma solucao com x y < z e
portanto com z > 1. O polin omio quadratico
T
2
3xyT + (x
2
+y
2
) = 0
possui duas solucoes, e uma dela e z. Assim a outra e
z

= 3xy z =
x
2
+y
2
z
Z \ {0}.
Vejamos que z

y < z, e assim (z

, x, y) e tambem solucao (menor)


da equa cao de Markov. Para isto, suponhamos por contradi cao que
x
2
+y
2
z
= z

> y, isto e, yz < x


2
+ y
2
2y
2
, e em particular z < 2y.
Segue que
5y
2
> y
2
+z
2
= 3xyz x
2
= x(3yz x) xy(3z 1),
e portanto 5y > x(3z 1). Observemos que se x 2, entao 5y
2(3z 1) 5z, o que contradiz a hip otese y > z. Logo x = 1 e
1+y
2
y
z,
assim
1
y
+ y z > y. Portanto devemos ter temos
1
y
+ y = z, e neste
caso y = 1 e z = 2, o que nos da z

= 3xy z = 1 y, contradi cao.


Do fato anterior, temos que dada uma solucao da equa cao de Mar-
kov (x, y, z) com z 2 ,e sempre possvel encontrar uma solucao me-
nor (z

, x, y) e este processo somente para quando chegamos `a solucao


(1, 1, 1), isto e, estamos gerando uma arvore de solucoes da seguinte
forma:
4.13. DESCENSO INFINITO DE FERMAT 183
(1, 1, 1)
(1, 1, 2)
(1, 2, 5)
(1, 5, 13)
(1, 13, 34)
(1, 34, 89)
.
.
.
(13, 34, 1385)
.
.
.
(5, 13, 194)
(5, 194, 2897)
.
.
.
(13, 194, 7561)
.
.
.
(2, 5, 29)
(2, 29, 169)
.
.
.
(5, 29, 433)
.
.
.
onde de cada no (x, y, z), com x y z, saem duas novas solucoes
(x, z, 3xz y) e (y, z, 3yz x).
Problemas Propostos
4.54. Seja p um n umero primo e n um inteiro maior do que 1. Usar o
metodo do descenso innito para mostrar que
n

p e um n umero irracio-
nal.
4.55. Seja p um n umero primo. Mostrar que n ao existem inteiros posi-
tivos a, b e c tais que a
3
+pb
3
+p
2
c
3
= 0.
4.56. Pode um tri angulo ret angulo com lados inteiros ter area que seja
o quadrado de um inteiro?
4.57. Mostre que a equacao x
2
+ y
2
+ z
2
= x
2
y
2
n ao possui solu coes
inteiras positivas.
4.58. Mostre que n ao existem inteiros n ao nulos x, y, z, w tais que cum-
prem o sistema de equacoes
_
x
2
+y
2
= z
2
x
2
y
2
= w
2
.
4.59. Mostrar usando o metodo do descenso innito que a equacao x
4
+
y
4
= 2z
2
n ao tem solu coes n ao triviais.
184 CAP

ITULO 4. EQUA C

OES DIOFANTINAS N

AO LINEARES
4.60. Sejam a e b inteiros postitivos tais que ab divide a
2
+ b
2
+ 2.
Mostrar que
a
2
+b
2
+2
ab
= 4.
4.61. Sejam a e b inteiros positivos tais que ab+1 divide a
2
+b
2
. Mostrar
que o n umero
a
2
+b
2
ab + 1
e um quadrado perfeito.
4.62. Seja k um n umero inteiro distinto de 1 e 3. Mostrar que a equacao
x
2
+y
2
+z
2
= kxyz n ao possui solu coes inteiras positivas.
4.63. Determine todas as solu coes inteiras de x
4
2y
2
= 1.
Bibliograa
[1] F. E. Brochero Martinez, C. G. Moreira, N. C. Saldanha, E. Tengan
- Teoria dos N umeros: um passeio com primos e outros n umeros
familiares pelo mundo inteiro - Projeto Euclides, IMPA, 2010.
[2] A. Caminha. Equacoes diofantinas, Revista Eureka! No. 7, pp. 39-
48.
[3] T. W. Cusick e M. E. Flahive, The Marko and Lagrange spectra,
Math. Surveys and Monographs, no. 30, A.M.S. (1989).
[4] S. C. Coutinho, N umeros inteiros e criptograa RSA, Cole cao Com-
putacao e Matem atica, SBM e IMPA (2000).
[5] H. Cramer, On the order of magnitude of the dierence between
consecutive prime numbers, Acta Arithmetica 2: 2346 (1936).
[6] Lorenzo J. Daz, Danielle de Rezende Jorge, Uma introducao aos
Sistemas Dinamicos via Fracoes Contnuas, 26
o
Coloquio Brasileiro
de Matem atica, IMPA (2007).
[7] A. Hefez. Elementos de Aritmetica, 2a. edicao. Textos Universita-
rios, SBM (2005).
[8] C. G. Moreira, O teorema de Ramsey, Revista Eureka! 6, 2329.
[9] C. G. Moreira, Geometric properties of the Markov and Lagrange
spectra. Preprint-IMPA-2009.
[10] D. H. J. Polymath, Deterministic methods to nd primes, preprint,
http://polymathprojects.files.wordpress.com/2010/07/
polymath.pdf; veja tambem http://polymathprojects.org/
2009/08/09/research-thread-ii-deterministic-way-to-
find-primes/ e http://michaelnielsen.org/polymath1/
index.php? title=Finding_primes
185
186 BIBLIOGRAFIA
[11] A. Politi, J. C. F. Matthews, J. L. OBrien, Shors Quantum Fac-
toring Algorithm on a Photonic Chip, Science 4 September 2009:
Vol. 325. no. 5945, p. 1221.
[12] P. Ribenboim, Selling primes, Math. Mag. 68 (1995), 175182. Tra-
duzido como Vendendo primos, Rev. Mat. Univ. 22/23 (1997), 113.
[13] J.P.O. Santos. Introducao `a Teoria dos N umeros, 3a. edicao. Cole-
cao Matem atica Universitaria, IMPA (2010).
[14] J.P. Serre, On a theorem of Jordan, Bull. Amer. Math. Soc. (N.S.)
40 (2003), no. 4, 429440.
[15] A. Shen e N. K. Vereshchagin, Basic Set Theory, AMS, 2002.
[16] P. W. Shor, Polynomial-Time Algorithms for Prime Factoriza-
tion and Discrete Logarithms on a Quantum Computer, SIAM
J. Comput. 26 (5), 1484-1509 (1997). Tambem em arXiv:quant-
ph/9508027v2.

Das könnte Ihnen auch gefallen